3.3.10

mrcp gp question

1. A 67-year-old man with lung cancer is currently taking MST 30mg bd for pain relief. What dose of oral morphine solution should he be prescribed for breakthrough pain?

A. 5 mg

B. 10 mg √`

C. 15 mg

D. 20 mg

E. 30 mg

2. A 54-year-old male with no past medical history is found to be in atrial fibrillation during a consultation regarding a sprained ankle. He reports no history of palpitations or dyspnoea. After discussing treatment options he elects not to be cardioverted. If the patient remains in chronic atrial fibrillation what is the most suitable treatment to offer?

A. Aspirin √

B. Warfarin, target INR 2-3

C. No anticoagulation

D. Warfarin, target INR 3-4

E. Warfarin, target INR 2-3 for six months then aspirin

3. A 19-year-old man presents with a compound fracture of his leg following a fall from scaffolding. Examination reveals soiling of the wound with mud. He is sure he has had five previous tetanus vaccinations. What is the most appropriate course of action to prevent the development of tetanus?

A. Clean wound + intramuscular human tetanus immunoglobulin √

B. Clean wound + tetanus vaccine

C. Clean wound + tetanus vaccine + intramuscular human tetanus immunoglobulin

D. Clean wound + tetanus vaccine + benzylpenicillin

E. Clean woundia

4. A 55-year-old man is admitted following an anterior myocardial infarction. Which of the following drugs is least likely to reduce mortality in the long-term?

A. Atorvastatin

B. Atenolol

C. Ramipril

D. Aspirinia

E. Isosorbide mononitrate √

5. A 19-year-old male with a history of asthma presents to the surgery due to shortness of breath. On examination his peak expiratory flow is 270 l/min (usual 600 l/min). Pulse is 96 bpm and the respiratory rate is 24 / min. Examination of the chest reveals a bilateral expiratory wheeze but is otherwise unremarkable. What is the most appropriate management?

A. Oxygen + nebulised salbutamol + prednisolone arrange immediate admission to A&E via ambulance

B. Nebulised salbutamol + advise to double inhaled steroids + allow home if settles with follow-up review

C. Oxygen + nebulised salbutamol + prednisolone arrange immediate admission to medical team via ambulance

A. Oxygen + nebulised salbutamol + prednisolone and review following treatment

B. Nebulised salbutamol + prednisolone + allow home if settles with follow-up review

6. A 78-year-old women is discharged following a fractured neck of femur. On review she is making good progress but consideration is given to secondary prevention of further fractures. What is the most suitable management?

A. Arrange DEXA scan + start strontium ranelate if T-score < -2.5 SD

B. Start oral bisphosphonate √

C. Arrange DEXA scan + start oral bisphosphonate if T-score < -1.0 SD

D. Arrange DEXA scan + start hormone replacement therapy if T-score < -2.5 SD

E. Arrange DEXA scan + start oral bisphosphonate if T-score < -1.5 SD

7. A 72-year-old man presents with lethargy and palpitations for the past four or five days. On examination his pulse is 123 bpm irregularly irregular, blood pressure is 128/78 mmHg and his chest is clear. An ECG confirms atrial fibrillation. What is the appropriate drug to control his heart rate?

A. Amiodarone

B. Atenolol √

C. Digoxin

D. Amlodipine

E. Flecainide

8. A 56-year-old man with metastatic prostate cancer comes for review. He is known to have spinal metastases but until now has not had any significant problems with pain control. Unfortunately he is now getting regular back pain despite taking paracetamol 1g qds. Neurological examination is unremarkable. What is the most appropriate next step?



A. Switch to co-codamol 30/500

B. Refer for radiotherapy

C. Add oral bisphosphonate

D. Add diclofenac √

E. Add dexamethasone



9. A 46-year-old man with suspected diabetes mellitus has an oral glucose tolerance test, following the standard WHO protocol. The following results are obtained:following results are obtained:

Time (hours) Blood glucose (mmol/l)

0 5.7

2 7.6

How should these results be interpreted?

A. Normal √

B. Impaired fasting glucose and impaired glucose tolerance

C. Diabetes mellitus

D. Impaired glucose tolerance

E. Impaired fasting glucose

Theme: Antibiotic guidelines

A. Erythromycin

B. Trimethoprim or nitrofurantoin or amoxicillin or cephalosporin

C. Quinolone or trimethoprim

D. Trimethoprim or vancomycin

E. Amoxicillin or tetracycline or erythromycin

F. Phenoxymethylpenicillin + flucloxacillin

G. Flucloxacillin

H. Amoxicillin or cephalosporin or erythromycin

I. Doxycycline

J. Broad-spectrum cephalosporin or quinolone

For each one of the following please select the most complete answer from the options above:ia

10. Infective exacerbation of COPD Amoxicillin or tetracycline or erythromycin

11. Acute pyelonephritis Broad-spectrum cephalosporin or quinolone

12. Cellulitis Phenoxymethylpenicillin + flucloxacillin

13. A 54-year-old man with stage 4 chronic kidney disease presents for review. Which one of the following drugs is it most important to avoid?

A. Erythromycin

B. Diazepam

C. Rifampicin

D. Tetracycline √

E. Warfarin

14. A 54-year-old man collapses in the surgery during a nurse-led diabetes clinic. He is not breathing and a carotid pulse cannot be felt. What is the correct ratio of chest compressions to ventilation?

A. 5:1

B. 10:1

C. 15:2

D. 20:2

E. 30:2 √

15. A 62-year-old woman with recently diagnosed chronic obstructive pulmonary disease (COPD) presents for review. Her FEV1 is 65% of the predicted value. She has managed to give up smoking and was prescribed a salbutamol inhaler to use as required. Despite this she is still symptomatic and complains of wheeze and shortness of breath. What is the most appropriate next step?

A. Add inhaled corticosteroid

B. Add long-acting anti-cholinergic inhaler √

C. Refer for consideration of long-term oxygen therapy

D. Add oral theophylline

E. Add combination long-acting beta2-agonist and corticosteroid inhaler

16. A 28-year-old woman is diagnosed with constipation predominant irritable bowel syndrome. She occasionally experiences spasms of pain in the left iliac fossa. Which one of the following is least likely to help her symptoms?

A. Mebeverine

B. Ispaghula

C. Methylcellulose

D. Sterculia

E. Lactulose √

17. A 34-year-old postman attends the Surgery following a dog bite to his right hand. What is the most appropriate antibiotic therapy?



A. Metronidazole + amoxicillin

B. Erythromycin

C. Co-amoxiclav √

D. Metronidazole

E. Flucloxacillin + penicillin





18. A 54-year-old man with a history of hypertension comes for review. He currently takes lisinopril 10mg od, simvastatin 40mg on and aspirin 75mg od. His blood pressure is well controlled at 124/76 mmHg but he also mentions that he is due to have a tooth extraction next week. What advice should be given with regards to his aspirin use?

A. Take aspirin as normal but take tranexamic 1g tds acid 24 hours before and after procedure

B. Stop 72 hours before, restart 24 hours after procedure

C. Stop 24 hours before, restart 12 hours after procedure

D. Take aspirin as normal √

E. Stop 48 hours before, restart 24 hours after procedure

19. A 35-year-old homosexual man is referred to the local genitourinary clinic following the development of a solitary painless penile ulcer associated with painful inguinal lymphadenopathy. He has recently developed rectal pain and tenesmus. What is the most likely diagnosis?

A. Herpes simplex infection

B. Syphilis

C. Granuloma inguinale

D. Chancroid

E. Lymphogranuloma venereum √

20. A 72-year-old woman presents with polyuria and polydipsia. Investigations reveal the following:

Fasting glucose 4.5 mmol/l Calcium 2.88 mmol/l Phosphate 0.75 mmol/l

Parathyroid hormone 6 pmol/L (normal range = 0.8 - 8.5)

What is the most likely underlying diagnosis?

A. Myeloma

B. Sarcoidosis

C. Primary hyperparathyroidism √

D. Vitamin D excess

E. Osteomalacia

21. A 41-year-old man presents to his GP with a two week history of headaches around the left side of his face associated with watery eyes. He describes having about two episodes a day each lasting around 30 minutes. What is the likely diagnosis?

A. Migraine

B. Cluster headache √

C. Trigeminal neuralgia

D. Acute angle closure glaucoma

E. Meningioma

Theme: Drug monitoring



A. U&E, LFT

B. TFT, LFT

C. TFT, U&E

D. LFT

E. FBC, U&E

F. FBC, LFT, U&E

G. U&E

H. Plasma glucose

I. ECG

J. No routine monitoring required



For each of the following drugs select the most appropriate monitoring tests once treatment has commenced

22. Methotrexate FBC, LFT, U&E

23. Carbamazepine No routine monitoring required

24. Amiodarone TFT, LFT

25. A 56-year-old man is reviewed in the Cardiology outpatient clinic following a myocardial infarction one year previously. During his admission he was found to be hypertensive and diabetic. He complains that he has put on 5kg in weight in the past 6 months. Which of his medications may be contributing to his weight gain?

A. Metformin

B. Losartan

C. Clopidogrel

D. Gliclazide √

E. Simvastatin

26. Which one of the following is not a risk factor for developing osteoporosis?

A. Smoking

B. Obesity √

C. Sedentary lifestyle

D. Premature menopause

E. Female sex





27. A 64-year-old man with a history of Parkinson's disease is reviewed in clinic and a decision has been made to start him on cabergoline. Which one of the following adverse effects is most strongly associated with this drug?

A. Optic neuritis

B. Transient rise in liver function tests

C. Pulmonary fibrosis √

D. Renal failure

E. Thrombocytopenia

28. A 24-year-old female presents with episodic wheezing and shortness of breath for the past 4 months. She has smoked for the past 8 years and has a history of eczema. Examination of her chest is unremarkable. Spirometry is arranged and is reported as normal. What is the most appropriate management of her symptoms?ia

A. Peak flow diary

B. Trial of lansoprazole

C. Baseline FEV1 repeated following inhaled corticosteroids

D. Arrange a chest x-ray

K. Trial of salbutamol inhaler √

29. A syringe driver is set-up for a terminal ill patient. Which one of the following combination of drugs should not be mixed in the same syringe?

A. Diamorphine + midazolam + hyoscine hydrobromide

B. Diamorphine + haloperidol + cyclizine

C. Diamorphine + cyclizine + hyoscine hydrobromide

D. Diamorphine + levomepromazine + hyoscine butylbromide

E. All the above combinations can be safely combined in the same syringe driver

30. A 35-year-old female presents with abdominkolal pain associated with bloating for the past 6 months, Which one of the following symptoms is least associated with a diagnosis of irritable bowel syndrome?

A. Feeling of incomplete stool evacuation

B. Weight loss √

C. Back pain

D. Lethargy

E. Nausea

31. A 68-year old woman is found to have the following blood tests:

TSH = 0.05 mu/l Free T4 = 19 pmol/l (range 9-25 pmol/l)

Free T3 = 7 pmol/l (range 3-9 pmol/l)

If left untreated, what are the most likely possible consequences?

A. Supraventricular arrhythmias and osteoporosis √

B. Supraventricular arrhythmias and hyperlipidaem

C. Hypothyroidism and impaired glucose tolerance

D. Myasthenia gravis and hypothyroidism

E. Impaired glucose tolerance and hyperlipidaemia

32. A 68-year old woman is found to have the following blood tests:

TSH = 0.05 mu/l Free T4 = 19 pmol/l (range 9-25 pmol/l) Free T3 = 7 pmol/l (range 3-9 pmol/l) If If the patient is taking amodiaron and also he is taking medicine for IHD then what would be the option?

A. Continue Amodiarone + add Carbimazole

B. Stop Amodiaron + start carbimazol √

C. Stop amodiaron + Start (Carbimazol + Flicanide)

D. Continue Amodiarone and add Properanolol

E. Keep the current drug regimn

33. A 65 year old female with a known history of heart failure presents to her GP for an annual check-up. She is found to have a blood pressure of 170/100 mmHg. Her current medications are furosemide and aspirin. What is the most appropriate medication to add? a

A. Bendroflumethiazide

B. Spironolactone

C. Bisoprolo

D. Verapamil

E. Enalapril √

34. A 64-year-old man is using neuroleptics and presented with symptoms of Parkinson's disease is reviewed in clinic. Which of the following drugs least suitable?

A. Procyclidine

B. Benzotropine

C. Trihexyphenidyl (Benzhexol)

D. Levodopa √

E. Apomorphine


35. A home visit is requested by the husband of a 71-year-old woman who is 'off her legs'. On arriving the patient states that since mid-morning her left arm has felt weak and a degree of facial asymmetry is noted when she smiles. She is normally fit and well other than a past history of hypertension for which she takes ramipril. What is the most appropriate action?

A. Arrange same-day medical admission

B. Dial 999 for emergency admission √

C. Arrange review at rapid access TIA clinic

D. Arrange review at rapid access TIA clinic + give aspirin 300 mg

E. Dial 999 for emergency admission + give aspirin 300 mg

36. A Patient, known case of Myasthenia Gravis is given Gentamycin & Flucloxacillin for treatment of bacterial infection He developed muscular weakness after the first dose of the antibiotics what could be the cause?

A. Gullian Barre Syndrome

B. Brain abscess

C. Neuromuscular blockade √

D. Ischemic stroke

E. Multiple sclerosis

Theme: Shortness of breath



A. Heart failure

B. Recurrent pulmonary emboli

C. Lung cancer

D. Obesity

E. Pulmonary fibrosis

F. Anaemia

G. Asthma

H. Chronic obstructive pulmonary disease

I. Bronchiectasis

J. Aortic stenosis



For each one of the following scenarios please select the most likely diagnosis:

37. A 54-year-old obese woman presents with shortness of breath. She currently uses HRT and smokes 20 per day. Chest auscultation is unremarkable. Spirometry shows an obstructive pattern with no reversibility to bronchodilators. Chronic obstructive pulmonary disease

38. A 62-year-old man who is currently being treated for colorectal cancer presents with progressive shortness of breath over the past 2 months. Respiratory examination is unremarkable other than a respiratory rate of 24 / min. He is also noted to have a raised JVP and a pulse rate of 96 / min. Recurrent pulmonary emboli

39. A 67-year-old man presents with increasing shortness of breath. His symptoms are worse at night. A third heart sound is noted on examination. Heart failure

40. A 47-year-old man is reviewed in the smoking cessation clinic. Which one of the following conditions would contraindicate the prescription of bupropion?

A. History of supraventricular tachycardia

B. Previous episodes of acute pancreatitis

C. Epilepsy √

D. Depression

E. Hypertension

41. A 58-year-old man with no past medical history of note is admitted to hospital with crushing central chest pain. ECG on arrival shows anterior ST elevation and he is subsequently thrombolysed with a good resolution of symptoms and ECG changes. Two months following discharge from hospital, which combination of drugs should he be taking?

A. ACE inhibitor + beta-blocker + statin + aspirin √

B. Spironolactone + beta-blocker + statin + aspirin

C. ACE inhibitor + beta-blocker + statin + aspirin + clopidogrel

D. ACE inhibitor + statin + aspirin + clopidogrel

E. Beta-blocker + statin + aspirin + clopidogrel

42. A 63-year-old man is diagnosed as having restless legs syndrome. What is the most relevant blood test to perform?

A. ESR

B. Ferritin √

C. Blood glucose

D. Urea and electrolytes

E. Liver function tests

43. A 52-year-old woman with a history of hypothyroidism presents with lethargy and a sore tongue. Blood tests are reported as follows:

Hb 10.7 g/dl MCV 121 fl Plt = 177 * 109/l WBC 5.4 * 109/l

Further tests are ordered:

Vitamin B12 64 ng/l (200-900 ng/l) Folic acid 7.2 nmol/l (> 3.0 nmol/l)

Antibodies to intrinsic factor are also found.

What is the most appropriate management?

A. 1 mg of IM hydroxocobalamin once every 3 months

B. 1 mg of IM hydroxocobalamin 3 times each week for 2 weeks, then once every 3 months √

C. 1 mg of IM hydroxocobalamin once every 2 months + folic acid 5mg OD

D. Give folic acid 5mg od one week then recheck bloods

E. 1 mg of IM hydroxocobalamin 3 times each week for 2 weeks, then once every 3 months + folic acid 5mg OD

44. A patient is noted to have an absent ankle reflex. Which nerve root does this correspond to?

A. L2-L3

B. L3-L4

C. S3-S4

D. S1-S2 √

E. L1-L2

45. A patient is noted to have an Foot drop. What is the cause?

A. L2-L3 Nerve root compression

B. L3-L4 Nerve root compression

C. S3-S4 Nerve root compression

D. Common peroneal nerve √

E. L1-L2 Nerve root compression

46. A 35-year-old man who is usually fit and well presents to his GP with a 2 month history of indigestion. His weight is stable and there is no history of dysphagia. Examination of the abdomen is unremarkable. Of the following options, what is the most suitable initial management?

A. Urea breath testing and non-urgent referral for endoscopyia

B. H pylori eradication therapy and full-dose proton pump inhibitor for three monthsia

C. Full-dose Proton pump inhibitor and immediate referral for endoscopyia

D. Three month course of a standard-dose proton pump inhibitoria

E. One month course of a full-dose proton pump inhibitoria √

47. A 54-year-old woman presents with jaundice shortly after being discharged from hospital. Liver function tests are reported as follows:

Albumin 49 g/l Bilirubin 89 µmol/l Alanine transferase (ALT) 66 iu/l

Alkaline phosphatase (ALP) 245 µmol/l Gamma glutamyl transferase (yGT) 529 u/l

Which of the following antibiotics is she most likely to have received?

A. Co-amoxiclav √

B. Gentamicin

C. Ciprofloxacin

D. Trimethoprim

E. Ceftazidime

48. A 72-year-old female known to have osteoporosis is started on alendronate. Which one of the following side-effects is it most important to warn her about?

A. Sore throat

B. Heartburn √

C. Headache

D. Diarrhoea

E. Palpitations

49. A 79-year-old female with a history of COPD and metastatic lung cancer is admitted with increasing shortness of breath. Following discussion with family it is decided to withdraw active treatment, including fluids and antibiotics, as the admission likely represents a terminal event. Two days after admission she becomes agitated and restless. What is the most appropriate management?

A. Subcutaneous midazolam √

B. Intramuscular haloperidol

C. Oral lormetazepam

D. Oral haloperidol

E. Recommence fluids and antibiotics

Theme: Epilepsy: treatment



A. Clonazepam

B. Ethosuximide

C. Levetiracetam

D. Sodium valproate

E. Phenytoin

F. Lamotrigine

G. Gabapentin

H. Carbamazepine



For each one of the following select the most appropriate medication from the list above:

50. A 24-year-old man with complex partial seizures Carbamazepine

51. First-line anti-epileptic in a 17-year-old girl with tonic-clonic seizures. She has the Depo-Provera injection for contraception. Sodium valproate

52. Useful in patients with absence seizures who are intolerant of sodium valproate Ethosuximide





53. A 68-year-old female presents to her GP with a two week history of intermittent headaches and lethargy. Blood tests reveal the following: ESR 67 mm/hr What is the most likely diagnosis?

A. Polymyalgia rheumatic

B. Cluster headaches

C. Polyarteritis nodosa

D. Migraine

E. Temporal arteritis √

54. A 31-year-old woman who initially presented with abdominal pain and constipation is diagnosed with irritable bowel syndrome. Which one of the following bits of dietary advice is it least suitable to give?

A. Avoid missing meals

B. Restrict tea and coffee to 3 cups per day

C. Increase the intake of fibre such as bran and wholemeal bread √

D. Reduce intake of alcohol

E. Drink at least 8 cups of fluid per day

55. A 54-year-old woman presents systemically unwell. She has recently started carbimazole for hyperthyroidism. What is the most important blood test to perform?

A. Liver function tests

B. Full blood count √

C. Prothrombin time

D. Urea and electrolytes

E. Cortisol

56. A 35-year-old woman presents concerned about her weight. Her body mass index is 34 kg/m2. Which one of the following best describes her weight?

A. Obese (Obese I) √

B. Morbidly obese (Obese III)

C. Overweight

D. Clinically obese (Obese II)

E. Normal

57. A 63-year-old female on long-term warfarin for atrial fibrillation attends the anticoagulation clinic. Despite having a stable INR for the past 4 years on the same dose of warfarin her INR is measured at 5.4. Which one of the following is most likely to be responsible?

A. St John's Wort

B. Smoking

C. Carrot juice

D. Cranberry juice √

E. Camomile tea

58. A 72-year-old women is reviewed following a course of oral flucloxacillin for right lower limb cellulitis. The local protocol suggest oral clindamycin should be used next-line. Which one of the following side-effects is it most important to warn her about?

A. Heartburn or indigestion

B. Jaundice

C. Sore throat, bruising or lethargy

D. Avoid any food or drink containing alcohol

E. Diarrhoea √

59. A 30-year-old man comes to surgery. He has been handed a slip from an ex-girlfriend stating she has tested positive for Chlamydia. He last slept with her 2 months ago. He has no symptoms of note, in particular no dysuria or discharge. What is the most appropriate management?

A. Reassure symptoms would have presented by now

B. Offer antibiotic therapy

C. Offer Chlamydia testing and antibiotic treatment immediately without waiting for the results √

D. Offer Chlamydia testing and antibiotic treatment if positive

E. Notify public health

60. A 24-year-old female presents to her GP due to increased frequency of migraine attacks. She is now having around four migraines per month. Which type of medication would it be most appropriate to prescribe to reduce the frequency of migraine attacks?

A. Specific 5-HT2 agonist

B. 5-HT1 antagonist

C. Tricyclic antidepressant

D. Beta-blocker √

E. Specific 5-HT1 agonist





61. Q 49 A 62-year-old man with a history of diabetes mellitus presents to surgery complaining of a heavy feeling in his chest for the past 2 hours. An ECG is taken:

What is the most likely diagnosis?


A. Posterior myocardial infarction

B. Anterolateral myocardial infarction

C. Incorrectly placed ECG leads

D. Acute pericarditis

E. Inferior myocardial infarction √









Theme: Side-effects of anti-hypertensives drugs



A. Hyponatraemia

B. Reduced seizure threshold

C. Hypocalcaem

D. Cold peripheries

E. Drug-induced lupus

F. Hypernatraem

G. Ankle oedema



For each one of the following drugs choose the side-effect most characteristically associated with itia

62. Amlodipine Ankle oedema

63. Bendroflumethiazide Hyponatraemia

64. Atenolol Cold peripheries

65. You are asked to attend a meeting at a local nursing home. There is currently an increased incidence of MRSA in the patients and a strategy is being drawn up to tackle this. What is the most effective single step to reduce the incidence of MRSA?

A. The use of personal protective equipment for staff including gloves and aprons

B. Hand hygiene √

C. Screening patients for MRSA on admission

D. Cohort nursing

E. Limiting the number of visitors

66. A 69-year-old man with terminal lung cancer is reviewed. He currently takes MST 60mg bd for pain. He has become unable to take oral medications and a decision is made to set-up a syringe driver. What dose of diamorphine should be prescribed for the syringe driver?

A. 60 mg

B. 40 mg 1/3 Total dose√

C. 120 mg

D. 30 mg

E. 20 mg

67. A 23-year-old man wakes up on a Sunday morning unable to extend his wrist . He had been drinking heavily the previous night. What is the likely cause of his weakness?

A. Wernicke's encephalopathy

B. Radial nerve palsy √

C. Subacute combined degeneration of the cord

D. Acute B12 deficiency

E. Ulnar nerve palsy

68. A 43-year-old man with type 2 diabetes mellitus presents with lethargy. His current medications include metformin and gliclazide, although the gliclazide may soon be stopped due to his obesity. A number of blood tests are ordered which reveal the following: HbA1c = 8.2%; Ferritin = 204 ng/ml; Bilirubin = 23 µmol/l; ALP=162 u/l; ALT=120u/l; AST=109 u/l On discussing these results he states that he does not drink alcohol. What is the most likely cause of these abnormal results?



Metformin-induced steatohepatitis
Haemochromatosis
Acute hepatitis secondary to gliclazide
Cryptogenic cirrhosis
Non-alcoholic fatty liver disease √


69. A 70-year-old woman is reviewed. She sustained a fracture of her wrist one year ago, following which a DEXA scan was performed. This showed a T-score of -2.8 SD. Calcium and vitamin D supplementation was started along with oral alendronate. This however was stopped due to oesophagitis. In accordance with NICE guidelines, what is the most suitable next management step?

A. Start hormone replacement therapy

B. Start raloxifene

C. Start teriparatide

D. Switch to risedronate √

E. Refer for hip protectors

70. What is the minimum steroid intake a patient should be taking before an assessment of osteoporosis prophylaxis should be undertaken?

A. Equivalent of prednisolone 10 mg or more each day for 6 months

B. Equivalent of prednisolone 7.5 mg or more each day for 6 weeks

C. Equivalent of prednisolone 5 mg or more each day for 6 weeks

D. Equivalent of prednisolone 7.5 mg or more each day for 3 months √

E. Equivalent of prednisolone 10 mg or more each day for 6 weeks

71. Which one of the following statements regarding NICE guidance on the primary prevention of cardiovascular disease is incorrect?

A. Premature coronary heart disease is defined as < 65 years in females

B. A 10-year risk of 15% is used to identify patients who should be considered for lipid-lowering therapy √

C. Simvastatin 40mg on is the first line treatment in patients with a significant risk

D. The 1991 Framingham equations are still recommended for calculating risk

E. If a patient has a first degree relative with premature heart disease the risk should be multiplied by 1.5

72. A 62-year-old heavy smoker presents with shortness of breath and a morning cough. A chest x-ray shows hyperinflated lung fields. Spirometry is organized. Which one of the following set of results would be most consistent with a diagnosis of chronic obstructive pulmonary disease?

A. FEV1 - reduced, FEV1/FVC - normal

B. FEV1 - increased, FEV1/FVC - reduced

C. FEV1 - reduced, FEV1/FVC - reduced √

D. FEV1 - normal, FEV1/FVC - reduced

E. FEV1 - reduced, FEV1/FVC - increased

73. A 54-year-old man is investigated for dyspepsia. An endoscopy shows a gastric ulcer and a CLO test done during the procedure demonstrates H. pylori infection. A course of H. pylori eradication therapy is given. What is the most appropriate test to confirm eradication?

A. Culture of gastric biopsy

B. pylori serology

C. Hydrogen breath test

D. Urea breath test √

E. Stool culture

Theme: Hypertension: secondary causes



A. Adult polycystic kidney disease

B. Cushing's syndrome

C. Congenital adrenal hyperplasia

D. Primary hyperaldosteronism

E. Phaeochromocytoma

F. Acromegaly

G. Liddle's syndrome

H. Medication-induced

I. Renal artery stenosis

J. Pregnancy-induced hypertension



For each one of the following select the most likely diagnosis:

74. A 28-year-old who is 10 weeks pregnant is noted to be hypertensive on her booking visit. Blood show a potassium of 3.1 mmol/l. Clinical examination is unremarkable Primary hyperaldosteronism

75. A 39-year-old man presents with headaches and excessive sweating. He also reports some visual loss. Visual fields testing reveal loss of temporal vision on the right side. Acromegaly

76. A 68-year-old with a history of ischaemic heart disease is seen in hypertension clinic. Despite four antihypertensives his blood pressure is 172/94 mmHg. An abdominal ultrasound shows asymmetrical kidneys Renal artery stenosis

77. A 24-year-old female comes for review. She was diagnosed with asthma two years ago and is currently using a salbutamol inhaler 100mcg prn combined with beclometasone dipropionate inhaler 200mcg bd. Despite this her asthma is not well controlled. On examination her chest is clear and she has a good inhaler technique. What is the most appropriate next step in management?

A. Increase beclometasone dipropionate to 400mcg bd

B. Switch steroid to fluticasone propionate

C. Trial of leukotriene receptor antagonist

D. Add salmeterol √

E. Add tiotropium

78. A 44-year-old man is diagnosed with a duodenal ulcer. CLO testing performed during the gastroscopy is positive for Helicobacter pylori. What is the most appropriate management to eradicate Helicobacter pylori?

A. Lansoprazole + clindamycin + metronidazole

B. Lansoprazole + amoxicillin + clindamycin

C. Lansoprazole + amoxicillin + clarithromycin √

D. Omeprazole + amoxicillin + clindamycin

E. Omeprazole + penicillin + metronidazole

79. A 79-year-old woman is reviewed. She has taken bendroflumethiazide 2.5mg od for the past 10 years for hypertension. Her current blood pressure is 150/94 mmHg. Clinical examination is otherwise unremarkable. An echocardiogram from two months ago is reported as follows:

Ejection fraction 48%, moderate left ventricular hypertrophy. Minimal MR noted

What is the most appropriate next step in management?

A. Increase bendroflumethiazide to 5mg od

B. Stop bendroflumethiazide + start frusemide 40mg od

C. Add ramipril 1.25mg od √

D. Stop bendroflumethiazide + start ramipril 1.25mg od

E. Add amlodipine 5mg od

80. A 35-year-old man with a known history of peanut allergy is presents to the surgery with a swollen face. On examination blood pressure is 85/60 mmHg, pulse 120 bpm and there is a bilateral expiratory wheeze. What is the most appropriate form of adrenaline to give?

A. 10ml 1:10,000 IV

B. 0.5ml 1:1,000 IM √

C. 0.5ml 1:10,000 IM

D. 5ml 1:1,000 IM

E. Nebulised adrenaline

81. A 34-year-old female presents to her GP due to a number of 'funny-dos'. She describes a sensation that her surroundings are unreal, 'like a dream'. Following this she has been told that she starts to smack her lips, although she has no recollection of doing this. What is the most likely diagnosis?

A. Myoclonic seizure

B. Simple partial seizure

C. Complex partial seizure √

D. Partial seizure progressing to generalised seizure

E. Absence seizure

82. A 54-year-old obese man presents with lethargy and polyuria. A fasting blood sugar is requested:

Fasting glucose 8.4 mmol/l

He is given dietary advice and a decision is made to start metformin.

What is the most appropriate prescription?

A. Metformin 500mg od with food for 5 days then metformin 500mg bd for 5 days then metformin 500mg tds for 20 days then review

B. Metformin 500mg tds with food

C. Metformin 500mg od with food for 14 days then metformin 500mg bd for 14 days then review √

D. Metformin 1g tds with food

E. Metformin 500mg tds taken at least 1 hour before meals

83. A 36-year-old female with a BMI of 34 kg/m^2 presents to her GP after managing to lose 3 kg in the past month. She asks about the possibility of starting a drug to help her lose weight. What is the primary mode of action of orilistat?

A. Leptin antagonist

B. Pancreatic lipase inhibitor √

C. Prevents intestinal absorption of low-density lipoproteins

D. HMG-CoA reductase inhibitor

E. Centrally-acting appetite suppressant

84. For a patient undergoing an elective splenectomy, when is the optimal time to give the pneumococcal vaccine?

A. Two weeks before surgery √

B. One week before surgery

C. Immediately following surgery

D. Two weeks after surgery

E. At least one month after surgery

85. A 67-year-old woman presents with lethargy, depression and constipation. A set of screening blood tests reveals the following: Calcium = 3.05 mmol/l Albumin = 41 g/l

What is the single most useful test for determining the cause of her hypercalcaemia?



A. ESR

B. Phosphate

C. Vitamin D level

D. Parathyroid hormone √

E. ACE level



Theme: NICE hypertension guidelines

The diagram below is taken from the NICE guidelines on the management of hypertension:



A. ACEI’s

B. Afrocrabian/Black

C. Calcium Channel Blockers

D. 55Years

E. Specialist Advice

F. Thiazide Diuretics

G. Alfa Blocker

H. β Blocker




For each of the numbered gaps above, please select one option from the list:

86. Gap (1) 55yrs

87. Gap (2) Afrocrabian/Black

88. Gap (3) ACEI’s

89. Gap (4) Calcium channel Blocker or Thiazide Diuretic

90. Gap (5) ACEI’s + (Calcium channel Blocker or Thiazide Diuretic)

91. Gap (6) ACEI’s + Calcium channel Blocker + Thiazide Diuretic

92. Gap (7) Alfa Blocker

93. Gap (8) β Blocker

94. Gap (9) Specialist Advice

95. A 31-year-old man with ulcerative colitis presents with a worsening of his symptoms. He is passing around four loose stools a day which do not contain blood. He has also experienced some urgency and tenesmus but is otherwise systemically well. What is the most appropriate management?

A. Rectal mesalazine √

B. Oral metronidazole

C. Rectal corticoteroids

D. Observe with review in 7 days time

E. Oral loperamide

96. Which of the following genetic conditions is caused by a trinucleotide repeat expansion?

A. Duchenne muscular dystrophy

B. Huntington's disease √

C. Achondroplasia

D. Neurofibromatosis

E. Charcot-Marie-Tooth syndrome ( àFoot drop )

97. A 42-year-old dentist presents to his GP complaining of persistent lethargy. Routine bloods show abnormal liver function tests so a hepatitis screen is sent. The results are shown below: Anti-HAV IgG= negative; HBsAg= negative; Anti-HBs= positive; Anti-HBc= negative

Anti-HCV= positive What do these results most likely demonstrate?

A. Hepatitis B infection

B. Hepatitis C infection

C. Previous vaccination to hepatitis B and C

D. Hepatitis C infection with previous hepatitis B vaccination √

E. Hepatitis B and C infection

98. A 19-year-old who is normally fit and well presents with a sore throat. A decision is made not to prescribe antibiotics. How long should he be advised that his illness will last on average?

A. 2 days

B. 4 days

C. 1 week √

D. 10 days

E. 2 weeks

99. Which one of the following patients should not automatically be prescribed a statin in the absence of any contraindication?

A. A 51-year-old man who had a myocardial infarction 4 years ago and is now asymptomatic

B. A 57-year-old female smoker with a 10-year cardiovascular risk of 23%

C. A 53-year-old man with intermittent claudication

D. A 62-year-old man who had a transient ischaemic attack 10 months ago

E. A 37-year-old man with well controlled diabetes mellitus type 2 √

100. A 24-year-old female with a history of asthma presents to the surgery complaining of being 'tight-chested'. On examination breath sounds are quiet bilaterally and oxygen saturations are 91%. The pulse is 90 bpm and her respiratory rate is 18 / min. She doesn't want to do a peak flow as she says it makes her feel more short of breath. What is the most appropriate management?

A. Nebulised salbutamol + prednisolone + allow home if settles with follow-up review

B. Oxygen + nebulised salbutamol + advise to double inhaled steroids + allow home if settles with follow-up review

C. Nebulised salbutamol + advise to double inhaled steroids + allow home if settles with follow-up review

D. Oxygen + nebulised salbutamol + allow home if settles with follow-up review

E. Oxygen + nebulised salbutamol + prednisolone arrange immediate admission to A&E via ambulance √

101. Which one of the following interventions is most likely to increase survival in patients with COPD?

A. Home nebulisers

B. Tiotropium inhaler

C. Long-term steroid therapy

D. Smoking cessation √

E. Long-term oxygen therapy

102. A 43-year-old man has a work-up for hypertension. He has found to have blood + on a urine dipstick of a freshly voided sample. Which one of the following may account for this finding?

A. Smoking

B. Exercise √

C. Obesity

D. Eating red meat the previous day

E. Use of ramipril

103. A 34-year-old man presents to an emergency surgery with abdominal pain. This started earlier on in the day and is getting progressively worse. The pain is located on his left flank and radiates down into his groin. He has had no similar pain previously and is normally fit and well. Examination reveals a man who is flushed and sweaty but is otherwise unremarkable. What is the most suitable initial management?

A. Oral ciprofloxacin

B. IM diclofenac 75 mg √

C. Oral co-amoxiclav and metronidazole

D. IM morphine 5 mg

E. IM diclofenac 75 mg + start bendroflumethiazide to prevent further episodes

Theme: Angina pectoris: drug management



A. Verapamil

B. Amlodipine

C. Nifedipine

D. Atenolol

E. Nicorandil

F. Isosorbide mononitrate

G. Simvastatin

H. Aspirin

I. Ramipril



For each one of the following please select the correct answer from the options listed above:ia

104. Patients may develop tolerance to this medication necessitating a change in dosing regime Isosorbide mononitrate

105. Is the most appropriate first-line anti-anginal if there are no contraindications Atenolol

106. Is contraindicated if a patient is already prescribed bisoprolol Verapamil

107. When assessing a patient with suspected chronic obstructive disease, which one of the following is least relevant?

A. Smoking history

B. Chest x-ray

C. Full blood count

D. Peak expiratory flow √

E. Spirometry

108. A 52-year-old man with a history of hypertension is found to have a 10-year cardiovascular disease risk of 28%. A decision is made to start simvastatin 40mg on. Liver function tests are performed prior to initialising treatment: Bilirubin=10 µmol/l (3 - 17 µmol/l)

ALP = 96 u/l (30 - 150 u/l) ALT = 30 u/l (10 - 45 u/l)

Gamma-GT = 28 u/l (10 - 40 u/l)

Three months later the LFTs are repeated:

Bilirubin= 12 µmol/l (3 - 17 µmol/l)

ALP = 107 u/l (30 - 150 u/l) ALT = 104 u/l (10 - 45 u/l)

Gamma-GT = 76 u/l (10 - 40 u/l)

What is the most appropriate course of action?

A. Continue treatment and repeat LFTs in 1 month √

B. Check creatine kinase

C. Reduce dose to simvastatin 10mg on and repeat LFTs in 1 month

D. Stop treatment and consider alternative lipid lowering drug

E. Stop treatment and refer to gastroenterology

109. A 42-year-old woman is investigated for lethargy and diarrhoea. Investigations reveal positive anti-endomysial antibodies. Each of the following food stuffs should be avoided, except:

A. Beer

B. Rye

C. Maize √

D. Bread

E. Pasta

110. A 29-year-old woman with a past history of hypothyroidism presents to the surgery complaining of weakness, particularly of her arms, for the past four months. She has also developed double vision towards the end of the day, despite having a recent normal examination at the opticians. What is the most likely diagnosis?

A. Chronic fatigue syndrome

B. Polymyositis

C. Polymyalgia rheumatica

D. Multiple sclerosis

E. Myasthenia gravis √

111. A 25-year-old man presents to his GP complaining of dysuria and pain in his left knee. Three weeks previously he had suffered a severe bout of diarrhoea. What is the most likely diagnosis?

A. Reactive arthritis √

B. Disseminated gonococcal infection

C. Behcet's syndrome

D. Ulcerative colitis

E. Rheumatoid arthritis

112. A 64-year-old woman who is reviewed due to multiple non-healing leg ulcers. She reports feeling generally unwell for many months. Examination findings include a blood pressure of 138/72 mmHg, pulse 90 bpm, pale conjunctivae and poor dentition associated with bleeding gums. What is the most likely underlying diagnosis?



A. Thyrotoxicosisia

B. Vitamin B12 deficiency

C. Vitamin C deficiency √

D. Diabetes mellitus

E. Sarcoidosis





113. A 44-year-old farmer presents with headache, fever and muscle aches. He initially thought he had a bad cold but his symptoms have got progressively worse over the past week. During the review of systems he reports nausea and a decreased urine output. On examination his temperature is 38.2ºC, pulse 102 / min and his chest is clear. Subconjunctival haemorrhages are noted but there is no evidence of jaundice. What is the most likely diagnosis?

A. Mycoplasma pneumonia

B. Lyme disease

C. Legionella pneumon

D. Listeria

E. Leptospirosis √

114. A 66-year-old female starts colchicine to try to settle the symptoms of gout which are not being controlled with diclofenac. What side-effect is it most important to warn her about?

A. Diarrhoea √

B. Heartburn

C. Chest pain

D. Sore throat

E. Visual disturbance

115. A 64-year-old man presents with a eight-month history of generalised weakness. On examination he has fasciculation and weakness in both arms with absent reflexes. Examination of the lower limbs reveal increased tone and exaggerated reflexes. Sensation was normal and there were no cerebellar signs. What is the most likely diagnosis?

A. Lead poisoning

B. Motor neuron disease √

C. Vitamin B12 deficiency

D. Syringomyelia

E. Multiple sclerosis

Theme: Vitamin deficiency



A. Vitamin A

B. Thiamine

C. Niacin

D. Pyridoxine

E. Folic acid

F. Vitamin B12

G. Vitamin C

H. Vitamin D

I. Vitamin E

J. Vitamin K



For each one of the following scenarios select the vitamin which may cause these features if deficient:ia

116. Bleeding gums Vitamin C

117. Diarrhoea, confusion and eczematous skin Niacin

118. Osteomalacia Vitamin D

119. A 72-year-old man with a history of chronic heart failure secondary to ischaemic cardiomyopathy is reviewed. Despite his current treatment with furosemide, ramipril, carvedilol, aspirin and simvastatin he remains short of breath on minimal exertion such as walking 30 metres. On examination his chest is clear and there is minimal peripheral oedema. What is the most appropriate next management step?

A. Stop aspirin

B. Refer for cardiac resynchronisation therapy

C. Switch carvedilol to bisoprolol

D. Add angiotensin-2 receptor blocker

E. Add spironolactone √

120. A 65-year-old woman is investigated for a 6 week history of worsening shortness of breath, lethargy and weight loss. Her past medical history includes chronic obstructive pulmonary disease, hypertension and she is an ex-smoker. Clinical examination is unremarkable. Investigation results are as follows: Chest x-ray Hyperinflated lung fields, normal heart size

Bloods: Sodium 131 mmol/l Potassium3.4 mmol/l Urea7.2 mmol/l Creatinine101µmol/l Hb10.4 g/dl MCV91 fl Plt452 * 109/l WBC3.7 * 109/l

What is the most appropriate management?

A. Screen for depression

B. Short synacthen test

C. Urgent referral to the chest clinic √

D. Stop bendroflumethiazide

E. Urgent gastroscopy

121. A diabetic man is diagnosed as having painful diabetic neuropathy in his feet. He has no other medical history of note. What is the most suitable first-line treatment to relieve his pain?

A. Amitriptyline √

B. Gabapentin

C. Carbamazepine

D. Referral to pain management clinic

E. Pregabalin

122. A 75-year-old female presents with sensory loss to the little finger and wasting of the hypothenar eminence. Where is the likely lesion?

A. Common peroneal nerve

B. Median nerve

C. Radial nerve

D. Anterior interosseous nerve

E. Ulnar nerve √

123. A 49-year-old woman with type 2 diabetes mellitus is being considered for exenatide therapy. Which one of the following is not part of the NICE criteria for starting or continuing this drug?

A. BMI > 35 kg/m2

B. Greater than 1.0 percentage point HbA1c reduction after 6 months

C. Has failed with insulin therapy √

D. Has type 2 diabetes mellitus

E. Weight loss > 3% at 6 months

124. A 64-year-old man with type 2 diabetes mellitus is reviewed. He is currently prescribed metformin and also takes aspirin and simvastatin. There has been no change to his medication for the past 18 months. According to recent NICE guidelines, how often should his HbA1c be checked?

A. 3 monthly

B. 6 monthly √

C. Once a year

D. Every 2 years

E. Only if new problems develop

125. A 62-year-old patient with type 2 diabetes mellitus presents with a 'rash' on his left shin. This has grown in size over the past two days and is now a painful, hot, erythematous area on his anterior left shin spreading around to the back of the leg. He is systemically well and a decision is made to give oral treatment. He has a past history of penicillin allergy. What is the most appropriate antibiotic to give?

A. Clarithromycin

B. Cefaclor

C. Clindamycin

D. Vancomycin

E. Erythromycin √

126. A 62-year-old female with a history of mitral regurgitation attends her dentist, who intends to perform dental polishing. She is known to be penicillin allergic. What prophylaxis against infective endocarditis should be given?

A. Oral doxycyclineia

B. Oral erythromycin

C. No antibiotic prophylaxis needed √

D. Oral ofloxacin

E. Oral clindamycin

127. You are considering prescribing varenicline to a 45-year-old man who is trying to stop smoking. Which one of the following conditions is most likely to contradict the prescription of varenicline?

A. Previous or current central nervous system tumour

B. Past history of deliberate self-harm √

C. Hypertension

D. Epilepsy

E. Obesity

Theme: Causes of headache



A. Temporal arteritis

B. Sinusitis

C. Tension headache

D. Cluster headache

E. Acute glaucoma

F. Migraine

G. Subarachnoid haemorrhage

H. Raised intracranial pressure

I. Medication overuse headache

J. Meningitis



For each one of the following clinical scenarios select the most likely diagnosis:

128. A 62-year-old woman presents with a one day history of pain around her right eye. She feels nauseous and has vomited once. On examination her right eye is red Acute glaucoma

129. A 42-year-old man presents with pain in the posterior and left side of his head. This came on over one minute and is now severe. The pain is worse when he bends his head. His temperature is 37.3ºC Subarachnoid haemorrhage

130. A 22-year-old woman presents with recurrent headaches around the time of her periods. These are typically on the left-side and severe. When she gets a headache it lasts several hours and she usually goes to bed. Migraine

131. A 26-year-old woman who is known to have type 1 diabetes mellitus presents with a three-month history of diarrhoea, fatigue and weight loss. She has tried excluding gluten from her diet for the past 4 weeks and feels much better. She requests to be tested so that a diagnosis of coeliac disease is confirmed. What is the most appropriate next step?

A. Check her HbA1c

B. No need for further investigation as the clinical response is diagnostic

C. Check anti-endomysial antibodies

D. Arrange a jejunal biopsy

E. Ask her to reintroduce gluten for the next 6 weeks before further testing √

132. A 62-year-old man with chronic kidney disease secondary to diabetes mellitus is reviewed. When assessing his estimated glomerular filtration rate (eGFR), which one of the following variables is not required by the Modification of Diet in Renal Disease (MDRD) equation?



A. Age

B. Serum creatinine

C. Ethnicity

D. Gender

E. Serum urea √



133. A 45-year-old man known to have haemochromatosis attends for blood test to assess when he next needs venesection. Of the options given, which one of the following blood tests should be used to assess the adequacy of venesection?

A. Ferritin √

B. Serum iron

C. Haemoglobin

D. Total iron binding capacity

E. Haematocrit

134. A 21-year-old student is brought to the surgery by his friends due to him being confused. They report he has been complaining of headaches for the past few weeks. He is pyrexial and on examination is noted to have abnormally pink mucosa. What is the most likely diagnosis?

A. Carbon monoxide poisoning √

B. Meningitis

C. Paracetamol overdose

D. Subarachnoid haemorrhage

E. Methaemoglobinaemia

135. A 25-year-old woman is diagnosed with a urinary tract infection. She has a past history of epilepsy and is currently taking sodium valproate. Which one of the following antibiotics should be avoided if possible?

A. Co-amoxiclav

B. Nitrofurantoin

C. Cefixime

D. Trimethoprim

E. Ciprofloxacin √

136. Which one of the following is not part of the diagnostic criteria for the metabolic syndrome?

A. High triglycerides

B. Low HDL

C. High LDL √

D. Central obesity

E. Hypertension

137. A 69-year-old man who is known to have Alzheimer's disease is reviewed in clinic. His latest Mini Mental State Examination (MMSE) score is 18 out of 30. According to NICE guidelines, what further action should be taken?

A. Supportive care + memantine

B. Supportive care + trial of citalopram

C. Continue supportive care

D. Supportive care + donepezil + low-dose aspirin

E. Supportive care + donepezil √

138. A 23-year-old man develops watery diarrhoea whilst travelling in Egypt. Which one of the following is the most likely responsible organism?

A. Salmonella

B. Shigella

C. Campylobacter

D. Escherichia coli √

E. Bacillus cereus

139. A 54-year-old man with a history of epilepsy and ischaemic heart disease is seen in clinic with a 3 month history of lethargy. Blood tests are as follows:

Hb7.6 g/dl MCV123 fl Plt134 * 109/l WCC2.6 *109/l

Which one of his medications is most likely to be responsible?

A. Clopidogrel

B. Atorvastatin

C. Carbamazepine

D. Atenolol

E. Phenytoin √

Theme: Thyroid function tests



A. Secondary hypothyroidism

B. Subacute (De Quervain's) thyroiditis

C. Primary atrophic hypothyroidism

D. Sick euthyroid syndrome

E. Poor compliance with thyroxine

F. Graves' disease

G. Steroid therapy

H. Pregnancy

I. Factitious thyroxine-induced hyperthyroidism

J. Hashimoto's thyroiditis



For each of the following scenarios please select the most likely diagnosis:

140. A 64-year-old man recently discharged from hospital following treatment for a pneumonia: TSH = 0.4 mU/l| Free T4 = 8.1 pmol/l Sick euthyroid syndrome

141. A 43-year-old woman presents with a tender goitre: TSH = <0.05 mU/l| Free T4 = 21.7 pmol/l Subacute (De Quervain's) thyroiditis

142. A 34-year-old woman: TSH = <0.05 mU/l| Free T4 = 22 pmol/l Graves' disease

143. A 62-year-old man is reviewed in diabetes clinic. His glycaemic control is poor despite weight loss, adherence to a diabetic diet and his current diabetes medications. He has no other past medical history of note. Which one of the following medications would increase insulin sensitivity?

A. Repaglinide

B. Tolbutamide

C. Pioglitazone √

D. Acarbose

E. Gliclazide

144. A 67-year-old female is prescribed Simvastatin for hyperlipidaemia. Which one of the following may potentially interact with her medication?

A. Orange juice

B. Apple juice

C. Grapefruit juice √

D. Cranberry juice

E. Carrot juice

145. A 62-year-old man with a history of recurrent lower respiratory tract infections is diagnosed as having bilateral bronchiectasis following a high resolution CT scan. Which one of the following is most important in controlling his symptoms?

A. Inhaled corticosteroids

B. Prophylactic antibiotics

C. Surgery

D. Postural drainage √

E. Mucolytic therapy

146. A 57-year-old man presents to surgery with palpitations for the past 24 hours. He has no past history of note. There is no associated chest pain or shortness of breath. Clinical examination is unremarkable other than an irregular tachycardia. An ECG shows atrial fibrillation at a rate of 126 bpm with no other changes. What is the most appropriate management?

A. Beta-blocker + Aspirin √

B. Beta-blocker + Warfarin

C. Digoxin + Warfarin

D. Digoxin + Aspirin

E. Admit patient

147. A 72-year-old man with metastatic small cell lung cancer is admitted to the local hospice for symptom control. His main problem at the moment is intractable hiccups. What is the most appropriate management?

A. Chlorpromazine √

B. Codeine phosphate

C. Diazepam

D. Methadone

E. Phenytoin

148. Which one of the following patients does the Department of Health not specifically recommend receives an annual influenza vaccination?

A. 57-year-old man who had a bowel resection for colorectal cancer last year √

B. 45-year-old man with motor neuron disease who lives in a nursing home

C. 6-year-old boy with diabetes mellitus

D. 24-year-old female with asthma who uses salbutamol and becotide

E. 72-year-old man with no past medical history

149. A 30-year-old woman presents with a three month history of indigestion. There is no history of weight less, anorexia, dysphagia, vomiting or change in bowel habit and abdominal examination is unremarkable. Which one of the following may decrease the accuracy of a 13C-urea breath test?

A. Use of Gaviscon around 10 days ago

B. Use of ranitidine stopping 4 weeks ago

C. Course of amoxicillin stopping 3 weeks ago √

D. Use of lansoprazole stopping 6 weeks ago

E. Current use of the combined oral contraceptive pill





150. A 59-year-old female presents to her GP with a two month history of indigestion. She is otherwise well, has not had a similar episode before and takes no regular medication. Of note there is no recent weight loss or vomiting and abdominal examination is unremarkable. What is the most appropriate initial management?

A. Long-term course of a H2 receptor antagonist

B. Lifestyle advice with follow-up appointment in one month

C. Urgent referral for endoscopy √

D. One month course of a full-dose proton pump inhibitor

E. Urea breath testing and treat for H pylori if positive

151. Primary sclerosing cholangitis is most associated with:

A. Primary biliary cirrhosis

B. Crohn's disease

C. Hepatitis C infection

D. Ulcerative colitis √

E. Coeliac disease

Theme: Malaria: prophylaxis



A. Quinine

B. Mefloquine

C. Atovaquone + proguanil

D. Doxycycline

E. Proguanil

F. Artemether + lumefantrine

G. Primaquine



For each one of the following statements, select the type of malaria prophylaxis from the list of options

152. Is associated with photosensitivity (Also Esophagitis) Doxycycline

153. Should be avoided if the patient has a history of depression Mefloquine

154. Is taken weekly Mefloquine

155. A 62-year-old man is reviewed. His blood pressure is poorly controlled at 152/90 mmHg despite treatment with ramipril 10mg od, bendroflumethiazide 2.5mg od and amlodipine 10mg od. There is no other past medical history of note apart from benign prostatic hyperplasia. In addition to the antihypertensives he also takes aspirin and simvastatin. What is the most appropriate change to his medication?

A. Add frusemide

B. Increase ramipril to 20mg od

C. Add doxazosin √

D. Add candesartan

E. Add atenolol

156. Which one of the following is not a recognised side-effect of sildenafil?

A. Blue discolouration of vision

B. Tinnitus √

C. Flushing

D. Nasal congestion

E. Non-arteritic anterior ischaemic optic neuropathy

157. According to recent NICE guidelines, which one of the following may have a role in the management of irritable bowel syndrome?

A. Reflexology

B. Acupuncture

C. Aloe vera

D. Homeopathy

E. Hypnotherapy √

158. A 74-year-old man presents to his GP for a medication review. Blood pressure is recorded as 184/72. This is confirmed on two further occasions. What is the most appropriate first line therapy?

A. Ramipril

B. Losartan

C. Frusemide

D. Bendroflumethiazide √

E. Atenolol

159. A 74-year-old man presents to his GP for a medication review. Blood pressure is recorded as 184/72. This is confirmed on two further occasions. In addition he also complains of lack of baldder control. What is the most appropriate therapy?

A. Ramipril

B. Frusemide

C. Verapamil √

D. Bendroflumethiazide

E. Atenolol

160. Which one of the following statements regarding the metabolic syndrome is correct?

A. The WHO criteria are used to define impaired glucose tolerance

B. The central pathophysiological change is thought to be reduced insulin production

C. A diagnosis cannot be made without weighing the patient

D. A raised LDL concentration is one of the key criteria in most definitions

E. Decisions on cardiovascular risk factor modification should be made regardless of whether patients meet the criteria for metabolic syndrome √

161. A patient with known heart failure is unable to carry out any physical activity without discomfort. Symptoms of heart failure are present even at rest with increased discomfort with any physical activity. What New York Heart Association class best describes the severity of their disease?

A. NYHA Class 0

B. NYHA Class I

C. NYHA Class II

D. NYHA Class III

E. NYHA Class IV √

162. A 51-year-old female presents to her GP following an episode of transient right sided weakness lasting 10-15 minutes. Examination reveals the patient to be in atrial fibrillation. If the patient remains in chronic atrial fibrillation what is the most suitable form of anticoagulation?

A. Aspirin

B. Warfarin, target INR 2-3 (2ndary Prevention) √

C. No anticoagulationia

D. Warfarin, target INR 3-4

E. Warfarin, target INR 2-3 for six months then aspirin

163. A 51-year-old woman who is known to have poorly controlled type 1 diabetes mellitus is reviewed. Her main presenting complaint is bloating and vomiting after eating. She also notes that her blood glucose readings have become more erratic recently. Which one of the following medications is most likely to be beneficial?

A. Helicobacter pylori eradication therapy

B. Lansoprazole

C. Amitriptyline

D. Metoclopramide √

E. Cyclizine

164. A 52-year-old man with a history of epilepsy is reviewed. Since having his medication change he has experienced a 'numbness' of his hands and feet. On examination he has reduced sensation in a glove-and-stocking distribution associated with a reduced ankle reflex. He is also noted to have lymphadenopathy in the cervical and inguinal region and some bleeding around the gums. Which one of the following medications is he most likely to have been taking?

A. Carbamazepine

B. Phenytoin √

C. Topiramate

D. Sodium valproate

E. Lamotrigine

Theme: Diarrhoea



A. Gastroenteritis

B. Crohn's disease

C. Ulcerative colitis

D. Colorectal cancer

E. Laxative abuse

F. Constipation causing overflow

G. Lactose intolerance

H. Diverticulitis

I. Irritable bowel syndrome

J. Coeliac disease



For each one of the following scenarios please select the most likely diagnosis

165. A 24-year-old smoker presents with intermittent diarrhoea for the past 3 months. She feels bloated, especially around her periods. Bloods tests are normal. Irritable bowel syndrome

166. A 23-year-old student is admitted due to a two-week history of bloody diarrhoea. He is normall fit and well and has not been abroad recently. His CRP is raised at 56 on admission. Ulcerative colitis

167. A 72-year-old woman presents with a two day history of diarrhoea and pain in the left iliac fossa. Her temperature is 37.8ºC. She has a past history of consitpation. Diverticulitis

168. A 72-year-old woman with a past history of treated hypertension presents for review. Yesterday she had a 2 hour episode where she couldn't find the right word when speaking. This has never happened before and there were no associated features. Neurological examination is unremarkable and blood pressure was 150/100 mmHg. Her only current medication is amlodipine. What is the most appropriate management?

A. Aspirin 300mg immediately + specialist review within 2 weeks

B. Specialist review within 2 weeks

C. Aspirin 300mg immediately + specialist review within 24 hours √

D. Aspirin 75mg + outpatient CT brain

E. Specialist review within 24 hours

169. A 54-year-old female presents to her GP with fatigue and xerostomia. Bloods tests reveal the following:

Hb=13.9 g/dl WBC=6.1 *109/l Platelets=246 *109/l Bilirubin= 33 µmol/l

ALP=292 u/l ALT=47 u/l

What is the most likely diagnosis?

A. Systemic lupus erythematous

B. Infectious mononucleosis

C. Primary biliary cirrhosis √

D. Autoimmune hepatitis

E. Sjogren's syndrome

170. A 26-year-old female presents to the surgery due to an exacerbation of her asthma. On examination her peak flow is 300 l/min (usual 450 l/min) and she can complete sentences. Pulse is 90 bpm and the respiratory rate is 18 / min. Examination of the chest reveals a bilateral expiratory wheeze but is otherwise unremarkable. What is the most appropriate management?

A. Oxygen + nebulised salbutamol + allow home if settles with follow-up review

B. Oxygen + nebulised salbutamol + advise to double inhaled steroids + allow home if settles with follow-up review

C. Nebulised salbutamol + prednisolone + allow home if settles with follow-up review √

D. Oxygen + nebulised salbutamol + prednisolone + immediate admission

E. Nebulised salbutamol + advise to double inhaled steroids + allow home if settles with follow-up review

171. Which one of the following types of anti-anginal medication do patients commonly develop tolerance to?

A. Nifedipine

B. Standard release isosorbide mononitrate

C. Nicorandil

D. Verapamil

E. Modified release isosorbide mononitrate √

172. Which of the following drugs is most likely to cause impaired glucose tolerance?

A. Bendroflumethiazide √

B. Perindopril

C. Salicylates

D. Co-amoxiclav

E. Beta-blockers

173. A 24-year-old man presents with a three day history of painful ulcers on the shaft of his penis and dysuria. He has had no similar previous episodes. A clinical diagnosis of primary genital herpes is made. What is the most appropriate management?

A. Topical famciclovir

B. No treatment is indicated

C. Topical podophyllotoxin

D. Topical valaciclovir

E. Oral aciclovir √

174. Which one of the following X-ray changes is not associated with osteoarthritis?

A. Decreased joint space

B. Subchondral sclerosis

C. Subchondral cysts

D. Osteophytes at the joint margin

E. Periarticular erosions √

175. A 36-year-old man is reviewed in clinic. He has recently been started on mesalazine 400mg tds for ulcerative colitis. Which of the following adverse effects is least likely attributable to mesalazine?

A. Interstitial nephritis

B. Headaches

C. Acute pancreatitis

D. Agranulocytosis

E. Infertility √

176. A patient presents to his GP following the development of an urticarial skin rash following the introduction of a new drug. Which one of the following is most likely to be responsible?

A. Omeprazole

B. Sodium valproate

C. Aspirin √

D. Paracetamol

E. Simvastatin







Theme: Drug monitoring



A. U&E, LFT

B. TFT, LFT

C. TFT, U&E

D. LFT

E. FBC, U&E

F. FBC, LFT

G. LFT, creatine kinase

H. Calcium

I. ECG

J. No routine monitoring required



For each of the following drugs select the most appropriate monitoring tests once treatment has commenced:

177. Gliclazide No routine monitoring required

178. Azathioprine FBC, LFT

179. Simvastatin LFT

180. A 23-year-old man presents to his GP. He describes episodes of leg weakness following bouts of laughing whilst out with friends. The following weekend his friends described a brief collapse following a similar episode. What is the most likely diagnosis?

A. Stokes-Adams attack

B. Cataplexy √

C. Hypokalaemic periodic paralysis

D. Absence seizure

E. Myasthenia gravis

181. A 54-year-old man with a history of ischaemic heart disease is currently taking atorvastatin 40mg at night. A repeat lipid profile is ordered:

Total cholesterol=3.9 mmol/l HDL=0.7 mmol/l LDL=2.6 mmol/l Triglycerides=1.2 mmol/l

What would be the most effective way of increasing HDL levels?

A. Add nicotinic acid √

B. Add ezetimibe

C. Switch atorvastatin to pravastatinia

D. Add bezafibrate

E. Add colestyramine

182. Which one of the following statements regarding the stopping of anti-epileptic drugs (AED) is most correct?

A. Can be considered if seizure free for > 5 years, with AEDs being stopped over 2-3 months

B. Can be considered if seizure free for > 2 years, with AEDs being stopped over 2-3 months √

C. Can be considered if seizure free for > 1 year, with AEDs being stopped over 2-3 months

D. Can be considered if seizure free for > 5 years, with AEDs being stopped over 8-12 months

E. Can be considered if seizure free for > 1 year, with AEDs being stopped over 8-12 months

183. A 24-year-old woman presents following a sudden, acute onset of pain at the back of the ankle whilst jogging, during which she heard a cracking sound. Which one of the following medications may have contributed to this injury?



A. Metronidazole

B. Nitrofurantoin

C. Fluconazole

D. Ciprofloxacin √

E. Terbinafine



184. A 54-year-old woman presents to surgery with a five day history of back pain. Her past medical history includes breast cancer and osteoarthritis. The back pain is located in the lower thoracic region and is made worse by coughing and sneezing. There has been no change in bowel habit or urinary symptoms. On examination there is diffuse tenderness in the lower thoracic region. Peri-anal sensation is normal and lower limb reflexes are brisk. Which one of the following is the most appropriate management plan?

A. Organise outpatient MRI

B. Oral paracetamol + exercise programme

C. Oral paracetamol + urgent thoracic/lumbar spine x-ray

D. Oral dexamethasone + urgent thoracic/lumbar spine x-ray

E. Oral dexamethasone + immediate oncological assessment √

185. A 45-year-old female with a history of bipolar disorder presents with an acute confusional state. Which one of the following drugs is most likely to precipitate lithium toxicity?

A. Sodium valproate

B. Atenolol

C. Aminophylline

D. Sodium bicarbonate

E. Bendroflumethiazide √





186. Which one of the following patients should not automatically be prescribed aspirin 75 mg OD in the absence of any contraindication?

A. 62-year-old man who had a myocardial infarction 11 years ago

B. 72-year-old man with intermittent claudication

C. 44-year-old man with hypertension and a 10-year cardiovascular risk of 14% √

D. 54-year-old man with well controlled type 2 diabetes mellitus with no evidence of end-organ damage

E. 64-year-old smoker with a 10-year cardiovascular risk of 24%

187. You are compiling a report on cancer care in the local community. Following lung cancer, what is the second most common cause of death from cancer in the UK?

A. Stomach

B. Prostate

C. Breast

D. Oesophagus

E. Colorectal √

188. A 60-year-old woman who has recently been diagnosed with chronic obstructive pulmonary disease (COPD) presents for review. She is still occasionally breathless despite bronchodilator therapy. Which one of the following criteria is most relevant when deciding who would benefit from inhaled corticosteroids?

A. Failure of long-acting beta2-agonist to improve breathlessness

B. FEV1/FVC < 50% predicted

C. FEV1 < 50% predicted and 2 or more exacerbations per year √

D. FEV1/FVC < 50% predicted and 3 or more exacerbations per year

E. Four or more than exacerbations per year

Theme: Causes of pruritus



A. Liver disease

B. Hypothyroidism

C. Diabetes mellitus

D. Pregnancy

E. Chronic kidney disease

F. Polycythaemia

G. Iron deficiency anaemia

H. Senile pruritus

I. Scabies

J. Lymphoma



For each of the following scenarios select the most likely diagnosis:

189. A 52-year-old woman presents with pruritus and lethargy. She has recently put on weight and is complaining about dry skin Hypothyroidism

190. A 48-year-old woman presents with pruritus. She states she has been gaining weight despite eating less and complains of constant nausea. On examination she is pale Chronic kidney disease

191. A 59-year-old man complains of pruritus and lethargy. On examination he has spoon shaped nails and a smooth tongue Iron deficiency anaemia

192. A 72-year-old presents to the surgery complaining of dizziness. An ECG is taken and shows the following:iaWhat is the diagnosis?


A. Ventricular tachycardia

B. Second degree heart block - Mobitz type 2

C. First degree heart block

D. Second degree heart block - Mobitz type 1

E. Third degree heart block √



193. Which of the following disease is associated with 3rd degree heart block

1. Leptospirosis

2. Legioner’s disease

3. Leishmaniasis

4. Lyme Disease √

5. Brucellosis

194. A phlebotomist gives herself a needlestick injury whilst taking blood from a patient who is known to be hepatitis B positive. The phlebotomist has just started her job and is in the process of being immunised for hepatitis B but has only had one dose to date. What is the most appropriate action to minimise her risk of contracting hepatitis B from the needle?

A. No action needed, complete hepatitis B vaccination course as normal

B. Give oral ribavirin for 4 weeks

C. Give an accelerated course of the hepatitis B vaccine + hepatitis B immune globulin √

D. Give hepatitis B immune globulin + oral ribavirin for 4 weeks

E. Give hepatitis B immune globulin





195. Which one of the following treatments has not been shown to improve mortality in patients with chronic heart failure?



A. Beta-blockers

B. Spironolactone

C. Frusemide √

D. Nitrates and hydralazine

E. Enalapril



196. A 19-year-old presents as she would like to start a combined oral contraceptive pill. During the history she states that in the past she has had migraine with aura in the past. She asks why the combined oral contraceptive pill is contraindicated. What is the most appropriate response?

A. Theoretical risk of ischaemic stroke

B. Significantly increased risk of ischaemic stroke √

C. Increased frequency of migraines

D. Migraine is an independent risk factor for venous thromboembolism

E. Increased severity of migraines

197. A 72-year-old man with metastatic colon cancer is reviewed. He currently takes co-codamol 30/500 2 tablets qds for pain relief. Unfortunately this is not controlling his pain. What is the most appropriate change to his medication?

A. Switch to MST 15mg bd + paracetamol 1g qds
B. Switch to MST 35mg bd + paracetamol 1g qds

C. Add tramadol 50-100mg 1-2 qds

D. Switch to MST 25mg bd

E. Switch to MST 15mg bd

198. A 41-year-old female presents with 3 day history of a dry cough and shortness of breath. This was preceded by flu-like symptoms. On examination there is a symmetrical, erythematous rash with 'target' lesions over the whole body. What is the likely organism causing the symptoms?



A. Pseudomonas

B. Staphylococcus aureus

C. Mycoplasma pneumoniae √

D. Chlamydia pneumoniae

E. Legionella pneumophilia



199. A 41-year-old female presents with 3 day history of a dry cough and shortness of breath. This was preceded by flu-like symptoms. She complains of pain in her ear. What is the likely cause?



A. Tonsillitis

B. Otitis Externa

C. Bullous Myringitis √

D. Septic embolus

E. Ramsy Hunt Syndrome



200. A 43-year-old man requests a 'medical' as he is concerned about his risk of heart disease. His father died at the age of 45-years following a myocardial infarction. His lipid profile is as follows:

HDL = 1.4 mmol/l LDL = 5.7 mmol/l Triglycerides = 2.3 mmol/l

Total cholesterol = 8.2 mmol/l

Clinical examination reveals tendon xanthomata around his ankles. What is the most likely diagnosis?

A. Mixed hyperlipidaemia

B. Nephrotic syndrome

C. Alcohol excess

D. Non-familial hypercholesterolaemia

E. Familial hypercholesterolaemia √

201. According to recent NICE guidelines on the management of respiratory tract infections, which one of the following patients should not be considered for immediate antibiotic prescribing:

A. A 12-year-old who has acute sinusitis and a temperature of 37.6ºC √

B. A 23-year-old woman who has acute tonsillitis. Her temp is 37.8ºC, tonsillar exudate is seen and there is tender lymph nodes

C. A 5-year-old who has acute otitis media associated with otorrhoea

D. A 7-month old who has bilateral otitis media and is apyrexial

E. An 18-month old who has bilateral otitis media and a temperature of 38.1ºC

202. A 73-year-old woman presents with lethargy for the past two weeks. Clinical examination is unremarkable. Her past medical history includes polymyalgia rheumatica and ischaemic heart disease. Screening blood tests are ordered and the full blood count is reported as follows:

Hb 12.9 g/dl Plt 158 * 109/l WBC 19.0 * 109/l Neuts 4.2 * 109/l

Lymphs 14.1 * 109/l

What is the most likely diagnosis?

A. Lymphoma

B. Nicorandil-related lymphocytosis

C. Transient viral illness

D. Chronic lymphocytic leukaemia √

E. Secondary to steroid use



Theme: Haemoptysis



A. Lung cancer

B. Pulmonary embolism

C. Goodpasture’s syndrome

D. Tuberculosis

E. Aortic regurgitation

F. Mitral stenosis

G. Coccidiomycosis

H. Aspergilloma

I. Wegener's granulomatosis

J. Bronchiectasis



For each of the following scenarios select the most likely diagnosis:

203. A 52-year-old man who was born in India presents with episodic haemoptysis. His only history is tuberculosis as an adolescent. Chest x-ray shows a rounded opacity in the right upper zone surrounded by a rim of air Aspergilloma

204. A 71-year-old woman presents with dyspnoea and haemoptysis for the past two weeks. Clinical examination reveals a third heart sound, a diastolic murmur and new-onset atrial fibrillation Mitral stenosis

205. A 62-year-old woman who is being investigated for renal impairment presents with haemoptysis. On examination she has a flat nose Wegener's granulomatosis

206. Which of the following results establishes a diagnosis of diabetes mellitus?

A. Asymptomatic patient with fasting glucose 7.9 mmol/L on one occasion

B. Symptomatic patient with fasting glucose 6.8 mmol/L on two occasions

C. Glycosuria +++

D. Asymptomatic patient with random glucose 22.0 mmol/L on one occasion

E. Symptomatic patient with random glucose 12.0 mmol/L on one occasion √

207. A 25-year-old man presents with lethargy and increased skin pigmentation. Blood test reveal deranged liver function tests and impaired glucose tolerance. Given the likely diagnosis of haemochromatosis, what is the most appropriate initial investigation strategy?

A. Transferrin saturation + ferritin √

B. Haematocrit + ferritin

C. Liver biopsy with Perl's stain

D. Serum iron + ferritin

E. Serum iron + haematocrit

208. A 43-year-old woman presents for follow-up in clinic. She was diagnosed with Hashimoto's thyroiditis four months ago and is currently being treated with levothyroxine 75 mcg OD. What is the single most important blood test to assess her response to treatment?

A. ESR

B. TSH √

C. Free T4

D. Total T4

E. Free T3

209. A 45-year-old man with a history of depression and gastro-oesophageal reflux disease presents to his GP due to a milky discharge from his nipples. The following blood results are obtained: Prolactin 700 mu/l Which one of his medications is most likely to be responsible?



A. Omeprazole

B. Fluoxetine

C. Metoclopramide √

D. Cimetidine

E. Amitriptyline



210. In line with NICE guidelines on the secondary prevention of osteoporotic fractures in postmenopausal women, which one of the following patients should not automatically be started on treatment?

A. A 81-year-old woman who has had a fractured neck of femur

B. A 64-year-old women with a wedge fractures of her thoracic spine. DEXA scan shows a T-score of -3.1 SD

C. A 55-year-old women who had a Colles fracture of the wrist. DEXA scan shows a T-score of -3.3 SD

D. A 64-year-old women with a BMI of 18 kg/m2. She has a wedge fractures of her thoracic spine. DEXA scan shows a T-score of -2.7 SD

E. A 71-year-old women who had a Colles fracture of the wrist. DEXA scan shows a T-score of -2.1 SD √

211. You receive the blood results of a 76-year-old man who takes warfarin following a pulmonary embolism two months ago. He recently completed a course of antibiotics. INR 8.4

On reviewing the patient he is well with no bleeding or bruising. What is the most appropriate action?

A. Stop warfarin + restart when INR < 5.0 + give low-molecular weight heparin until warfarin restarted

B. Stop warfarin + restart when INR < 3.0

C. Oral vitamin K 5mg + stop warfarin + restart when INR < 5.0 √

D. Admit

E. Stop warfarin + restart when INR < 5.0





212. A 19-year-old man presents as he is concerned he may be asthmatic. Which one of the following points in the history would make this diagnosis less likely?

A. Smoking since age of 16 years

B. Peripheral tingling during episodes √

C. Peripheral blood eosinophilia

D. Chest tightness whilst exercising

E. History of eczema

213. Which one of the following antibiotics is most likely to cause pseudomembranous colitis?

A. Ceftriaxone √

B. Cefalexin

C. Gentamicin

D. Co-amoxiclav

E. Piperacillin-tazobactam

214. A 45-year-old women with type 1 diabetes mellitus is reviewed in the diabetes clinic. Three months ago her blood tests were as followed: K+ = 4.5 mmol/l; Creatinine=116µmol/l eGFR=47 ml/min At the time she was started on lisinopril to treat both the hypertension and act as a renoprotective agent. Lisinopril had been titrated up to treatment dose. Her current bloods are as follows:

K+ = 4.9 mmol/l Creatinine = 123 µmol/l eGFR = 44 ml/min

Of the following options, what is the most appropriate course of action?

A. Stop lisinopril and arrange investigations to exclude renal artery stenosis

B. Switch to a angiotensin 2 receptor blocker

C. Switch to a different ACE inhibitor

D. No action √

E. Reduce dose of lisinoprilia

Theme: Abdominal swelling



A. Irritable bowel syndrome

B. Endometrial cancer

C. Ovarian cancer

D. Pregnancy

E. Intestinal obstruction

F. Urinary retention

G. Ascites

H. Gastric cancer

I. Colorectal cancer

J. Bladder cancer



For each one of the following scenarios select the most likely diagnosis:

215. 62-year-old woman with a 3 month history of urinary symptoms, early satiety and a raised CA125 Ovarian cancer

216. 26-year-old female with a history of constipation, episodic abdominal pain and bloating.

Irritable bowel syndrome

217. 72-year-old woman with a history of congestive cardiac failure. She reports having a poor appetite and feeling bloated. She is admitted frequently to hospital with left ventricular failure due to poor compliance with medication Ascites

218. A 23-year-old student is investigated following an anaphylactic reaction suspected to be secondary to a wasp sting. Which one of the following is the most appropriate first-line test to investigate the cause of the reaction?

A. Hair analysis

B. Radioallergosorbent test (RAST) √

C. Desensitization therapy

D. Skin patch test

E. Skin prick test

219. Which of the following relating to St John's Wort is false?

A. Adverse effects in trials is similar to placebo

B. May cause serotonin syndrome

C. Mechanism of action is similar to selective serotonin reuptake inhibitors

D. Causes inhibition of the P450 system √

E. Has been shown to be effective in treating mild-moderate depression

220. Which one of the following is least recognised as an adverse effect of taking bendroflumethiazide?

A. Hypokalaemia

B. Pseudogout √

C. Hypercalcaemia

D. Impotence

E. Impaired glucose tolerance





221. A 71-year-old man who had a bioprosthetic aortic valve replacement three years ago is reviewed. What antithrombotic therapy is he likely to be taking?

A. Nothing

B. Aspirin √

C. Warfarin: INR 2.0-3.0

D. Aspirin + clopidogrel

E. Warfarin: INR 3.0-4.0

222. A 61-year-old woman who is normally fit and well is admitted with chest pain. An ECG shows anterolateral T wave inversion. The troponin T value at 12 hours is 0.54. On discharge her medications include aspirin, atorvastatin, bisoprolol and ramipril. Which one of the following statements best describes the role of clopidogrel in this situation?

A. Is only given if aspirin is contraindicated

B. Should be prescribed for life for patients < 65 years old

C. Should be prescribed for the next 12 months for all patients √

D. Should be prescribed for the next 12 months for patients < 65 years old

E. Should be prescribed for life for all patients

223. A 46-year-old man with a history of hyperlipidaemia is reviewed in clinic. He is currently taking simvastatin 10mg on but his cholesterol level remains high. Previous attempts to increase the dose of Simvastatin have resulted in myalgia. Given the history of myalgia, which lipid-regulating drug should be avoided?

A. Nicotinic acid

B. Bezafibrate √

C. Colestyramine

D. Omega-3 fatty acid

E. Ezetimibe

203. A 69-year-old man who had a stroke 6 months ago is reviewed. After his diagnosis he was started on Simvastatin 40mg on for secondary prevention of further cardiovascular disease. A fasting lipid profile taken one week ago is reported as follows:

Total cholesterol = 4.4 mmol/l HDL cholesterol = 1.0 mmol/l LDL cholesterol= 3.1 mmol/l

Triglyceride = 1.5 mmol/l

According to recent NICE guidelines, what is the most appropriate action?

A. Switch to simvastatin 80mg on √

B. No change in medication, repeat lipid profile in 6 months

C. Add nicotinic acid

D. Switch to atorvastatin 80mg onia

E. Add ezetimibe

224. A 57-year-old man with a history of ischaemic heart disease is keen to try sildenafil for erectile dysfunction. Which one of the following medications may contraindicate its use?

A. Nebivolol

B. Losartan

C. Nicorandil √

D. Nifedipine

E. Ramipril

204. A 65-year-old Asian female presents to her GP with generalised bone pain and muscle weakness. Investigations show:- Calcium = 2.07 mmol/l Phosphate = 0.66 mmol/l ALP = 256 U/l

What is the most likely diagnosis?

A. Bone tuberculosis

B. Hypoparathyroidism

C. Myeloma

D. Osteomalacia √

E. Paget's disease

Theme: Diabetes mellitus: management of type 2



A. < 130/80 mmHg

B. < 125/75 mmHg

C. < 120/70 mmHg

D. 6.5%

E. 6.0%

F. 6.2%

G. Aspirin

H. Statin

I. Ramipril

J. No additional treatment



For each one of the following select the most appropriate answer:

225. A 43-year-old man with type 2 diabetes mellitus is reviewed. His HbA1c is 6.6% on metformin therapy. His blood pressure is 128/78 mmHg, he is a non-smoker and is not overweight. There is no family history of note. What addition, if any, should be made to his medication? No additional treatment

226. The general HbA1c target for patients with type 2 diabetes mellitus 6.5%

227. The target blood pressure for a 60-year-old man with type 2 diabetes mellitus and diabetic nephropathy < 130/80 mmHg

228. A health trust in the United Kingdom which serves a population of 100,000 is planning services for patients with rheumatoid arthritis. How many of the population would be expected to have the disease?

A. 100

B. 300

C. 1,000 √

D. 2,000

E. 10,000

229. Which of the following is not a recognised feature of temporal arteritis?

A. Rapid response to high dose prednisolone

B. Pyrexia

C. Skip lesions in temporal artery

D. Visual disturbance

E. Elevated creatine kinase √

230. A 31-year-old woman is reviewed in surgery with an 8 month history of abdominal discomfort associated with bloating. Which one of the following tests is it least useful to perform before making a positive diagnosis of irritable bowel syndrome?

A. Erythrocyte sedimentation rate

B. Thyroid function tests √

C. Full blood count

D. C-reactive protein

E. Tissue transglutaminase antibodies

231. A 54-year-old woman with a long history of rheumatoid arthritis is reviewed in clinic complaining of shortness of breath. Oxygen saturations are 92% on room air with spirometry showing a restrictive pattern associated with a reduced transfer factor. Which one of the following drugs is most likely to be responsible?

A. Depomederone

B. Hydroxychloroquine

C. Methotrexate √

D. Ciclosporin

E. Celecoxib

205. A 34-year-old female presents due to the development of a purpuric rash on the back of her legs. Her only regular medication is Microgynon 30. She also reports frequent nose bleeds and menorrhagia. A full blood count is requested:

Hb = 11.7 g/dl Platelets = 62 * 109/l WCC = 5.3 * 109/l

What is the most likely diagnosis?

A. Drug-induced thrombocytopen

B. Henoch-Schonlein purpura

C. Thrombotic thrombocytopenic purpura

D. Idiopathic thrombocytopenic purpura √

E. Antiphospholipid syndrome`e

206. A 40-year-old man presents to surgery as he has noted an abnormality around his right eye:

What is the most likely diagnosis?


A. Hypertriglyceridaemia

B. Hypercholesterolaemia √

C. Hypothyroidism

D. Wilson's disease

E. Diabetes mellitus





207. Which one of the following statements regarding the use of triptans agonists in the treatment of migraine is incorrect?

A. May be given subcutaneously

B. Are second line therapy in the management of acute migraine

C. Should be taken as soon as possible after the onset of an aura √

D. Should be avoided in patients with ischaemic heart disease

E. Adverse effects include tingling and chest tightness





208. Which one of the following statements regarding hepatitis C is correct?

A. Cannot be transmitted vertically from mother to child

B. Interferon-alpha and ribavirin are the treatments of choice √

C. It is more infectious than hepatits B following a needle stick injury

D. Breast feeding is contraindicated in mothers with hepatitis C

E. HCV RNA is the initial investigation of choice for at-risk groups

209. You are called to assess a man who has collapsed in the waiting room. The receptionist has already called 999. On arrival the man is laid on his back. You open the airway with a head-tilt chin lift - after assessing for 10 seconds there are no signs of breathing. What is the most appropriate next step?

A. Start chest compressions at a ratio of 15:2

B. Place in the recovery position

C. Check for a carotid pulse for 10 seconds

D. Give 2 rescue breaths

E. Start chest compressions at a ratio of 30:2 √

Theme: Antibiotic guidelines



Erythromycin
Trimethoprim or nitrofurantoin or amoxicillin or cephalosporin
Quinolone or trimethoprim
Trimethoprim or vancomycin
Amoxicillin or tetracycline or erythromycin
Phenoxymethylpenicillin + flucloxacillin
Flucloxacillin
Amoxicillin or doxycycline or erythromycin
Doxycycline
Broad-spectrum cephalosporin or quinolone


For each one of the following please select the most complete answer from the options above:

210. Atypical pneumonia Erythromycin

211. Lower urinary tract infection Trimethoprim or nitrofurantoin or amoxicillin or cephalosporin

212. Should be added in the treatment of pneumonia if secondary to influenza Flucloxacillin

213. A 72-year-old man is reviewed in the diabetes clinic. He has a history of heart failure and type 2 diabetes mellitus. His current medications include furosemide 40mg od, ramipril 10mg od and bisoprolol 5mg od. Clinical examination is unremarkable with no evidence of peripheral oedema, a clear chest and blood pressure of 130/76 mmHg. Recent renal and liver function tests are normal. Which one of the following medications is contraindicated?

A. Sitagliptin

B. Pioglitazone √

C. Gliclazide

D. Exenatide

E. Metformin

214. A 46-year-old female presents with a burning sensation over the antero-lateral aspect of her right thigh. A diagnosis of meralgia paraesthetica is suspected. Which nerve is most likely to be affected?

A. Common peroneal nerve

B. Anterior cutaneous nerve of thigh

C. Posterior cutaneous nerve of thigh

D. Lateral cutaneous nerve of thigh √

E. Sciatic nerveia

215. Which one of the following statements is true regarding the difference between Qvar and CFC-containing beclometasone inhalers?

A. Qvar is the only CFC-free beclometasone inhaler currently available

B. There are no studies comparing Qvar to CFC-containing beclometasone inhalers

C. The equivalent dose of Qvar is lower than that of CFC-containing beclometasone inhalers √

D. Qvar must be taken 4 times per day

E. New prescriptions should be labelled 'CFC-free beclometasone inhaler

216. A 59-year-old man with a history of type 2 diabetes mellitus and benign prostatic hypertrophy develops urinary retention associated with acute renal failure. Which one of the following drugs should be discontinued?

A. Gliclazide

B. Paroxetine

C. Atenolol

D. Metformin √

E. Finasteride





217. A 35-year-old woman is noticed to be jaundiced. As part of a liver screen the following results are obtained: Anti-HBs Positive Anti-HBc Positive HBs antigen Negative

Anti-HBs = Hepatitis B Surface Antibody; Anti-HBc = Hepatitis B Core Antibody; HBs antigen = Hepatitis B Surface Antigen What is the patient's hepatitis B status?

A. Previous immunisation to hepatitis B

B. Chronic hepatitis B - highly infectious

C. Previous hepatitis B infection, not a carrier √

D. Chronic hepatitis B - not infectious

E. Acute hepatitis B infection

218. A syringe driver is prescribed for a patient who is dying from metastatic breast cancer. Which one of the following drugs is incompatible with dexamethasone, metoclopramide and midazolam?

A. Diamorphine

B. Hyoscine hydrobromide

C. Cyclizine √

D. Levomepromazine

E. Haloperidol

219. A 34-year-old man presents with a widespread maculopapular rash and mouth ulcers. Two months ago he reports developing a painless penile ulcer. Which one of the following organisms is most likely to be responsible?

A. Lymphogranuloma venereum

B. Herpes simplex virus type 2

C. Mycoplasma genitalium

D. Treponema pertenue

E. Treponema pallidum √

220. A 33-year-old female is referred by her GP with thyrotoxicosis. Following a discussion of management options she elects to have radioiodine therapy. Which one of the following is the most likely adverse effect?

A. Hypothyroidism √

B. Thyroid malignancy

C. Agranulocytosis

D. Oesophagitis

E. Precipitation of thyroid eye disease

221. According to current BNF guidelines which one of the following antibiotics would be least appropriate in the initial management of a lower urinary tract infection in a 44-year-old woman?

A. Oral trimethoprim

B. Oral nitrofurantoin

C. Oral amoxicillin

D. Oral cephalosporin

E. Oral ciprofloxacin √

Theme: Cytotoxic agents: side-effects



A. Doxorubicin

B. Cisplatin

C. Methotrexate

D. Dactinomycin

E. Vincristine

F. Fludarabine

G. Bleomycin

H. Cyclophosphamide

I. Paclitaxel

J. Pentostatin



For each of the following side-effects please select the cytotoxic agent which is most likely to be responsible:

222. Hypomagnesaemia Cisplatin

223. Haemorrhagic cystitis Cyclophosphamide

224. Cardiomyopathy Doxorubicin

225. A 45-year-old man is started on ciclosporin following a renal transplant. Which one of the following adverse effects is most likely to occur?



A. Depression

B. Increased risk of ischaemic heart disease

C. Pulmonary fibrosis

D. Impaired glucose tolerance

E. Nephrotoxicity √



226. A 23-year-old man with difficult to control epilepsy is reviewed in surgery, four months after a change in his antiepileptic medication. He has remained seizure free but has gained 5 kg in weight since last reviewed. Which one of the following drugs is he most likely to have been prescribed?

A. Ethosuximide

B. Sodium valproate √

C. Levetiracetam

D. Carbamazepine

E. Lamotrigine

227. A 45-year-old man presents with dizziness and right-sided hearing loss to his GP. Which one of the following tests would most likely indicate an acoustic neuroma?

A. Jerky nystagmusia

B. Left homonymous hemianopia

C. Tongue deviated to the left

D. Fasciculation of the tongue

E. Absent corneal reflex √

228. A 67-year-old female complains of visual problems during follow-up in the medical outpatient clinic. Which one of the following drugs is most likely to cause visual impairment?

A. Amiodarone √

B. Sodium valproate

C. Methotrexate

D. Frusemide

E. Amoxicillin

229. A 24-year-old nulliparous female with a history of recurrent deep vein thrombosis presents with shortness of breath. The full blood count and clotting screen reveals the following results:

Hb = 12.4 g/dl Plt = 137 WBC = 7.5 * 109/l PT = 14 secs APTT= 46 secs

What is the most likely underlying diagnosis?

A. Third generation oral contraceptive pill use

B. Protein C deficiency

C. Antithrombin III deficiency

D. Antiphospholipid syndrome √

E. Activated protein C resistance

230. A 45-year-old man is prescribed bupropion to help him quit smoking. What is the mechanism of action of bupropion?

A. Nicotinic receptor partial agonist

B. Selective serotonin reuptake inhibitor

C. Norepinephrine and dopamine reuptake inhibitor, and nicotinic antagonist √

D. Dopamine agonist

E. Dopamine antagonist

231. Of the following scenarios, which one would indicate it was inappropriate for the patient to take an airline flight?

A. A 54-year-old woman who had a laparoscopic cholecystectomy 5 days ago

B. A 17-year-old flying back to the UK who broke his leg whilst skiing in Canada. Had a plaster cast applied 24 hours ago √

C. A 59-year-old man who had a colonoscopy 2 days ago

D. A 62-year-old man who had an uncomplicated myocardial infarction 3 weeks ago

E. A woman who is 27-weeks pregnant with twins

232. A patient with known heart failure has slight limitation of physical activity. She is comfortable at rest but housework results in fatigue, palpitations or dyspnoea. What New York Heart Association class best describes the severity of their disease?

A. NYHA Class 0

B. NYHA Class I

C. NYHA Class II

D. NYHA Class III √

E. NYHA Class IV

233. A 61-year-old man with peripheral arterial disease is prescribed simvastatin. What is the most appropriate blood test monitoring?

A. LFTs + creatinine kinase at baseline, 1-3 months and at intervals of 6 months for 1 yearia

B. LFTs at baseline and every 3 months for first year

C. Routine blood tests not recommended

D. LFTs at baseline and annually

E. LFTs at baseline, 1-3 months and at intervals of 6 months for 1 year √

Theme: Side-effects of diabetes mellitus drugs



A. Metformin

B. Acarbose

C. Glimepiride

D. Nateglinide

E. Rosiglitazone

F. Diazoxide

G. Repaglinide



Select the drug most likely to cause each one of the following side-effects:

234. Syndrome of inappropriate ADH secretion Glimepiride

235. Lactic acidosis Metformin

236. Fluid retention Rosiglitazone

237. A 19-year-old female is brought to the Emergency Department by her friends following a night out. Her friends state she has taken an unknown drug whilst out clubbing. Which one of the following features would most point towards the use of ecstasy?

A. Temperature of 39.5ºC √

B. Respiratory depression

C. Hypernatraemia

D. Miosis

E. Urinary incontinence

238. A 55-year-old man is prescribed sibutramine to try and help him lose weight. He currently has a body mass index of 34 kg/m2. After initiating therapy what is the most important type of monitoring to perform?

A. Visual acuity

B. Liver function tests

C. Anxiety and depression levels

D. QT interval on ECG

E. Blood pressure √

239. A 22-year-old male blood donor is noted to have the following blood results:

Bilirubin =41 µmol/L ALP=84 U/L ALT=23 U/L Albumin=41 g/L

Dipstick urinalysis=normal He has recently complained of coryzal symptoms and a non- productive cough. What is the most likely diagnosis?

A. Gilbert's syndrome √

B. Dubin-Johnson syndrome

C. Rotor syndromeia

D. Hepatitis C infection

E. Infectious mononucleosis

240. A 65-year-old man is referred to the neurology outpatient clinic due to a resting tremor of his right hand. A diagnosis of Parkinson's disease is made. He is otherwise well and is not currently disabled by his symptoms. What is the most appropriate treatment?

A. Selegiline

B. No treatment √

C. New generation dopamine receptor agonist e.g. ropinirole

D. Conventional dopamine receptor agonist e.g. bromocriptine

E. Antimuscarinics

241. A 54-year-old woman with a 30-pack-year history of smoking presents due to increasing breathlessness. A diagnosis of chronic obstructive pulmonary disease (COPD) is suspected. Which of the following diagnostic criteria should be used when assessing a patient with suspected COPD?

A. FEV1 > 70% of predicted value + FEV1/FVC < 60%

B. FEV1 < 80% of predicted value + FEV1/FVC < 70% √

C. FEV1 < 70% of predicted value + FEV1/FVC < 70%

D. FEV1 < 80% of predicted value + FEV1/FVC < 60%

E. FEV1 < 70% of predicted value + FEV1/FVC > 70%

242. A 62-year-old man presents four weeks after initiating metformin for type 2 diabetes mellitus. His body mass index is 27.5 kg/m2. Despite slowly titrating the dose up to 500mg tds he has experienced significant diarrhoea. He has tried reducing the dose back down to 500mg bd but his symptoms persisted. What is the most appropriate action?

A. Switch to pioglitazone 15mg od

B. Switch to gliclazide 40mg od

C. Start modified release metformin 500mg od with evening meal √

D. Add loperamide as required

E. Arrange colonoscopy

243. A 52-year-old woman is diagnosed with non-alcoholic steatohepatitis following a liver biopsy. What is the single most important step to help prevent the progression of her disease?

A. Stop smoking √

B. Start statin therapy

C. Eat more omega-3 fatty acids

D. Start sulfonylurea therapy

E. Weight loss





244. A 24-year-old man who has a sister with adult polycystic kidney disease asks his GP if he could be screened for the disease. What is the most appropriate screening test?

A. PKD1 gene testing

B. CT abdomen

C. Urine microscopy

D. Ultrasound abdomen √

E. Anti-polycystin 1 antibodies levels

245. A 45-year-old man is diagnosed as having pulmonary tuberculosis. He currently lives in the UK and his sputum is positive for acid-fast bacilli. His past medical history includes hypertension for which he takes bendroflumethiazide and amlodipine. Which of the following combination of medications should he be taking initially?

A. Rifampicin, isoniazid, pyrazinamide and ethambutol √

B. Isoniazid, pyrazinamide and ethambutol

C. Rifampicin, isoniazid and pyrazinamide

D. Rifampicin, isoniazid, pyrazinamide, ethambutol and streptomycinia

E. Rifampicin and isoniazid

Theme: Causes of tremor



A. Anxiety

B. Opiate withdrawal

C. Alcohol withdrawal

D. Hepatic encephalopathy

E. Parkinsonism

F. Essential tremor

G. Thyrotoxicosis

H. Carbon dioxide retention

I. Cerebellar disease

J. Wilson's disease



For each one of the following scenarios select the most likely diagnosis:

246. A 62-year-old man is seen on a home visit. He has a history of treated hypothyroidism and COPD. His wife reports he is more confused. When he stretches both arms out a 'flap' is noted. Carbon dioxide retention

247. A 55-year-old woman complains she is losing weight. On examination her pulse is 102 bpm and she has a fine tremor when her hands are outstretched Thyrotoxicosis

248. A 28-year-old man presents with a tremor in both hands. This is most noticeable when he uses his hands. He has a strong family history of similar problems Essential tremor

249. Which of the following drugs is least likely to cause cholestasis?

A. Gliclazide

B. Amiodarone √

C. Chlorpromazine

D. Oral contraceptive pill

E. Co-amoxiclav

250. Which tumour is the most common cause of bone metastases?

A. Breast

B. Lung

C. Colorectal

D. Prostate √

E. Testicular

251. Which is the most common site of metastiasis?

A. Pelvis

B. Spine √

C. Ribs

D. Long Bones

E. Skull

252. A 54-year-old woman with chronic obstructive pulmonary disease (COPD) is prescribed an inhaled corticosteroid. What is the main therapeutic benefit of inhaled corticosteroids in patients with COPD?

A. Reduced severity of exacerbations

B. Improved all cause mortality

C. Reduced use of bronchodilators

D. Slows decline in FEV1

E. Reduced frequency of exacerbations √

253. A 36-year-old former intravenous drug user is to commence treatment for hepatitis C with interferon-alpha and ribavirin. Which of the following adverse effects treatment are most likely to occur when patients are treated with interferon-alpha?

A. Diarrhoea and transient rise in ALT

B. Cough and haemolytic anaemia

C. Flu-like symptoms and transient rise in ALT

D. Haemolytic anaemia and flu-like symptoms

E. Depression and flu-like symptoms √

254. A 57-year-old woman with a history of polymyalgia rheumatica has been taking prednisolone 10 mg for the past 5 months. A DEXA scan is reported as follows:

L2 T-score -1.6 SD Femoral neck T-score -1.7 SD

What is the most suitable management?

A. No treatmentia

B. Vitamin D + calcium supplementation + repeat DEXA scan in 6 months

C. Vitamin D + calcium supplementation

D. Vitamin D + calcium supplementation + hormone replacement therapy

E. Vitamin D + calcium supplementation + oral bisphosphonate √

255. A 17-year-old girl presents to her GP with a 6 week history of nausea and abdominal discomfort. Routine blood tests reveal the following:

Hb= 10.9 g/dl WBC= 6.7 *109/l Platelets= 346 *109/l Calcium= 2.43 mmol/l

Bilirubin= 7 µmol/l ALP= 262 u/l ALT= 35 u/l

What is the most likely diagnosis?

A. Alcoholic liver disease

B. Cholangiocarcinoma

C. Pregnancy √

D. Gallstones

E. Primary biliary cirrhosis

256. A 47-year-old female is reviewed in the neurology clinic. She was diagnosed with epilepsy whilst a teenager and her seizures are well controlled. She is however concerned about increasing numbness of her fingers and soles of her feet. Which one of the following medications is most likely to be responsible?

A. Phenytoin √

B. Lamotrigine

C. Sodium valproate

D. Ethosuximide

E. Levetiracetam

257. A 43-year-old woman with multiple sclerosis presents for review. She is having increasing problems with painful involuntary contractions of the leg muscles. What is the most appropriate first-line therapy?

A. Referral for relaxation therapy

B. Baclofen √

C. Diazepam

D. Dantrolene

E. Natalizumab

258. A 36-year-old man presents with dyspepsia. No alarm symptoms are present. This is his first episode and he has no significant medical history of note. A test-and-treat strategy is agreed upon. What is the most appropriate investigation to test for Helicobacter pylori?

A. Gastric aspiration + culture

B. CLO test (rapid urease test)

C. Stool culture

D. Hydrogen breath test

E. 13C-urea breath test √

Theme: Dysphagia



A. Pharyngeal pouch

B. Achalasia

C. Kaposi's sarcoma

D. Systemic sclerosis

E. Oesophageal cancer

F. Myasthenia gravis

G. Oesophagitis

H. Motor neuron disease

I. Oesophageal candidiasis

J. Plummer-Vinson syndrome



For each one of the following scenarios please select the most likely diagnosis:

259. An 80-year-old man presents with dysphagia. He reports being reasonably well in himself other than an occasional cough. The dysphagia occurs with both liquids and solids. Clinical examination is normal. Achalasia

260. A 55-year-old woman presents with swallowing difficulties for the past 5 weeks. She has also noticed some double vision Myasthenia gravis

261. A 42-year-old haemophiliac who is known to be HIV positive presents with pain on swallowing for the past week. He has been generally unwell for the past 3 months with diarrhoea and weight loss Oesophageal candidiasis





262. A 34-year-old man who is known to have glucose-6-phosphate dehydrogenase deficiency presents to his GP with symptoms of a urinary tract infection. He is prescribed an antibiotic. A few days later he becomes unwell and is noticed by his partner to be pale and jaundiced. What drug is mostly likely to have been prescribed?

A. Co-amoxiclav

B. Trimethoprim

C. Ciprofloxacin √

D. Cefalexin

E. Erythromycin

263. Which one of the following statements regarding prosthetic heart valves is correct?

A. Antibiotic prophylaxis is still recommended for patients with mechanical valves who have dental procedures

B. The majority of mechanical valves are of the ball-and-cage type

C. Bioprosthetic valves are now usually obtained from human cadavers

D. The target INR for patients with mechanical aortic valves is 3.0-4.0

E. Mechanical valves have a lower failure rate than bioprosthetic valves √

264. Which one of the following statements regarding B-type natriuretic peptide is incorrect?

A. Effective treatment for heart failure lowers a patients BNP level

B. Acts as a diuretic

C. A hormone produced mainly by the left ventricular myocardium in response to strain

D. Is a good marker of prognosis in patients with chronic heart failure

E. The positive predictive value of BNP is greater than the negative predictive value √

265. A 61-year-old female is reviewed in the rheumatology clinic with increasing shortness of breath. She has been on long-term drug therapy to control her rheumatoid arthritis. Her oxygen saturations on room air are on 89%. Investigations reveal the following:

Chest x-ray Bilateral interstitial shadowing

Which drug is most likely to be responsible for her symptoms?

A. Infliximab

B. Hydroxychloroquine

C. Sulphasalazine

D. Azathioprine

E. Methotrexate √

266. Which one of the following side-effects is not recognised in patients taking sodium valproate?

A. Alopecia

B. Weight gain

C. Hepatitis

D. Induction P450 system √

E. Teratogenicity

267. Which one of the following features is least typical of polymyalgia rheumatica?

A. Elevated creatinine kinase

B. Low-grade fever

C. Morning stiffness in proximal limb muscles

D. Polyarthralgia

E. Anorexia √

268. Which one of the following statements best describes the prevention and treatment of hepatitis C?

A. No vaccine is available and treatment is only successful in around 10-15% of patients

B. No vaccine and no treatment is available

C. A vaccine is available and treatment is successful in around 50% of patients √

D. A vaccine is available but no treatment has been shown to be effective

E. No vaccine is available but treatment is successful in around 50% of patients

269. A 23-year-old woman presents to her GP due to lethargy. The following blood results are obtained:

Hb 10.4 g/dl Plt 278 * 109/l WCC 6.3 * 109/l MCV 65 fl

HbA2 4.5% (< 3%)

What is the most likely diagnosis?

A. Beta-thalassaemia major

B. Sickle cell anaemia

C. Beta-thalassaemia trait √

D. Hereditary spherocytosis

E. Acute lymphoblastic leukaem





270. A 55-year-old man presents with multiple erythematous target lesions two days after starting a new medication. Which one of the following drugs is most likely to have been started?

A. Levetiracetam

B. Olanzapine √

C. Carbamazepine

D. Fluoxetine

E. Diazepam

Theme: Respiratory tract infections: NICE guidelines



A. 2 days

B. 4 days

C. 7 days

D. 10 days

E. 2 weeks

F. 2 1/2 weeks

G. 3 weeks

H. 4 weeks



For each one of the following respiratory tract infections select the average total illness length:

271. Acute otitis media 10 days

272. Common cold 7 days

273. Acute sore throat 10 days

274. A 39-year-old man is diagnosed as having cluster headaches. He has received subcutaneous sumatriptan on two occasions but would like to start medication to help prevent further attacks. Of the following options, which one is the most suitable treatment?

A. Atenolol

B. Amitriptyline

C. Sodium valproate

D. Verapamil √

E. Gabapentin

275. An obese 24-year-old female presents with headaches and blurred vision to her GP. Examination reveals bilateral blurring of the optic discs but is otherwise unremarkable with no other neurological signs. Blood pressure is 130/74 and she is apyrexial. What is the most likely underlying diagnosis?

A. Multiple sclerosis

B. Meningococcal meningitis

C. Brain abscess

D. Normal pressure hydrocephalus

E. Idiopathic intracranial hypertension √

276. Which one of the following symptom is least associated with salicylate overdose?

A. Tremor √

B. Tinnitus

C. Hyperventilation

D. Seizures

E. Nausea

277. A 45-year-old female presents to her GP with a two month history of lethargy. Blood tests reveal the following:

Na+= 128 mmol/l K+= 5.6 mmol/l Urea= 5.3 mmol/l Creatinine= 99 µmol/l

Total T4= 66 nmol/l (70 - 140 nmol/l)

Which one of the following investigations is most likely to reveal the diagnosis?

A. Serum glucose

B. TSH

C. Free T4

D. Overnight dexamethasone suppression test

E. Short synacthen test √

278. A patient attends the local walk-in centre with central crushing chest pain. The nurse phones 999 and performs an ECG which shows ST elevation in leads II, III and aVF. What is the most appropriate management whilst waiting for the ambulance?

A. Oxygen + aspirin 300mg + clopidogrel 300mg + sublingual glyceryl trinitrate √

B. Aspirin 300mg + sublingual glyceryl trinitrate

C. Oxygen + aspirin 300mg + sublingual glyceryl trinitrate

D. Aspirin 300mg

E. Oxygen + aspirin 300mg + lansoprazole 30mg + sublingual glyceryl trinitrate





279. A 65-year-old man with a history of hypertension is reviewed. As part of routine blood tests to monitor his renal function whilst taking ramipril the following blood tests are received:

Na+ 140 mmol/l K+ 4.8 mmol/l Urea 6.2 mmol/l Creatinine 102 µmol/l

eGFR 68 ml/min

A urine dipstick is subsequently performed which is normal and a renal ultrasound sound shows normal sized kidneys with no abnormality detected. What stage of chronic kidney disease does this patient have?

A. No chronic kidney disease √

B. Chronic kidney disease stage 4

C. Chronic kidney disease stage 3

D. Chronic kidney disease stage 2

E. Chronic kidney disease stage 1

280. A 50-year-old man with no past medical history is investigated for ongoing back pain. He is found to have a vertebral collapse secondary to osteoporosis. What is the most appropriate test to determine the cause of his osteoporosis?

A. Thyroid function tests

B. Prostate specific antigen

C. Oestrogen level

D. Prolactin level

E. Testosterone level √

281. A 34-year-old is diagnosed with chronic fatigue syndrome. Which one of the following interventions is least useful?

A. Advice to go to the Gym √

B. Cognitive behaviour therapy

C. Referral to a pain management clinic if pain is a predominant feature

D. Advice about pacing of activities

E. Low-dose amitriptyline for poor sleep

282. A 31-year-old woman presents with symptoms consistent with coeliac disease. Which one of the following tests should be used first-line when screening patients for coeliac disease?

A. Anti-casein antibodies

B. Tissue transglutaminase antibodies √

C. Anti-gliadin antibodies

D. Xylose absorption test

E. Anti-endomyseal antibodies

Theme: Genital ulceration



A. Lymphogranuloma venereum

B. Behcet's disease

C. Herpes simplex

D. Chancroid

E. Carcinoma

F. Granuloma inguinale

G. Reiter's syndrome

H. Antiphospholipid syndrome

I. Donovanosis

J. Syphilis



For each of the following scenarios please select the most likely diagnosis:

283. A 27-year-old woman presents with painful genital and oral ulceration. Her past medical history includes treatment for a deep vein thrombosis three years ago. Behcet's disease

284. A 19-year-old woman presents with multiple painful blisters and ulcers around her labia. She has been feeling like she has the flu for the past five days. It is extremely painful when she urinates. Herpes simplex

285. A 23-year-old man presents with an ulcer on the coronary sulcus of the penis. The ulcer is not causing him any discomfort. On examination an ulcer with an erythematous border and a clean base is found. Syphilis

286. Which one of the following interventions is most likely to increase survival in patients with COPD?

A. Home nebulisers

B. Prophylactic antibiotic therapy

C. Pulmonary rehabilitation

D. Long-term steroid therapy

E. Long-term oxygen therapy √

287. Which one of the following is least associated with cocaine toxicity?

A. Metabolic alkalosis √

B. Hyperthermia

C. Psychosis

D. Rhabdomyolysis

E. Seizures

288. A 12-year-old boy who had a splenectomy following a road traffic accident is reviewed in clinic. He had his full immunisation course as a child and was given a repeat pneumococcal vaccination 5 days following surgery. What is the most appropriate ongoing management?

A. Booster dose of Hib and MenC vaccine + lifelong penicillin V

B. Booster dose of Hib and MenC vaccine + penicillin V for 1 year

C. Lifelong penicillin V

D. Booster dose of Hib and MenC vaccine + annual influenza vaccination + penicillin V for 1 year

E. Booster dose of Hib and MenC vaccine + annual influenza vaccination + lifelong penicillin V √

289. A 64-year-old female is referred to rheumatology out-patients by her GP with a history of arthritis in both hands. Which one of the following x-ray findings would most favour a diagnosis of rheumatoid arthritis over other possible causes?

A. Loss of joint space

B. Juxta-articular osteoporosis √

C. Subchondral sclerosis

D. Osteophytes

E. Subchondral cysts

290. A 45-year-old female with a history of epilepsy is reviewed in the surgery. Which one of the following features is most likely to be attributable to sodium valproate therapy?

A. Clubbing

B. Weight loss

C. Hirsuitism

D. Renal impairment

E. Tremor √

291. A 65-year-old female with metastatic breast cancer is reviewed in clinic. Her husband reports that she is increasingly confused and occasionally appears to talk to relatives that are not in the room. Following investigations for reversible causes, what is the most appropriate management?

A. Subcutaneous midazolam

B. Oral lithium

C. Oral haloperidol √

D. Oral diazepam

E. Oral quetiapine

292. A 54-year-old female is presents to surgery one week following a cholecystectomy with profuse diarrhoea. What is the most likely diagnosis?

A. Campylobacter

B. E.Coli

C. Clostridium difficile √

D. Salmonella

E. Staphylococcus aureus

293. Which one of the following is true regarding anti-tuberculous therapy?

A. Rifampicin is a potent liver enzyme inhibitor

B. Pyrazinamide should be added to therapy after 8 weeks

C. Major side-effects of pyrazinamide include peripheral neuropathy

D. Visual acuity should be checked before starting ethambutol √

E. Ethambutol should only be added if drug-resistant tuberculosis is suspected

294. Which one of the following is least recognised as a cause of vertigo?

A. Gentamicin

B. Meniere's disease

C. Acoustic neuroma

D. Multiple sclerosis

E. Motor neuron disease √

295. A 54-year-old man with hypertension is reviewed in clinic. He complains that over the past two months he has developed ankle swelling. Which one of the following drugs is most likely to be responsible?

A. Sodium valproate

B. Nifedipine √

C. Doxazosin

D. Moxonidine

E. Losartan

296. Each of the following drugs are known to induce cytochrome p450 enzyme, except:



A. Rifampicin

B. Isoniazid √

C. Phenobarbitone

D. Griseofulvin

E. Carbamazepine





297. A 52-year-old man presents to his GP concerned about a discharge from his nipples. Which one of the following drugs may be responsible?

A. Cimetidine

B. Isoniazid

C. Digoxin

D. Spironolactone

E. Chlorpromazine √

298. An 80-year-old woman is started on oral alendronate following a fractured neck of femur. How would you explain how to take the tablet?

A. Take it on a full stomach to minimise gastric irritation and avoid lying down for 30 minutes afterwards

B. Dissolve tablet in water and take just before breakfast + sit-upright for 30 minutes following

C. Take during main evening meal + sit-upright for 2 hours following

D. Take at least 30 minutes before breakfast with plenty of water + sit-upright for 30 minutes following √

E. Take at least 30 minutes after a main meal + sit-upright for 30 minutes following

299. A 67-year-old with chronic kidney disease stage 4 and metastatic prostate cancer presents as his pain is not controlled with Co-codamol. Which one of the following opioids is it most appropriate to use given his impaired renal function?

A. Buprenorphine √

B. Morphine

C. Hydromorphone

D. Diamorphine

E. Tramadol

300. A man is found to have a mass in the left upper quadrant during abdominal exam. Which one of the following is least likely to cause splenomegaly?

A. Malaria

B. Sickle cell anaemia √

C. Visceral leishmaniasis

D. Chronic myeloid leukaemia

E. Myelofibrosis

301. A 23-year-old male presents with a purulent urethral discharge. A sample of the discharge is shown to be a Gram negative diplococcus. What is the most appropriate antimicrobial therapy?

A. Ciprofloxacin 500mg BD PO for 7 days

B. Penicillin V 500mg QDS PO for 7 days

C. Doxycycline 100mg BD PO for 7 days

D. Ciprofloxacin 500mg PO (single dose)

E. Cefixime 400mg PO (single dose) √

302. One of your patients is diagnosed with having the metabolic syndrome. Which one of the following is associated with this condition?

A. Endometriosis

B. Hypothyroidism

C. Asymptomatic rise in amylase levels

D. Elevated albumin levels

E. Raised uric acid levels √

303. A 52-year-old female presents to surgery with weakness and pins and needles in her right hand. On examination she has wasting of the thenar eminence associated with sensory loss to the palmar aspect of lateral (radial) three fingers. Which nerve is likely to be affected?

A. Common peroneal nerve

B. Median nerve √

C. Radial nerve

D. Anterior interosseous nerve

E. Ulnar nerve

Theme: Heart failure: drug management



A. Pneumococcal vaccine

B. Influenza vaccine

C. Beta-blocker

D. Spironolactone

E. ACE inhibitor

F. Hydralazine + nitrates

G. Frusemide

H. Digoxin

I. Echocardiogram

J. Electrocardiogram



For each one of the following select the most appropriate answer:

304. Should be arranged annually for all patients with heart failure Influenza vaccine

305. Should be introduced first-line in patients with stable impaired left ventricular function Beta-blocker

306. Has only been demonstrated to improve mortality in patients with NYHA class III or IV heart failure who are already taking an ACE inhibitor Spironolactone

307. A 25-year-old man presents with two day history of fever and arthralgia. On examination the following rash is seen: What is the most likely diagnosis?

A. Legionella

B. Leptospirosis

C. Lyme disease √

D. Actinomycosis

E. Rheumatic fever

308. What is the type of rash shown in picture?

A. Erythema multiformis

B. Erythems nodosum

C. Erythema Circinate

D. Erythema Marginatum

E. Erythems Chronicum Migranes √

309. A 41-year-old man presents with persistent fatigue for the past 8 months. Which one of the following features is least consistent with a diagnosis of chronic fatigue syndrome?

A. Dizziness

B. Painful lymph nodes without enlargement

C. Having a busy day improves the symptoms √

D. Palpitations

E. Headaches

310. A 67-year-old man with a history of atrial fibrillation and ischaemic heart disease presents with symptoms consistent with a chest infection. His current medication includes amiodarone, warfarin and simvastatin. Which one of the following antibiotics is it most important to avoid if possible?ia

A. Trimethoprim

B. Co-amoxiclav

C. Cefaclor

D. Levofloxacin

E. Erythromycin √

311. A 65-year-old man is referred to the neurology outpatient clinic due to a resting tremor of his right hand. A diagnosis of Parkinson's disease is made. He is otherwise well and is not currently disabled by his symptoms. What is the most appropriate treatment?

A. Selegiline

B. No treatment √

C. New generation dopamine receptor agonist e.g. ropinirole

D. Conventional dopamine receptor agonist e.g. bromocriptine

E. Antimuscarinics

312. An 84-year-old nursing home resident is treated for a urinary tract infection. During the night she is found wandering outside the nursing home in an agitated state. Despite appropriate antibiotic therapy, nursing care and modification of her environment she remains agitated and aggressive and it is judged a potential danger to herself. What is the most appropriate management?

A. Haloperidol 5 mg orally

B. Lorazepam 2 mg intramuscularly

C. Haloperidol 0.5 mg orally √

D. Diazepam 5 mg orally

E. Ask for on-call psychiatric opinion for consideration of section under the Mental Health Act

313. Which one of the following may be associated with an increased risk of venous thromboembolism?

A. Fluoxetine

B. Selegiline

C. Diazepam

D. Amitriptyline

E. Olanzapine √

314. A phlebotomist in the surgery sustains a needlestick injury whilst taking blood from a patient who is known to be HIV positive. Following thorough washing of the wound what is the most appropriate management?

A. HIV test of phlebotomist in 3 months to determine treatment

B. Refer to GUM for immediate p24 HIV test of phlebotomist to determine treatment

C. Refer to Emergency Department + oral antiretroviral therapy for 4 weeks √

D. Refer to Emergency Department + oral antiretroviral therapy for 3 months

E. Reassure low risk of transmission

315. A man with glucose-6-phosphate dehydrogenase deficiency asks for advice regarding his son. Given the x-linked recessive inheritance of the condition, what is the chance his son will also develop the disease?

A. 2 in 3

B. No increased risk √

C. Will definitely be affected

D. 1 in 2

E. 1 in 4

316. An 82-year-old man is reviewed. He is known to have ischaemic heart disease and is still getting regular attacks of angina despite taking atenolol 100mg od. Examination of his cardiovascular system is unremarkable with a pulse of 72 bpm and a blood pressure of 158/96 mmHg. What is the most appropriate next step in management?

A. Add verapamil 80mg tds

B. Add nicorandil 10mg bd √

C. Add diltiazem 60mg tds

D. Add nifedipine MR 30mg od

E. Add isosorbide mononitrate 30mg bd

Theme: NICE heart failure guidelines



Digoxin
β-Blocker
ARBs
ACE inhibitors
Spironolatone


The diagram below is taken from the NICE guidelines on the management of heart failure:


For each of the numbered gaps above, please select one option from the list:





317. Gap (1) ACE inhibitors



318. Gap (2) ARBs



319. Gap (3) β-Blocker



320. Gap (4) Spironolatone



321. Gap (5) Digoxin











322. A 52-year-old man has a set of fasting bloods as part of a work-up for hypertension. The fasting glucose comes back as 6.5 mmol/l. The test is repeated and reported as 6.7 mmol/l. He says he feels constantly tired but denies any polyuria or polydipsia. How should these results be interpreted?

A. Impaired fasting glycaemiaia √

B. Suggestive of diabetes mellitus but not diagnosticia

C. Diabetes mellitusia

D. Normalia

E. Impaired glucose toleranceia

323. Which one of the following drugs may be safely continued at the same dose in renal failure?ia

A. Tetracyclineia

B. Morphineia

C. Warfarinia √

D. Nitrofurantoinia

E. Lithiumia

324. A 71-year-old patient presents to the Emergency Department with a 30 minute history of crushing central chest pain. ECG shows tall R waves in V1-2. Which coronary territory is likely to be affected?ia

A. Lateralia

B. Posterioria √

C. Anteroseptalia

D. Anterolateralia

E. Inferioria

325. A 50-year-old sewage worker presents with a one week history of fever and feeling generally unwell. Which one of the following features would be least consistent with a diagnosis of leptospirosis?i

A. Meningismia

B. Conjunctival erythemaia

C. Productive coughia √

D. Decreased urine outputia

E. Severe myalgiaia

326. Which one of the following is an established indication for the use of Botulinum toxin?

A. Strabismusia

B. Hirschsprung's diseaseia

C. Blepharospasmia √

D. Bell's palsyia

E. Upper limb rigidity in Parkinson's diseaseia

327. You refer a 24-year-old female to rheumatology with intermittent pain and swelling of the metacarpal phalangeal joints for the past 3 months. An x-ray shows loss of joint space and soft-tissue swelling. Rheumatoid factor is positive and a diagnosis of rheumatoid arthritis is made. What initial management is she most likely to be given to help slow disease progression?ia

A. Infliximabia

B. Methotrexateia

C. Sulfasalazineia

D. Methotrexate + sulfasalazine + short-course of prednisoloneia √

E. Diclofenacia

328. A 45-year-old man with a history of alcoholic liver disease presents with abdominal distension. Examination reveals tense ascites which is drained. What is the appropriate type of diuretic to help prevent reaccumulation of ascites?i

A. Aldosterone antagonistia √

B. Loop diureticia

C. Thiazide diureticia

D. Osmotic diureticia

E. Carbonic anhydrase inhibitoria

329. Which one of the following is most likely to indicate an underlying connective tissue disorder in a patient with Raynaud's phenomenon?

A. Chilblainsia

B. Bilateral symptomsia

C. Female patientia

D. Onset at 18 years oldia

E. Recurrent miscarriagesia √

330. A 34-year-old female with a history of depression is reviewed. She is currently taking St John's Wart which she bought from the local health food shop and a combined oral contraceptive pill. What is the most likely affect of taking both medications concurrently?

A. Worsening of depressive symptomsia

B. Increased risk of severe skin reactionsia

C. Increased risk of serotonin syndromeia

D. Increased risk of venous thromboembolismia

E. Reduced effectiveness of combined oral contraceptive pill √

Malaria: prophylaxis



A. Quinine

B. Chloroquine

C. Atovaquone + proguanil

D. Doxycycline

E. Proguanil

F. Artemether + lumefantrine

G. Primaquine



For each one of the following statements, select the type of malaria prophylaxis from the list of options:

331. Should be avoided if the patient has a history of epilepsy Chloroquine

332. Can be stopped just one week after returning from a malarial zone Atovaquone + proguanil

333. Is taken weekly Chloroquine

334. A 24-year-old female presents complaining of a painful lower lip. She has recently been on holiday to Mexico. What is the most likely causative organism?


A. Coxsackie A virusia

B. Leishmania donovaniia

C. Herpes simplex virus type 1ia √

D. Staphylococcus aureusia

E. Herpes simplex virus type 2ia



335. A 24-year-old man comes to the surgery as he is planning an expedition to the Andes. He asks for advice on preventing acute mountain sickness (AMS), other than gradual ascent. What is the most appropriate advice?i

A. Acetazolamide (Diamox)ia √

B. Non-steroid anti-inflammatoriesia

C. Ensure maximal physical fitness prior to tripia

D. Dexamethasone starting 2 days prior to arrivalia

E. There is no evidence of any effective intervention to prevent AMSia

336. A 70-year-old man who takes warfarin for atrial fibrillation is found to have an INR of 6.2. Which of the following drugs is he most likely to have recently taken?ia

A. Ciprofloxacinia

B. Flucloxacillinia

C. St John's Wortia √

D. Carbamazepineia

E. Aspirinia

337. A 34-year-old man with a history of migraine finds that paracetamol taken at the recommend dose often fails to relieve his acute attacks. He drinks 12 units of alcohol per week and smokes 15 cigarettes per day. What factor is likely to contribute to this problem?

A. Bacterial overgrowthia

B. Delayed gastric emptyingia √

C. P450 enzyme inductionia

D. First pass metabolismia

E. P450 enzyme inhibitionia

338. An 28-year-old man is investigated for recurrent lower back pain. A diagnosis of ankylosing spondylitis is suspected. Which one of the following investigations is most useful?ia



A. Anti-nuclear antibodiesia

B. X-ray of the sacro-iliac jointsia

C. HLA-B27 testingia √

D. X-ray of the thoracic spineia

E. Spirometryia



339. A 71-year-old man presents with two year history of intermittent problems with swallowing. His wife has also noticed he has halitosis and is coughing at night. He has a past medical history of type 2 diabetes mellitus but states he is otherwise well. Of note his weight is stable and he has a good appetite. Clinical examination is unremarkable. What is the most likely diagnosis?ia



A. Oesophageal canceria

B. Hiatus herniaia

C. Pharyngeal pouchia √

D. Oesophageal candidiasisia

E. Benign oesophageal strictureia



340. A 62-year-old man presents to surgery complaining of painful gums. On examination he is noted to have gingival hyperplasia. Which one of the following drugs is most likely to be responsible?

A. Carvedilolia

B. Amiodaroneia

C. Atorvastatinia

D. Nifedipineia √

E. Digoxinia

341. A 62-year-old man who had a myocardial infarction six months ago presents for review. What should his target cholesterol levels be?

A. Total cholesterol < 3.5 mmol/l; LDL < 1.5 mmol/lia

B. Total cholesterol < 4.0 mmol/l; LDL < 2.0 mmol/lia √

C. Total cholesterol:HDL ratio < 5.0 mmol/lia

D. Total cholesterol:HDL ratio < 4.0 mmol/lia

E. Total cholesterol < 5.0 mmol/l; LDL < 3.0 mmol/lia

342. A 39-year-old man returns from a two week business trip to Kenya. Four weeks after his return he presents to his GP complaining of malaise, headaches and night sweats. On examination there is a symmetrical erythematous macular rash over his trunk and limbs associated with cervical and inguinal lymphadenopathy. What is the most likely diagnosis? i

A. Typhoid feveria

B. Tuberculosisia

C. Dengue feveria

D. Schistosomiasisia

E. Acute HIV infection √

Theme: Fitness to fly



A. No restriction

B. 24 hours

C. 48 hours

D. 5 days

E. 7 days

F. 10 days

G. 3 weeks

H. 4 weeks

I. Cannot fly



For each one of the following scenarios select the earliest time that the patient can fly:

343. Colonoscopy 24 hours

344. Coronary artery bypass graft 10 days

345. Laparoscopic cholecystectomy 24 hours

346. A 48-year-old salesman presents with a 5 day history of cough and
pleuritic chest pain. He has no past medical history of note. On examination his temperature is 38.2ºC, blood pressure is 120/80 mmHg, respiratory rate 18/min and pulse 84/min. Auscultation of the chest reveals decreased air entry in the left base and the same area is dull to percussion. What is the most suitable management?

A. Oral amoxicillinia

B. Oral co-amoxiclavia

C. Oral amoxicillin + erythromycinia

D. Oral erythromycinia

E. Admitia √

347. A 69-year-old female with a history of multiple myeloma presents with confusion. Blood tests are taken and the following results are obtained:

Adjusted calcium 3.1 mmol/l

What is the most appropriate initial management?

A. Oral alendronate + prednisoloneia

B. Oral alendronateia

C. Oral prednisoloneia

D. Admit for IV pamidronateia

E. Admit for IV normal salineia √

348. A 56-year-old lady with a BMI of 27 is reviewed in the diabetic clinic due to poor glycaemic control. She is currently being treated with gliclazide 160mg bd. Her latest bloods show:

Na+=139 mmol/l K+=4.1 mmol/l Urea=8.4 mmol/l Creatinine=170 µmol/l

ALT=25 iu/l yGT=33 iu/l HbA1c=9.4%

Which one of the following medications should be added next?

A. Guar gumia

B. Pioglitazoneia √

C. Metforminia

D. Acarboseia

E. Repaglinideia

349. A 65-year-old man is discharged from hospital following a thrombolysed ST-elevation myocardial infarction. Other than a history of depression he has no past medical history of note. His stay on the coronary care unit was complicated by the development of dyspnoea and an echo show a reduced left ventricular ejection fraction. The patient was not given clopidogrel during his hospital admission. Other than standard treatment with an ACE inhibitor, beta-blocker, aspirin and statin, what other type of drug should he be taking?

A. Angiotensin 2 receptor antagonistia

B. Potassium channel activatoria

C. Aldosterone antagonistia √

D. Thiazide diureticia

E. Clopidogrelia

350. A 25-year-old man returns from a gap-year in Central and South America and presents with a 2 month history of an ulcerating lesion on his lower lip. Examination of his nasal and oral mucosae reveals widespread involvement. What is the likely diagnosis?

A. Leishmaniasisia √

B. Chagas diseaseia

C. Cutaneous larva migransia

D. Trypanosomiasisia

E. Cutaneous gonococcal infectionia

351. A 51-year-old man is noticed to have the following hand abnormality: Which one of the following is least likely to be responsible?


A. Bacterial endocarditisia √

B. Empyemaia

C. Fibrosing alveolitisia

D. Coeliac diseaseia

E. Lung canceri







352. Which one of the following statements regarding allergy testing is incorrect?i

A. Both irritants and allergens may be tested for using skin patch testingia

B. The radioallergosorbent test determines the level of IgE to a specific allergenia

C. Skin prick testing is easy to perform and inexpensiveia

D. Skin prick testing should be read after 48 hoursia √

E. Skin prick testing normally includes a histamine controlia

353. A 45-year-old man with a history of epilepsy and psychiatric problems is admitted to the Emergency Department with confusion following a seizure earlier in the day. On examination he is noted to have a coarse tremor, blood pressure = 134/86 mmHg, pulse = 84/min regular and temperature = 36.7ºC. What is the most likely diagnosis?

A. Carbamazepine overdoseia

B. Lithium toxicityia √

C. Benzodiazepine toxicityia

D. Tricyclic overdoseia

E. Neuroleptic malignant syndromeia

354. A patient is diagnosed with type 2 diabetes mellitus. What target should be set for the HbA1c?

A. Agree target with patient but generally aim for 7.0%ia

B. Agree target with patient but generally aim for 6.0%ia

C. As low as possibleia

D. Agree target with patient but generally aim for 6.5%ia √

E. 6.5%ia

Theme: Drug monitoring



A. U&E, LFT

B. TFT, LFT

C. TFT, U&E

D. LFT

E. FBC, U&E

F. FBC, LFT

G. U&E

H. Calcium

I. ECG

J. No routine monitoring required



For each of the following drugs select the most appropriate monitoring tests once treatment has commenced:

355. Perindopril U&E

356. Sodium valproate during the first 6 months of therapy Calcium

357. Rosiglitazone LFT

358. A patient is started on cyclophosphamide for vasculitis associated with Wegener's granulomatosis. Which of the following is most characteristically associated with cyclophosphamide?

A. Haemorrhagic cystitisia √

B. Cardiomyopathyia

C. Ototoxicityia

D. Alopeciaia

E. Weight gainia

359. A 39-year-old patient is taking phenelzine, a monoamine oxidase inhibitor, for the treatment of depression. Which one of the following foods can the patient eat safely?ia

A. Bovrilia

B. Cheeseia

C. Oxoia

D. Eggsia √

E. Broad beansia

360. A 62-year-old female is reviewed in the nurse-led heart failure clinic. Despite current treatment with furosemide, bisoprolol, enalapril and spironolactone she remains breathless on minimal exertion. On examination the chest is clear to auscultation and there is minimal ankle oedema
Recent results are as follows:

ECG Sinus rhythm, rate 84 bpm

Chest x-ray Cardiomegaly, clear lung fields

Echo Ejection fraction 35%

What additional medication would best help her symptoms?

A. Bosentania

B. Isosorbide mononitrateia

C. Diltiazemia

D. Ivabradineia

E. Digoxinia √





361. A patient who was commenced on a simvastatin six months ago presents with generalised muscles aches. Which one of the following is not a risk factor for statin-induced myopathy?

A. Female genderia

B. Large fall in LDL-cholesterolia √

C. Low body mass indexia

D. Advanced ageia

E. History of diabetes mellitusia

362. A nurse undergoes primary immunisation against hepatitis B. Levels of which one of the following should be checked four months later to ensure an adequate response to immunisation?

A. Anti-HBsia √

B. Anti-HBcia

C. Hepatitis B viral loadia

D. HbeAgia

E. HBsAgia

363. The presence of anti-cyclic citrullinated peptide antibody is suggestive of which one of the following conditions?ia



A. Systemic lupus erythematousia

B. Rheumatoid arthritisia √

C. Type 1 diabetes mellitusia

D. Addison's diseaseia

E. Dermatomyositisia



364. A 62-year-old man attends your clinic. He has a history of hypertension and atrial fibrillation for which he is anticoagulated with warfarin. A urine dipstick taken 8 weeks ago during a routine hypertension clinic appointment showed blood +. This has been repeated on two further occasions. What is the most appropriate action?

A. Take no further actionia

B. Send a 24-urine sample for protein estimationia

C. Refer to nephrologyia √

D. Refer to urologyia

E. Confirm with urine microscopyia

365. A 60-year-old man is referred to the rapid access TIA clinic after experiencing a number of transient episodes of right-sided weakness. Which one of the following factors is not part of ABCD2 score used to estimate his risk of future stroke?



A. Clinical featuresia

B. History of diabetes mellitusia

C. Blood pressureia

D. History of aspirin useia √

E. Duration of symptomsia



366. A 15-year-old boy is diagnosed with glandular fever. What is the most appropriate advice to give regarding playing sports?

A. Can play contact sports as normalia

B. Avoid contact sports for 2 weeks after having glandular fever if clinical evidence of splenomegalyia

C. Avoid contact sports for 2 weeks after having glandular feveria

D. Avoid contact sports for 8 weeks after having glandular feveria

E. Avoid contact sports for 8 weeks after having glandular fever if clinical evidence of splenomegalyi √

Theme: Tumour markersia



A. Breast cancer

B. Prostate cancer

C. Gastric cancer

D. Ovarian cancer

E. Colorectal cancer

F. Pancreatic cancer

G. Seminoma testicular cancer

H. Non-seminomatous testicular cancer

I. Liver cancer



For each one of the following select the most likely diagnosis from the options above:

367. Raised beta-human chorionic gonadotropin with a raised alpha-feto protein level Non-seminomatous testicular cancer

368. Elevated CA 19-9 Pancreatic cancer

369. Raised alpha-feto protein level in a 54-year-old woman Liver cancer

370. A 33-year-old man is seen in the asthma clinic. He was referred with poorly control asthma and has recently had salmeterol added in addition to beclometasone dipropionate inhaler 200mcg bd and salbutamol prn. There has unfortunately been no response to adding the salmeterol. What is the most appropriate action?

A. Stop salmeterol + trial of leukotriene receptor antagonistia

B. Continue salmeterol + increase beclometasone dipropionate inhaler to 400mcg bdia

C. Continue salmeterol + trial of leukotriene receptor antagonistia

D. Stop salmeterol + trial of oral theophyllineia

E. Stop salmeterol + increase beclometasone dipropionate inhaler to 400mcg bdia √

371. A 44-year-old man with a BMI of 37 kg/m2 presents to his GP. Despite diet and exercise he has failed to lose a significant amount of weight. His GP considers starting sibutramine. What is the mechanism of action of sibutramine?ia

A. Serotonin and noradrenaline uptake inhibitoria √

B. Leptin antagonistia

C. CB1 cannabinoid receptor antagonistia

D. Blocks intestinal absorption of lipidsia

E. Pancreatic lipase inhibitoria

372. Primary biliary cirrhosis is most characteristically associated with:ia

A. Anti-nuclear antibodiesia

B. Anti-ribonuclear protein antibodiesia

C. Anti-mitochondrial antibodiesia√

D. Rheumatoid factoria

E. Anti-neutrophil cytoplasmic antibodiesia

373. A 52-year-old obese lady with type 2 diabetes mellitus is interested in changing her diet. Which one of the following foods has the highest glycaemic index?

A. Baked potatoia √

B. Appleia

C. Peanutia

D. Digestive biscuitia

E. Brown riceia

374. A 72-year-old woman is investigated for shortness of breath. Auscultation of the lungs reveals fine bibasal crackles. Which one of the following set of results would be most consistent with a diagnosis of pulmonary fibrosis?ia

A. FEV1 - reduced, FEV1/FVC - reducedia

B. FEV1 - increased, FEV1/FVC - reducedia

C. FVC - increased, FEV1/FVC - increasedia

D. FEV1 - normal, FEV1/FVC - reducedia

E. FVC - reduced, FEV1/FVC - normalia √

375. A 27-year-old woman with chronic left iliac fossa pain and alternating bowel habit is diagnosed with irritable bowel syndrome. Initial treatment is tried with a combination of antispasmodics, laxatives and anti-motility agents. Unfortunately after 6 months there has been no significant improvement in her symptoms. According to recent NICE guidelines, what is the most appropriate next step?ia

A. Low-dose tricyclic antidepressantia √

B. Cognitive behavioural therapyia

C. Refer for sigmoidoscopyia

D. Trial of probioticsia

E. Selective serotonin reuptake inhibitoria

376. Which one of the following is least recognised in patients taking amiodarone?ia

A. Pulmonary fibrosisia

B. Corneal depositsia

C. Peripheral neuropathyia

D. Hyperthyroidismia

E. Gynaecomastiaia √

377. Which one of the following statements regarding restless legs syndrome is incorrect?ia

A. Movements may be seen during sleepia

B. May be secondary to uraemiaia

C. Affects approximately 5% of the general populationia

D. Family history is found in up to 50% of patientsia

E. It is three times as common in femalesia √

378. Each one of the following is seen in Amiodarone therapy, except:

A. Hyperthyroidismia

B. Liver cirrhosisia

C. Hypokalaemiaia √

D. Hypothyroidismia

E. Pulmonary fibrosisia







Theme: Chest painia



A. Myocardial infarction

B. Gastro-oesophageal reflux disease

C. Anxiety

D. Pleurisy

E. Pneumothorax

F. Pericarditis

G. Myocarditis

H. Dissecting aortic aneurysm

I. Pulmonary embolism

J. Shingles



For each one of the following scenarios select the most likely diagnosis:ia

379. A 42-year-old overweight man presents with a two day history of anterior chest pain that is worse on deep inspiration and laying down Pericarditis

380. A 67-year-old female with a history of chronic lymphocytic leukaemia presents with a 3 day history of burning pain in the right lower chest wall. Clinical examination is unremarkable Shingles

381. A 25-year-old man with a history of Marfan's disease presents with sudden onset shortness of breath and pleuritic chest pain Pneumothorax

382. A 15-year-old girl presents with abdominal pain. She is normally fit and well and currently takes a combined oral contraceptive pill. The patient is accompanied by her mother, who is known to have hereditary spherocytosis. The pain is located in the upper abdomen and is episodic in nature, but has become severe today. There has been no change to her bowel habit and no nausea or vomiting. What is the most likely diagnosis?i

A. Inferior vena cava thrombosisia

B. Acute pancreatitisia

C. Renal vein thrombosisia

D. Gastritisia

E. Biliary colicia

383. A 43-year-old man presents to surgery with lethargy. Examination is unremarkable apart from a blood pressure of 192/112 mmHg. Routine blood tests reveal:

Na+= 146 mmol/l K+= 2.4 mmol/l Bicarbonate= 34 mmol/l Urea= 5.3 mmol/l

Creatinine= 75 µmol/l

What is the most likely diagnosis?

A. Phaeochromocytomaia

B. Renal artery stenosisia

C. Diabetes mellitusia

D. Bartter's syndromeia

E. Primary hyperaldosteronismia √

384. Which one of the following drugs used in the management of tuberculosis is most associated with peripheral neuropathy?

A. Rifampicinia

B. Pyrazinamideia

C. Ethambutolia

D. Streptomycinia

E. Isoniazidia √

385. A 78-year-old woman with no past medical history of note presents with palpitations and shortness of breath, having been unwell for the past three days. Examination reveals an irregularly irregular pulse of 130 bpm, blood pressure of 108/70 mmHg, oxygen saturations of 97% on air and bibasal lung crepitations. She refuses admission to hospital. What is the most appropriate therapy to control her heart rate?ia

A. Amiodaroneia

B. Flecainideia

C. Verapamilia

D. Digoxinia √

E. Bisoprololia

Digoxin is strongly indicated for coexistent atrial fibrillation and heart failure. Beta-blockers should not be introduced until any heart failure has been stabilised

386. Which one of the following causes of gastroenteritis has the longest incubation period?ia

A. Campylobacteria

B. Bacillus cereusia

C. Shigellaia

D. Giardiasisia √

E. Salmonellaia


387. A 79-year-old man presents to his GP with a history of lower back pain and right hip pain. Blood tests reveal the following: Calcium= 2.20 mmol/l Phosphate= 0.8 mmol/l ALP= 890 u/L

What is the most likely diagnosis?

A. Primary hyperparathyroidismia

B. Chronic renal failureia

C. Osteomalaciaia

D. Osteoporosisia

E. Paget's diseaseia √

388. A 65-year-old man with a history of ischaemic heart disease is admitted with chest pain. The 12-hour troponin T is negative. During admission his medications were altered to reduce the risk of cardiovascular disease and to treat previously undiagnosed type 2 diabetes mellitus. Shortly after discharge he presents to his GP complaining of diarrhoea. Which one of the following medications is most likely to be responsible?

A. Gliclazideia

B. Clopidogrelia

C. Rosiglitazoneia

D. Metforminia √

E. Atorvastatinia

389. Which one of the following medications used in the management of hyperlipidaemia is most likely to cause flushing?ia

A. Bezafibrateia

B. Ezetimibeia

C. Nicotinic acidia √

D. Atorvastatinia

E. Cholestyramineia

390. Which one of the following features is least recognised in long-term lithium use?

A. Alopeciaia √

B. Weight gainia

C. Fine tremoria

D. Goitreia

E. Diarrhoeaia

Theme: Asthma: stepwise management in adultsia



A. Add inhaled steroid at 400 mcg/day

B. Increase inhaled steroid to 800 mcg/day

C. Increase inhaled steroid to 2000 mcg/day

D. Course of prednisolone for 5 days

E. Double inhaled steroids until symptoms resolve

F. Add inhaled long-acting B2 agonist

G. Trial of leukotriene receptor antagonist

H. Refer to specialist

I. Admit to hospital



For each of the following scenarios select the most appropriate action:

391. A 30-year-old man who has asthma presents with a 5 day history of cough and wheeze. He currently takes salbutamol prn and beclometasone 200mcg bd. His peak flow is 70% of normal. Course of prednisolone for 5 days

392. A 45-year-old man with a five year history of asthma comes for review. He currently uses salbutamol prn and regular beclometasone + salmeterol. He mentions that he never gets symptoms at the weekend or on holiday. Refer to specialist

393. A 23-year-old woman comes for review. Despite using beclometasone 200mcg bd she is regularly having to use her salbutamol inhaler. Her inhaler technique is good. Add inhaled long-acting B2 agonist

394. A 54-year-old man with type 2 diabetes mellitus is started on exenatide. Which one of the following statements regarding exenatide is incorrect?

A. Typically results in weight lossia

B. May be combined with a sulfonylureaia

C. The major adverse effect is flu-like symptomsia √

D. May be combined with a thiazolidinedioneia

E. Must be given by subcutaneous injectionia

Exenatide causes vomiting The major adverse effect is nausea and vomiting

395. A 27-year-old female presents with alternating loose and hard stools associated with abdominal discomfort and bloating. Which one of the following is it most important to do before making a positive diagnosis of irritable bowel syndrome?i

A. Arrange ultrasound abdomenia

B. Flexible sigmoidoscopyia

C. Ask about family history of ovarian canceria √

D. Use a standardised screening tool for depressionia

E. Perform thyroid function testsia

396. A 34-year-old man is reviewed in the neurology clinic. He has been established on sodium valproate for primary generalised epilepsy. Despite now taking a therapeutic dose he continues to have seizures and is troubled by weight gain since starting sodium valproate. He asks to stop the his current medication and try a different drug. Which one of the following drugs would be the most appropriate second-line treatment?

A. Lamotrigineia √

B. Ethosuximideia

C. Pregabalinia

D. Gabapentinia

E. Tiagabineia

397. A 62-year-old HGV driver is reviewed. He was diagnosed last year with type 2 diabetes mellitus. Following weight loss and metformin his HbA1c has decreased from 8.9% to 8.4%. What is the most suitable next step in management?ia

A. Add exenatideia

B. Make no changes to managementia

C. Add gliclazideia

D. Stop metformin for a period to ensure hypoglycaemic awareness is not lostia

E. Add pioglitazoneia √

398. A 72-year-old man is started on amlodipine 5mg OD for hypertension. He has no other past medical history of note and routine bloods (including fasting glucose) and ECG were normal. What should his target blood pressure be once on treatment?ia

A. 130/80 mmHgia

B. 140/80 mmHgia

C. 140/85 mmHgia

D. 140/90 mmHgia √

E. 150/90 mmHgia

399. Which one of the following drugs should be prescribed using the proprietary, rather than generic, nameia



A. Amiodaroneia

B. Carbamazepineia

C. Short-acting beta 2 agonistsia

D. Modified release calcium channel blockersia √

E. Sodium valproateia



400. A 25-year-old female currently under investigation for secondary amenorrhoea presents with jaundiced sclera. On examination spider naevi are present along with tender hepatomegaly. Blood tests show: Hb 11.6 g/dl Plt 145 * 109/l WCC 6.4 * 109/l Albumin 33 g/l Bilirubin 78 µmol/l ALT 245 iu/l

What is the most likely diagnosis?

A. Haemochromatosisia

B. Wilson's diseaseia

C. Primary biliary cirrhosisia

D. Autoimmune hepatitisia √

E. Primary sclerosing cholangitisia

401. A 76-year-old woman is diagnosed with Alzheimer's disease. Which one of the following could be a contraindication to the prescription of donepezil?ia

A. History of depressionia

B. Sick sinus syndromeia √

C. Concurrent simvastatin therapyia

D. Concurrent citalopram therapyia

E. Ischaemic heart diseaseia

402. A 22-year-old man consults you as he and his housemate have been feeling generally unwell for the past few weeks. Which one of the following is the most common symptom seen in carbon monoxide poisoning?

A. Hyperpyrexiaia

B. Nauseaia

C. Cherry red skinia

D. Confusionia

E. Headachei √





Theme: Antibiotic guidelines

A. Cefixime

B. Trimethoprim or nitrofurantoin or amoxicillin or cephalosporin

C. Quinolone or trimethoprim

D. Doxycycline + metronidazole + ceftriaxone

E. Doxycycline + amoxicillin + ciprofloxacin

F. Phenoxymethylpenicillin + flucloxacillin

G. Flucloxacillin

H. Amoxicillin or doxycycline or erythromycin

I. Doxycycline

J. Broad-spectrum cephalosporin or trimethoprim

For each one of the following please select the most complete answer from the options above

403. Gonorrhoea Cefixime

404. Extensive otitis externa Flucloxacillin

405. Pelvic inflammatory disease Doxycycline + metronidazole + ceftriaxone

406. A 68-year-old woman presents with a two month history of electric shock like pains on the right side of her face. She describes having around 10-20 episodes a day which, each lasting for around 30-60 seconds. A recent dental check was normal. Neurological examination is unremarkable. What is the most suitable first-line management?

A. Amitriptyline

B. Sodium valproate

C. Carbamazepine √

D. Atenolol

E. Zolmitriptan

407. A 54-year-old female presents with a 3 month history of dysphagia affecting both food and liquids from the start, along with symptoms of heartburn. What is the most likely underlying diagnosis?

A. Pharyngeal pouch

B. Gastric adenocarcinoma

C. Benign stricture

D. Oesophageal cancer √

E. Achalasia

408. A 52-year-old woman who was diagnosed as having primary atrophic hypothyroidism 12 months ago is reviewed following recents thyroid function tests (TFTs): TSH= 12.5 mU/l Free T4=14 pmol/l

She is currently taking 75mcg of levothyroxine once a day. How should these results be interpreted?

A. Poor compliance with medication √

B. Taking extra thyroxine

C. Evidence of recent systemic steroid therapy

D. Keep on same dose

E. T4 to T3 conversion disorder

409. A 61-year-old woman attends the surgery as she reports being generally unwell with muscle twitching. Blood pressure is recorded at 114/78 mmHg, pulse 84/min and she is apyrexial. You arrange some blood tests which reveal the following: Calcium= 1.94 mmol/l Albumin= 38 g/l

Which one of the following tests is most useful in elucidating the cause of her symptoms?

A. Urea

B. Vitamin D

C. Phosphate

D. Parathyroid hormone √

E. Magnesium

410. Which one of the following statements regarding metformin is true?

A. Should be stopped in a patient admitted with a myocardial infarction √

B. Hypoglycaemia is a recognised adverse effect

C. May cause a metabolic alkalosis

D. May aggravate necrobiosis lipoidica diabeticorum

E. Increases vitamin B12 absorption

411. A 23-year-old female is commenced on varenicline to help her stop smoking. Which one of the following adverse effects is most likely to occur?

A. Vivid dreams

B. Nausea √

C. Constipation

D. Insomnia

E. Drug-induced lupus





412. Which one of the following is least recognised as an adverse effect of taking bendroflumethiazide?

A. Photosensitivity rash

B. Agranulocytosis

C. Hypokalaemia

D. Pancreatitis

E. Hirsuitism √

413. A 45-year-old man with a persistent chest infection presents to surgery for review. He has now had two courses of antibiotics with no real improvement. A decision is made to perform a chest x-ray and do screening blood tests. When interpreting the blood tests which one of the following would not normally be raised in response to an acute infection?

A. CRP

B. Platelets

C. ESR

D. Albumin √

E. Ferritin

414. An elderly man who has metastatic prostate cancer is reviewed. His pain is currently controlled with MST 30mg bd but he is now unable to take medication by mouth and has become drowsy. A syringe driver is set-up. What is the most appropriate prescription?

A. Diamorphine 15mg over 24 hours in 'water for injection'

B. Diamorphine 20mg over 24 hours in sodium chloride 0.9%

C. Diamorphine 20mg over 24 hours in 'water for injection' √

D. Diamorphine 30mg over 24 hours in 'water for injection'

E. Diamorphine 30mg over 24 hours in sodium chloride 0.9%

Theme: Diabetes mellitus: diagnosis

A. Normal

B. Diabetes mellitus

C. Impaired fasting glucose

D. Samples mixed up

E. Impaired glucose tolerance

F. Suggests diabetes mellitus but further testing needed

G. Impaired fasting glucose and impaired glucose tolerance

Please select the diagnosis for each of the following scenarios:

415. After fasting overnight a patients urine sample shows glucose ++, no ketones

416. A patient who presents with polydipsia has a non-fasting glucose sample taken which is reported as 11.4 mmol/l

417. An oral glucose tolerance test is performed. Fasting sample = 6.4 mmol/l, 2-hour sample = 8.4 mmol/l

Which one of the following unwanted effects is most likely to occur in patients taking gliclazide?

A. Peripheral neuropathy

B. Cholestasis

C. Photosensitivity

D. Syndrome of inappropriate ADH secretion

E. Increased appetite and weight gain √

418. Which one of the following statements regarding smoking cessation is incorrect?

A. Clinicians should not favour one method over another

B. A repeat treatment course should not normally be offered within 6 months

C. Nicotine replacement therapy and varenicline may be combined if patients have failed monotherapy √

D. Varenicline should be started 1 week before stopping smoking

E. Flu-like symptoms is a recognised adverse effect of nicotine replacement therapy

419. A 71-year-old man with chronic kidney disease stage 3 is reviewed. He is known to have hypertension and ischaemic heart disease but a recent fasting glucose result confirmed he is not diabetic. A recent early morning urine result is reported as follows: Albumin:creatinine ratio = 5.2 mg/mmol What is the most appropriate action?

A. Refer to a nephrologist

B. No action as not clinically significant √

C. Obtain a 24-hour urine collection

D. Repeat using a late-evening sample

E. Arrange renovascular imaging





420. Which one of the following statements regarding raloxifene in the management of osteoporosis is incorrect?

A. Has been shown to prevent bone loss and to reduce the risk of vertebral fractures

B. Is a selective oestrogen receptor modulator

C. May worsen menopausal symptoms

D. Increases risk of thromboembolic events

E. Increases the risk of breast cancer √

421. Which one of the following adverse effects is most likely to be seen in patients taking ciclosporin?

A. Hypertension √

B. Hypokalaemia

C. Alopecia

D. Diabetes insipidus

E. Atrophy of the gums

422. A 26-year-old female is commenced on carbamazepine for complex partial seizures. She has no previous medical history of note and consumes a moderate amount of alcohol. Three months later she is admitted due to series of seizures and carbamazepine levels are noted to be subtherapeutic. A pill-count reveals the patient is fully compliant. What is the most likely explanation?

A. Auto-inhibition of liver enzymes

B. Prescription of omeprazole

C. Prescription of fluoxetine

D. Auto-induction of liver enzymes √

E. Alcohol binge

423. Which one of the following side-effects is least recognised in patients taking ciclosporin?

A. Hypokalaemia √

B. Hyperplasia of the gum

C. Hypertension

D. Tremor

E. Excessive hair growth

424. A 34-year-old accountant presents with a one week history of pain around his right eye occurring once or twice a day. They are described as being very severe and lasting between 10-30 minutes each. He also describes a feeling of a blocked nose. What is the treatment of choice to treat this current episode?

A. Ibuprofen

B. Acetazolamide + topical pilocarpine

C. Prednisolone

D. Subcutaneous sumatriptan √

E. Ergotamine

425. Which one of the following dopamine receptor agonists used in the management of Parkinson's disease is least associated with pulmonary, retroperitoneal and pericardial fibrosis?

A. Pergolide

B. Lisuride

C. Bromocriptine

D. Cabergoline

E. Ropinirole √

Theme: Side-effects of common antibiotics



A. Suppression of haemopoiesis

B. Headaches

C. Gastrointestinal upset

D. Reaction following alcohol ingestion

E. Rash with infectious mononucleosis

F. Flushing

G. Cholestasis

H. Reduced seizure threshold



Select the side-effect most characteristically associated with the following antibiotics:

426. Flucloxacillin Chilestasis,

427. Erythromycin Gastrointestinal upset, QT prolongation

428. Ciprofloxacin Reduced seizure threshold

429. Metronidazol Reaction following alcohol ingestion

430. Chloramphenicol Suppression of haemopoiesis

431. Clarythromycin Gastrointestinal upset, QT prolongation

432. Trimethoprim Flushing, Pruritis, Hemopoesis suppression

433. Amoxil Rash with infectious mononucleosis

434. Which one of the following would not be considered a normal variant on the ECG of an athletic 28-year-old man?

A. Wenckebach phenomenon

B. Sinus Bradycardia

C. Junctional rhythm

D. First degree heart block

E. LBBB √

435. Which one of the following side-effects is least recognised in patients taking isotretinoin?

A. Hypertension √

B. Teratogenicity

C. Nose bleeds

D. Depression

E. Raised triglycerides

436. What is the most appropriate dose of adrenaline to give during a cardiac arrest?

A. 1ml 1:100,000 IV

B. 10ml 1:1,000 IV

C. 0.5ml 1:1,000 IM

D. 1ml 1:10,000 IV

E. 10ml 1:10,000 IV √

437. You are reviewing the provision of cancer care by the practice. What is the most common type of cancer in the UK, excluding non-melanoma skin cancers?

A. Prostate

B. Colorectal

C. Breast √

D. Lung

E. Malignant melanoma

438. A 55-year-old woman with type 2 diabetes mellitus is reviewed. A decision is made to start thiazolidinedione therapy. Which one of the following points is it most relevant to consider before starting treatment?

A. History of oesophageal problems

B. Fracture risk √

C. History of depression

D. History of cardiac arrhythmias

E. Visual acuity

439. A 46-year-old man presents to his GP as he is concerned about reduced libido and erectile dysfunction. His wife also reports has 'no energy' and comments that he has a 'permanent suntan'. During the review of systems he also complains of pains in both hands. Which one of the following investigations is most likely to reveal the diagnosis?

A. Ferritin √

B. Testosterone

C. Cortisol

D. Blood glucose

E. Prolactin

440. A 34-year-old woman with a history of antiphospholipid syndrome presents with a swollen and painful leg. Doppler ultrasound confirms a deep vein thrombosis (DVT). She had a previous DVT 4 months ago and was taking warfarin (with a target INR of 2-3) when the DVT occurred. How should her anticoagulation be managed?

A. Life-long warfarin, increase target INR to 3 – 4 √

B. Add in life-long low-dose aspirin

C. A further 6 months warfarin, target INR 2 – 3

D. A further 6 months warfarin, target INR 3 – 4

E. Life-long warfarin, target INR 2 – 3

441. A 45-year-old man is noted to have non-tender, smooth hepatomegaly associated Dupuytren's contracture and parotid enlargement. He recently returned from a holiday in Thailand. What is the likely diagnosis?

A. Primary hepatoma

B. Hydatid disease

C. Alcoholic liver disease √

D. Viral hepatitis

E. Tricuspid regurgitation

442. A 54-year-old female is receiving a course of chemotherapy for breast cancer. She is experiencing troublesome vomiting which has not been helped by domperidone. What is the most appropriate next management step?

A. Add an antihistamine

B. Add a 5HT2 antagonist

C. Add a phenothiazine

D. Add a dopamine receptor antagonist

E. Add a 5HT3 antagonist √





Theme: Palliative care prescribing: converting opioids



A. 10 mg / day

B. 20 mg / day

C. 30 mg / day

D. 40 mg / day

E. 50 mg / day

F. 60 mg / day

G. 70 mg / day

H. 80 mg / day

I. 100 mg / day



For each of the following select the equivalent daily dose of oral morphine:

443. Codeine 200mg / day 20 mg / day

444. Tramadol 400 mg / day 80 mg / day

445. Oral oxycodone 30 mg / day 20 mg / day

446. A 33-year-old known to be HIV positive presents with a 2 day history of diarrhoea. What is the most likely cause of his diarrhoea?

A. Herpes simplex enteritis

B. Cryptosporidium √

C. Histoplasmosis

D. Shigella

E. Mycobacterium avium intracellulare

447. Which of the following findings is not typical in a patient with antiphospholipid syndrome?

A. Prolonged APTT

B. Thrombocytosis √

C. Recurrent venous thrombosis

D. Recurrent arterial thrombosis

E. Livedo reticularis

448. A 45-year-old female with multiple sclerosis complains of tingling in her hands which comes on when she flexes her neck. What is this an example of?

A. Werdnig-Hoffman's sign

B. Lhermitte's sign √

C. Oppenheim's sign

D. Lambert's sign

E. Uhthoff's phenomenon

449. A 67-year-old man who has a history of type 2 diabetes mellitus and benign prostatic hypertrophy presents with burning pain in his feet. This has been present for the past few months and is getting gradually worse. He has tried taking paracetamol and ibuprofen at night but unfortunately has received no benefit. Clinical examination is unremarkable other than diminished sensation to fine touch on both soles. What is the most suitable initial management?

A. Carbamazepine

B. Amitriptyline

C. Gabapentin √

D. Fluoxetine

E. Referral to the pain management clinic

450. A 68-year-old woman presents with lethargy and generalised aches. As part of a blood screen the following results are obtained: Calcium= 2.83 mmol/l Albumin= 42 g/l ESR= 26 mm/hr

What is the most likely cause of these blood results?

A. Multiple myeloma

B. Sarcoidosis

C. Normal

D. Breast cancer metastases

E. Primary hyperparathyroidism √

451. Which one of the following patients would it be most suitable to offer a screening test for coeliac disease to?

A. A patient who is 'tired all the time' √

B. A patient with rheumatoid arthritis

C. A patient who has a family history of inflammatory bowel disease

D. A patient with type 2 diabetes mellitus

E. A patient who develops erythema nodosum

452. A 55-year-old man who has a history of ischaemic heart disease presents with myalgia. His long-term medications include aspirin, simvastatin and atenolol. Given his statin use a creatine kinase is measured and reported as follows: Creatine kinase 1,420 u/l (< 190 u/l) His problems seem to have followed the prescription of a new medication. Which one of the following is most likely to have caused the elevation in creatine kinase?



A. Rifampicin

B. Felodipine

C. Clarithromycin √

D. Isosorbide mononitrate

E. Amitriptyline



453. A 56-year-old man with a history of epilepsy, atrial fibrillation and ischaemic heart disease is noted to have a rash on his forearms and face. Which one of the following drugs is most likely to be responsible?

A. Verapamil

B. Carbamazepine

C. Amiodarone √

D. Digoxin

E. Clopidogrel

454. Which one of the following investigations is essential prior to starting anti-tuberculosis therapy?

A. Liver functions tests √

B. Urine for acid-fast bacilli

C. Vitamin B6 level

D. Blood glucose

E. Full blood count

Theme: Side-effects of anti-anginalsia



A. Anal ulceration

B. Reduced seizure threshold

C. Hyponatraemia

D. Thrombocytopaenia

E. Constipation

F. Drug-induced lupus

G. Tachycardia

H. QT interval prolongation

I. Sleep disturbances



For each one of the following drugs choose the side-effect which it is characteristically associated with

455. Verapamil Constipation

456. Atenolol Sleep disturbances

457. Isosorbide mononitrate Tachycardia

458. A 45-year-old man is reviewed in the diabetes clinic. The following results are obtained:

Urinalysis= NAD HbA1c=8.6% Gliclazide is added to the metformin he already takes. What is the minimum time period after which the HbA1c should be repeated ?

A. 6 months

B. 1 month

C. 2 weeks

D. 3 months √

E. 4 months

459. Which of the following is not known to cause acute pancreatitis?

A. Hypocalcaemia √

B. Hypothermia

C. Mumps

D. Hypertriglyceridaem

E. Steroids

460. Each of the following drugs are known to inhibit cytochrome P450, except:

A. Ketoconazole

B. Ciprofloxacin

C. Erythromycin

D. Clopidogrel √

E. Amiodarone

461. A 17-year-old man attends the local sexual health clinic. He has developed a large, keratinised genital wart on the shaft of his penis. This has been present for around three months but he has been too embarrassed to present before now. What is the most appropriate initial management?

A. Topical aciclovir

B. Cryotherapy √

C. Topical salicylic acid

D. Electrocautery

E. Topical podophyllum

462. A prison GP is bitten by a patient who is known to have hepatitis B. The GP has a documented full history of hepatitis B vaccination and was known to be a responder. What is the most appropriate action to reduce the chance of contracting hepatitis B?

A. Admit for intravenous interferon

B. Give hepatitis B immune globulin

C. Give hepatitis B immune globulin + hepatitis B vaccine booster √

D. Give hepatitis B vaccine booster

E. Give oral ribavirin for 4 weeks





463. A 58-year-old man who is taking lithium for bipolar disorder presents for review. During routine examination he found to be hypertensive with a blood pressure of 166/82 mmHg. This is confirmed with two separate readings. Urine dipstick is negative and renal function is normal. What is the most appropriate medication to start?

A. Amlodipine √

B. Ramipril

C. Losartan

D. Bendroflumethiazide

E. Doxazosin

464. Which one of the following may be associated with an increased risk of venous thromboembolism?

A. Reiter's syndrome

B. Addison's disease

C. Hypothyroidism

D. Behcet's syndrome √

E. Penicillin

465. Which of the following cytotoxic agents is most associated with lung fibrosis?

A. Doxorubicin

B. Bleomycin √

C. Cisplatin

D. Cyclophosphamide

E. Vincristine

466. A 26-year-old man returns to the genito-urinary medicine clinic. He is a known intravenous drug user. Five days ago he was seen with a urethral discharge. A swab taken in the clinic showed a Gram-negative diplococcus and treatment with IM ceftriaxone was given. Unfortunately his symptoms have not resolved. What is the most likely explanation?

A. Gonorrhoea-resistant to ceftriaxone

B. Co-existent Candida infection

C. HIV infection

D. Co-existent syphilis infection

E. Co-existent Chlamydia infection √

Theme: Diarrhoea



A. Gastroenteritis

B. Crohn's disease

C. Ulcerative colitis

D. Colorectal cancer

E. Laxative abuse

F. Constipation causing overflow

G. Lactose intolerance

H. Diverticulitis

I. Irritable bowel syndrome

J. Coeliac disease



For each one of the following scenarios please select the most likely diagnosis:

467. A 41-year-old man with cerebral palsy is admitted with abdominal pain and diarrhoea. His carers report him passing 5-6 watery stools per day for the past four days. On examination he has a mass in the left side of the abdomen. Constipation causing overflow

468. A 37-year-old woman presents with a three week history of diarrhoea and crampy abdominal pains. On examination she is noted to have a number of perianal skin tags. Crohn's disease

469. A 4-year-old boy is investigated for chronic diarrhoea, abdominal bloating and failure to thrive. Coeliac disease

470. A 65-year-old man with a history of type 2 diabetes mellitus and ischaemic heart disease presents with erectile dysfunction. It is decided to try sildenafil therapy. Which one of the following existing medications may be continued without qualification?

A. GTN spray

B. Nicorandil

C. Nateglinide √

D. Doxazosin

E. Isosorbide mononitrate

471. A 30-year-old man presents to his GP enquiring about screening for haemochromatosis as his brother was diagnosed with the condition 2 years ago. The patient is currently well with no features suggestive of haemochromatosis. What is the most appropriate investigation?

A. Serum total iron-binding capacity

B. HFE gene analysis √

C. Serum transferrin saturation

D. Serum ferritin

E. Serum iron





472. Which one of the following is not associated with oesophageal cancer?

A. Achalasia

B. Smoking

C. Gastro-oesophageal reflux disease

D. Helicobacter pylori √

E. Alcohol

473. A patient is given ondansetron for chemotherapy related nausea. What is the most likely side-effect?

A. Constipation √

B. Dry mouth

C. Insomnia

D. Visual disturbance

E. Pruritus

474. A 14-year-old girl is taken to the Emergency Department, after being found lying on her bed next to an empty bottle of pills prescribed for her mother. On examination she is agitated, has a clenched jaw and her eyes are deviated upwards. Which drug is she most likely to have consumed?

A. Phenytoin

B. Metoclopramide √

C. Amitriptyline

D. Carbamazepine

E. Nifedipine

475. Which one of the following is a notifiable disease in the UK?

A. Listeria

B. Creutzfeldt–Jakob disease

C. Leptospirosis √

D. HIV

E. Mycoplasma pneumonia

476. A 52-year-old woman with suspected diabetes mellitus has an oral glucose tolerance test, following the standard WHO protocol. The following results are obtained:

Time (hours) Blood glucose (mmol/l)

0 5.9

2 8.4

How should these results be interpreted?

A. Impaired fasting glucose and impaired glucose tolerance

B. Normal

C. Diabetes mellitus

D. Impaired glucose tolerance √

E. Impaired fasting glucose

477. A 49-year-old presents with hot flushes and irregular periods. She has a past history of migraine and is keen for hormone replacement therapy (HRT). What is the most appropriate management?

A. HRT is contraindicated

B. HRT can be given but may worsen migraine √

C. HRT can only be given if the women has not a migraine in the past 12 months

D. Tell the patient HRT does not affect migraine

E. Suggest a progestogen only formula

478. A 42-year-old female is noted to have a Hb of 17.8 g/dL. Which one of the following is least likely to be the cause?

A. Polycythaemia rubra vera

B. Chronic obstructive pulmonary disease

C. Hypernephroma

D. Haemochromatosis √

E. Dehydration

479. A 36-year-old woman presents with feeling tired and cold all the time. On examination a firm, non-tender goitre is noted. Blood tests reveal the following: TSH= 34.2 mU/l Free T4= 5.4 pmol/l What is the most likely diagnosis?

A. Primary atrophic hypothyroidism

B. Pituitary failure

C. De Quervain's thyroiditis

D. Iodine deficiency

E. Hashimoto's thyroiditis √





480. Which one of the following statements regarding metformin is false?

A. Does not cause hypoglycaemia

B. Increases insulin sensitivity

C. Decreases hepatic gluconeogenesis

D. Increases endogenous insulin secretion Х

E. Reduces GI absorption of carbohydrates

481. A 54-year-old patient takes hydrocortisone 20mg in the mornings and 5mg at night for Addison's disease. The endocrinology consultant would like her to take prednisolone instead. What dose of prednisolone should be started?

A. 5 mg

B. 7 mg √

C. 8 mg

D. 10 mg

E. 12.5 mg

482. A 31-year-old woman presents with a 4 month history of headache. She has brought a headache diary which demonstrates that her symptoms are present on around 20-25 days of each month. The headache is typically unilateral and she is currently taking paracetamol 1g qds and ibuprofen 400mg tds everyday to try and relieve her symptoms. A diagnosis of medication overuse headache is suspected. What is the most appropriate management?

A. Add metoclopramide + start propranolol

B. Gradually withdraw analgesics + start propranolol

C. Abruptly stop analgesics √

D. Gradually withdraw analgesics

E. Continue analgesics + start propranolol

483. A 62-year-old man is reviewed the day after a successful elective DC cardioversion for atrial fibrillation. Six weeks ago he presented in fast atrial fibrillation. A decision was made at the time to warfarinise him for six weeks after which he was to be cardioverted. During this time he had a normal transthoracic echocardiogram. He has no past medical history of note other than treatment for a basal cell carcinoma. What is the most appropriate plan regarding anticoagulation?....

A. Can stop immediately

B. Continue warfarinisation for 1 week then review following

C. Lifelong warfarin

D. Lifelong aspirin

E. Continue warfarinisation for 4 weeks then review following √

484. A 28-year-old man who has recently emigrated from Nigeria presents with a penile ulcer. It initially started as a papule which later progressed to become a painful ulcer 15mm in diameter with an undermined ragged edge. Examination of the testes was unremarkable but tender inguinal lymphadenopathy was noted. What is the most likely diagnosis?

A. Chancroid

B. Lymphogranuloma venereum √

C. Syphilis

D. Herpes simplex infection

E. Granuloma inguinale

485. A 54-year-old female with rheumatoid arthritis is noted to have proteinuria on annual review. Which one of the following drugs is most associated with the development of proteinuria?

A. Ciclosporin

B. Gold √

C. Methotrexate

D. Infliximab

E. Sulfasalazine

486. A 69-year-old man with a history of ischaemic heart disease and type 2 diabetes mellitus is seen in the diabetes clinic. Which one of the following medications is it most important to avoid?

A. Nateglinide

B. Metformin

C. Pioglitazone

D. Glimepiride

E. Rosiglitazone √





487. Which one of the following statements regarding the assessment of proteinuria in patients with chronic kidney disease is incorrect?

A. Albumin:creatinine ratio (ACR) is more sensitive than protein:creatinine ratio (PCR)

B. An ACR of 30 mg/mmol is approximately equal to a PCR of 50 mg/mmol

C. An ACR sample is collected over 24 hours √

D. Women typically have higher ACR values

E. An ACR of 3.1 mg/mmol in a diabetic man is clinically significant

Theme: Respiratory tract infections: NICE guidelines



A. 2 days

B. 4 days

C. 7 days

D. 10 days

E. 2 Weeks

F. 2 ½ Weeks

G. 3 Weeks

H. 4 Weeks



For each one of the following respiratory tract infections select the average total illness length:

488. Acute tonsillitis 7 days

489. Acute rhinosinusitis 2 ½weeks

490. Acute bronchitis 3 Weeks

491. A 33-year-old woman is prescribed varenicline to help her quit smoking. What is the mechanism of action of varenicline?

A. Norepinephrine and dopamine reuptake inhibitor, and nicotinic antagonist

B. Dopamine agonist

C. Dopamine antagonist

D. Selective serotonin reuptake inhibitor

E. Nicotinic receptor partial agonist √

492. You are giving dietary advice to an obese patient who has been diagnosed with type 2 diabetes mellitus. Following recent NICE guidelines, which one of the following should not be encouraged?

A. Food products specifically targeted at diabetics √

B. Initial weight loss of 5-10%

C. Limited substitution of sucrose-containing foods for other carbohydrates

D. High-fibre, low glycaemic index carbohydrates

E. Low-fat dairy products

493. A 54-year-old man is diagnosed with type 2 diabetes mellitus. A decision is made to start simvastatin 40mg. What is the ideal time to advise patients to take this medication?

A. After breakfast

B. Last thing in the evening √

C. After evening meal

D. Just before evening meal

E. First thing in the morning

494. A 60-year-old man who is currently receiving chemotherapy for non-small cell lung cancer presents for review. He is currently being treated with oral calcium supplements as hypocalcaemia was detected during a recent admission. Bloods taken two days ago reveal the following: Calcium 2.01 mmol/l Which one of the following tests may help determine why his calcium level remains low despite calcium supplementation?

A. Vitamin D

B. Parathyroid hormone

C. Phosphate

D. Alkaline phosphatase

E. Magnesium √

495. A 71-year-old woman who takes warfarin for atrial fibrillation presents with bruising on her hands and arms. A blood test is arranged:

Hb 14.7 g/dl Plt 198 * 109/l WBC 5.3 * 109/l INR= 7.1

What is the most appropriate management?

A. Stop warfarin + restart when INR < 3.0

B. Stop warfarin + restart when INR < 5.0 + give low-molecular weight heparin until warfarin restarted

C. Oral vitamin K 5mg + stop warfarin + restart when INR < 3.0

D. Stop warfarin + restart when INR < 5.0 √

E. Admit

496. A patient who was an intravenous drug user in the 1990s asks for a hepatitis C test. What is the most appropriate action?

A. Refer him for pre-test counselling to discuss the pros and cons of testing √

B. Advise him that no accurate test is currently available but that he should undertake normal precautions

C. Arrange an anti-HCV antibody test

D. Arrrange a HCV RNA test

E. Refer him to gastroenterology for a liver biopsy

497. Which one of the following is not part of the American College of Rheumatology criteria for diagnosing rheumatoid arthritis?

A. Raised ESR or CRP √

B. Morning stiffness > 1 hr

C. Subcutaneous nodules

D. Symmetrical arthritis

E. Rheumatoid factor positive

498. An elderly man with terminal prostate cancer decides to die at home. A syringe driver is set-up by the district nurses. For which one of the following medications is it least suitable to use water for injection as the diluent?

A. Haloperidol

B. Levomepromazine

C. Ondansetron √

D. Metoclopramide

E. Cyclizine

499. Each one of the following is a known cause of occupational asthma, except:

A. Isocyanates

B. Cadmium √

C. Soldering flux resin

D. Flour

E. Platinum salts

Theme: Causes of headache



A. Temporal arteritis

B. Sinusitis

C. Tension headache

D. Cluster headache

E. Acute glaucoma

F. Migraine

G. Subarachnoid haemorrhage

H. Raised intracranial pressure

I. Medication overuse headache

J. Meningitis



For each one of the following clinical scenarios select the most likely diagnosis:

500. A 22-year-old man presents with a one day history of a generalised headache. He prefers being in the dark and says he is 'sleepy'. He has no neck stiffness. His temperature is 37.9ºC Meningitis

501. A 69-year-old woman presents with a 3 week history of a headache which is worse on the right side. She is generally unwell and feels 'weak' Temporal arteritis

502. A 33-year-old woman presents due to a severe frontal headache. She developed a cold around 2 weeks ago but has now been left with a severe headache. The pain is worse when she bends forward Sinusitis

503. A 40-year-old man is investigated for increasing shortness of breath. He has smoked for the past 25 years. Pulmonary function tests are performed and are reported as follows:

FEV1= 1.4 L FVC= 1.7 L FEV1/FVC= 82% (normal > 75%)

Which one of the following disorders is most consistent with these results?

A. Asthma

B. Bronchiectasis

C. Neuromuscular disorder √

D. Chronic obstructive pulmonary disease

E. Laryngeal malignancy

504. A 23-year-old female with a history of diarrhoea and weight loss has a colonoscopy to investigate her symptoms. A biopsy is taken and reported as follows:

Pigment laden macrophages suggestive of melanosis coli

What is the most likely diagnosis?

A. Intestinal melanoma

B. Haemochromatosis

C. Ulcerative colitis

D. Laxative abuse √

E. Colorectal cancer

505. Which one of the following is not a feature of Peutz-Jeghers syndrome?

A. Intestinal obstruction

B. Pigmented lesions on palms

C. Malignant change in polyps

D. Osteomas √

E. Iron-deficiency anaemia





506. A 25-year-old man is counselled regarding the genetics of Huntington's disease. Which one of the following best describes the concept of anticipation?

A. The psychological effect of a patient knowing they will develop an incurable condition

B. Earlier age of onset in successive generations √

C. Less severe disease in successive generations

D. Where there is a known history of inherited conditions, patients may attribute symptoms to the onset of the disease

E. Screening at risk families to allow early intervention and improve outcomes

507. You review a 72-year-old man with metastatic bowel cancer who is in the terminal phase and has a syringe driver. Unfortunately he has developed intestinal obstruction and is suffering with bowel colic. What is the most appropriate drug to add to the syringe driver?

A. Metoclopramide

B. Hyoscine hydrobromide

C. Levomepromazine

D. Haloperidol

E. Hyoscine butylbromide √

508. A patient with chronic kidney disease stage 4 is started on lisinopril. Bloods are checked two weeks later. There have been no other changes to his medication and on examination the patient is volume replete. According to NICE, up to what increase in creatinine is acceptable following the introduction of an ACE inhibitor?

A. No increase

B. 5%

C. 10%

D. 15%

E. 30% √

509. A 54-year-old man has a routine medical for work. He is asymptomatic and clinical examination is unremarkable. Which of the following results establishes a diagnosis of impaired fasting glucose?

A. Fasting glucose 7.1 mmol/L on one occasion

B. Fasting glucose 6.8 mmol/L on two occasions √

C. Glycosuria ++

D. 75g oral glucose tolerance test 2 hour value of 8.4 mmol/L

E. HbA1c of 6.7%

510. An 18-year-old male presents to his GP concerned about delayed pubertal development, despite being 1.77m tall. On examination he has scant pubic hair and reduced testicular volume. The following blood results are obtained:

Testosterone 6.7 nmol/l (9 - 30) LH 3 .1 mu/l (3 - 10) FSH 5.7 mu/l (3 - 10)

What is the most likely diagnosis?

A. Klinefelter's syndrome

B. Acute lymphoblastic leukaemia

C. Testicular feminisation syndrome

D. Primary testicular failure

E. Kallman's syndrome √

511. A 66-year-old comes for review. He had a prosthetic aortic valve replacement five years for which he is warfarinised. Over the past three months he has been complaining of fatigue and a full blood count was requested: Hb= 10.3 g/dl MCV 68 fl Plt= 356 * 109/l WBC= 5.2 * 109/l

Blood film= Hypochromia INR= 3.0

An upper GI endoscopy was reported as normal. What is the most appropriate next investigation?

A. Transthoracic echocardiogram

B. Colonoscopy √

C. Three sets of blood cultures

D. Transoesophageal echocardiogram

E. Reticulocyte count

Theme: Alternative medicine



A. Common cold

B. Benign prostatic hyperplasia

C. Osteoarthritis

D. Irritable bowel syndrome

E. Reduction in menopausal symptoms

F. Depression

G. Migraine

H. Reduction in cholesterol

I. Asthma

J. Cold sores



For each one of the following herbal medicines please select the condition it is most associated with:

512. Saw Palmetto Benign prostatic hyperplasia

513. Feverfew Migraine

514. Echinacea Common cold

515. A 34-year-old steelworker presents complaining of episodic shortness of breath. This is particularly noted whilst at work where he describes feeling wheezy and having a tendency to cough. A diagnosis of occupation asthma is suspected. Which one of the following is the most appropriate diagnostic investigation?

A. Patch testing

B. High resolution computed tomography of thorax

C. Serial peak flow measurements at work and at home √

D. Specific IgE measurements

E. Skin prick test

516. A 66-year-old female presents to her GP due to pain at the base of her left thumb. She has no past medical history of note. On examination there is diffuse tenderness and swelling of her left first carpometacarpal joint. What is the most likely diagnosis?

A. Osteoarthritis √

B. De Quervain's tenosynovitis

C. Gout

D. Rheumatoid arthritis

E. Primary hyperparathyroidism

517. A nurse who is known to have an allergy to latex develops a widespread urticarial rash and facial oedema shortly after eating lunch. Which food is she most likely to have consumed?

A. Orange

B. Apple

C. Grapes

D. Pear

E. Banana √

518. A 61-year-old female who has recently emigrated from the Indian subcontinent presents to her GP with muscle weakness. Bloods reveal a low serum calcium. A diagnosis of osteomalacia is suspected. Which one of the following, per average serving, provides the best source of vitamin D?

A. Lentils

B. Sunflower seeds

C. Salmon

D. Cod liver oil √

E. Milk

519. Each one of the following is associated with carbon monoxide poisoning, except:

A. Cardiac arrhythmias

B. Chorea

C. Confusion

D. Blue skin and mucosae √

E. Hyperpyrexia

520. A 57-year-old female presents after being discharged form the acute medical unit two weeks ago, following an admission with shortness of breath and pleuritic chest pain. During her admission a pulmonary embolism was diagnosed and warfarin commenced. She has no past medical history of note and enjoys good health. What is the recommended length of warfarinisation for this patient?

A. 6 weeks

B. 3 months √

C. 6 months

D. 12 months

E. Life-long

521. A 63-year-old female is reviewed in the rapid access transient ischaemic attack clinic. For the past three weeks she has been having episode of transient loss of vision in the right eye. Carotid ultrasound reveals a 50% stenosis of her right carotid artery and an ECG shows sinus rhythm. What is the most appropriate management of this patient?

A. Warfarin

B. Aspirin and clopidogrel

C. Carotid endarterectomy

D. Aspirin √

E. Aspirin and dipyridamole





522. A 69-year-old man who takes warfarin for atrial fibrillation asks for advice. He is due to have a tooth extraction at the dentist and is unsure what to do with regards to his 'blood-thinning' tablets. There is no other past medical history of note. The last INR was taken two weeks ago and reported as 2.2 with his target INR being 2.0-3.0. What is the most appropriate advice?

A. Switch to low-molecular weight heparin prior to extraction

B. Switch to aspirin prior to extraction

C. Check INR 72 hours before procedure, proceed if INR < 4.0 √

D. Check INR 72 hours before procedure, proceed if INR < 2.5

E. Should be performed at local hospital rather than community dentist

523. A 62-year-old man is commenced on finasteride for symptoms of bladder outflow obstruction. Which one of the following adverse effects is most associated with this treatment?

A. Alopecia

B. Gynaecomastia √

C. Prostate cancer

D. Increased levels of serum prostate specific antigen

E. Postural hypotension

Theme: Shortness of breath



A. Heart failure

B. Recurrent pulmonary emboli

C. Lung cancer

D. Obesity

E. Pulmonary fibrosis

F. Anaemia

G. Asthma

H. Chronic obstructive pulmonary disease

I. Bronchiectasis

J. Aortic stenosis



For each one of the following scenarios please select the most likely diagnosis:

524. A 42-year-old smoker presents with chronic cough and shortness of breath. He describes bringing up copious amounts of sputum each morning. Chest x-ray shows numerous parallel line shadows. Bronchiectasis

525. A 71-year-old man who is being investigated for recurrent collapse presents with progressive shortness of breath. His pulse is 84 / min and blood pressure 110/90 mmHg. Aortic stenosis

526. A 73-year-old man presents with shortness of breath. He smokes 20 / day but is otherwise well. Spirometry shows a restrictive picture. Pulmonary fibrosis

527. A 25-year-old man presents with bloating and alteration in his bowel habit. He has been keeping a food diary and feels his symptoms may be secondary to a food allergy. Blood tests show a normal full blood count, ESR and thyroid function tests. Anti-endomysial antibodies are negative. What is the most suitable test to investigate possible food allergy?

A. Urine chromatography

B. Hair analysis

C. Radioallergosorbent test (RAST)

D. Skin prick test √

E. Jejunal biopsy

528. A patient presents with gastrointestinal symptoms. Which one of the following features in the history would be least consistent with making a diagnosis of irritable bowel syndrome?

A. Past medical history of epilepsy

B. Symptoms made worse by eating

C. 62-year-old female

D. Passage of mucous with stool √

E. Bladder symptoms

529. An obese man presents as he is concerned about his risk of developing cardiovascular disease. Which one of the following sets of results would suggest a diagnosis of the metabolic syndrome using the Scottish Intercollegiate Guidelines Network (SIGN) criteria?i

A. Waist circumference = 98 cm; fasting glucose = 7.2 mmol/l; HDL = 1.2 mmol/l

B. Triglycerides = 2.0 mmol/l; HDL = 1.2 mmol/l; fasting glucose = 5.4 mmol/l

C. Blood pressure = 140/90 mmHg; waist circumference = 90 cm; HDL = 1.4 mmol/l

D. Waist circumference = 110 cm; fasting glucose = 5.8 mmol/l; HDL = 0.8 mmol/l √

E. LDL = 3.0 mmol/l; blood pressure = 130/80; fasting glucose = 6.4 mmol/l

530. A patient is noted to have an absent triceps reflex. Which nerve root does this correspond to?

A. C7-C8 √

B. C5-C6

C. C3-C4

D. C6-C7

E. C5-C7





531. A 76-year-old man is reviewed. He was recently admitted after being found to be in atrial fibrillation. This was his second episode of atrial fibrillation. He also takes ramipril for hypertension but has no other history of note. During admission he was warfarinised and discharged with planned follow-up in the cardiology clinic. However, on review today he is found to be in sinus rhythm. What should happen regarding anticoagulation?

A. Stop warfarin

B. Continue warfarin for 1 month

C. Stop warfarin + start aspirin

D. Continue lifelong warfarin √

E. Continue warfarin for 6 months

532. A 58-year-old man is investigated for a chronic cough and is found to have lung cancer. He enquires whether it may be work related. Which one of the following is most likely to increase his risk of developing lung cancer?

A. Isocyanates

B. Soldering flux resin

C. Passive smoking √

D. Coal dust

E. Polyvinyl chloride

533. What is the main mechanism of action of simvastatin?

A. Bile acid sequestrant

B. Decreases hepatic HDL synthesis

C. Inhibits lipoprotein lipase

D. Decreases hepatic cholesterol synthesis √

E. Agonists of PPAR-alpha

534. A 54-year-old female is being investigated for a macrocytic anaemia. Bloods test reveal a low vitamin B12 level. Which one of the following medications may be contributing to this?

A. Bendroflumethiazide

B. Digoxin

C. Amiodarone

D. Sodium valproate

E. Metformin √

535. A 75-year-old woman is brought to the surgery by her family. She has been getting more short-of-breath over the last 6 weeks and says her energy levels are low. An ECG on shows atrial fibrillation at a rate of 114 / min. Blood pressure is 128/80 mmHg and a chest x-ray is unremarkable. What is the appropriate drug to control the heart rate?



A. Felodipine

B. Amiodarone

C. Digoxin

D. Flecanide

E. Bisoprolol



Theme: Side-effects of diabetes mellitus drugs



A. Hypocalcaemia

B. Diarrhoea

C. Sinusitis

D. Worsening of heart failure

E. Headaches

F. Hypoglycaemia



Select the side-effect most characteristically associated with the following drugs:

536. Metformin Diarrhoea

537. Pioglitazone Worsening of heart failure

538. Gliclazide Hypoglycaemia

539. A 29-year-old man presents with a six day history of watery diarrhoea that developed one week after returning from India. He had travelled around northern India for two months. On examination he is apyrexial and his abdomen is soft and non-tender. What is the most likely causative organism?



A. Amoebiasis

B. Giardiasis √

C. Campylobacter

D. Shigella

E. Salmonella



540. A 38-year-old woman is noticed to be jaundiced. As part of a liver screen the following results are obtained: Anti-HBs Negative Anti-HBc Positive HBs antigen Positive IgM anti-HBcNegative Anti-HBs = Hepatitis B Surface Antibody; Anti-HBc = Hepatitis B Core Antibody; HBs antigen = Hepatitis B Surface Antigen

What is the patient's hepatitis B status?

A. Probable hepatitis D infection

B. Acute hepatitis B infectionia

C. Previous immunisation to hepatitis B

D. Chronic hepatitis B √

E. Previous hepatitis B infection, not a carrier

541. A 54-year-old man with type 2 diabetes mellitus is reviewed in clinic. He is currently taking rosiglitazone, aspirin and simvastatin. Which one of the following problems is most likely caused by rosiglitazone?

A. Photosensitivity

B. Thrombocytopaenia

C. Myalgia

D. Peripheral oedema √

E. Hyponatraemia

542. Which of the following secondary causes of hyperlipidaemia result in predominantly hypercholesterolaemia?

A. Diabetes mellitus

B. Bendrofluazide

C. Nephrotic syndrome √

D. Alcohol

E. Obesity

543. A patient with metastatic cancer asks to be switched from MST 90 mg bd to fentanyl patches. What is the equivalent number of patches which should be applied?

A. Half a fentanyl '25' patch

B. One fentanyl '25' patch

C. One fentanyl '50' patch √

D. One fentanyl '75' patch

E. One fentanyl '100' patch

544. A 62-year-old man who had a mechanical mitral valve replacement four years ago is reviewed. What long term antithrombotic therapy is he likely to be taking?

A. Nothing

B. Warfarin

C. Aspirin

D. Aspirin + clopidogrel for the first 12 months

E. Warfarin + aspirin √

545. You are counselling a 26-year-old man who has recently had a positive HIV test. His most recent CD4 count is 650 Cells/mm3. Which one of the following vaccinations is contraindicated?

A. Oral poliomyelitis √

B. Yellow fever

C. Pneumococcus

D. Parenteral poliomyelitis

E. Measles, Mumps, Rubella

546. A 53-year-old female with a history of primary atrophic hypothyroidism is assessed two months following a change in her dose of levothyroxine. Which one of the following best describes what the TSH should ideally be?

A. Between 0.5 to 1.0 mU/l

B. Between 0.5 to 2.5 mU/l √

C. Between 2.5 to 4.5 mU/l

D. Between 1.5 to 3.5 mU/l

E. Between 3.5 to 5.5 mU/l

547. A 29-year-old man presents concerned about his weight. His body mass index is 38 kg/m^2. Which one of the following best describes his weight?

A. Morbidly obese (Obese III)

B. Normal

C. Overweight

D. Clinically obese (Obese II) √

E. Obese (Obese I)

Theme: Diabetes mellitus: management of type 2



A. < 130/70 mmHg

B. < 135/75 mmHg

C. < 140/80 mmHg

D. < 4.0 mmol/l

E. < 3.5 mmol/l

F. < 5.0 mmol/l

G. Atorvastatin

H. Ezetimibe

I. Fenofibrate



For each one of the following select the most appropriate answer

548. Target total cholesterol for type 2 diabetics < 4.0 mmol/l

549. Target blood pressure for type 2 diabetics with no end-organ damage < 140/80 mmHg

550. Should be prescribed if serum triglyceride levels are > 4.5 mmol/l Fenofibrate

551. A 55-year-old man with a history of type 2 diabetes mellitus, bipolar disorder and chronic obstructive pulmonary disease has bloods taken as part of his annual diabetic review: Na+= 129 mmol/l K+=3.8 mmol/l Bicarbonate=24 mmol/l Urea=3.7 mmol/l Creatinine 92 µmol/l Due to his smoking history a chest x-ray is ordered which is reported as normal. Which one of the following medications is most likely to be responsible?

A. Metformin √

B. Sodium valproate

C. Carbamazepine

D. Betamethasone dipropionate inhaler

E. Pioglitazone

552. Which one of the following drugs is not associated with galactorrhoea?

A. Metoclopramide

B. Bromocriptine √

C. Chlorpromazine

D. Haloperidol

E. Domperidone

553. A 25-year-old man with a history of Crohn's disease is reviewed. Over the past week he has developed painful perianal ulcers. On examination numerous shallow ulcers can be seen with a small number of skin tags. What is the most appropriate first-line treatment?

A. Topical mesalazine

B. Oral metronidazole √

C. Barrier creams + laxatives

D. Oral prednisolone

E. Oral mesalazine

554. A 42-year-old man of Afro-Caribbean origin is diagnosed as having hypertension. Secondary causes of hypertension have been excluded. What is the most appropriate initial drug therapy?

A. Losartan

B. Bisoprolol

C. Doxazosin

D. Perindopril

E. Amlodipine √

555. A 55-year-old man presents due to an uncontrollable urge to move his legs during the night-time. He has also experience the sensation of spiders crawling over his legs. Simple measures such as walking and massaging the affected limb have not alleviated the problem. What is the most appropriate medical therapy?



A. Selective serotonin reuptake inhibitor

B. Low-dose tricyclic antidepressant

C. Dopamine agonist √

D. 5-HT3 antagonist

E. Dopamine antagonist



556. A 18-year-old male presents with generalised lymphadenopathy. Which one of the following is least likely to result in this presentation?

A. Kawasaki disease √

B. Cytomegalovirus

C. Acute lymphoblastic leukaemia

D. Phenytoin therapy

E. Infectious mononucleosis

557. A 38-year-old man is reviewed in the respiratory clinic complaining of episodic wheezing whilst playing rugby. There is no history of cough, atopy or smoking. He is generally fit and well and has no past medical history of note. Clinical examination is unremarkable. Following history and examination it is thought he has an intermediate probability of asthma. Which one of the following is the most appropriate next investigation?

A. Spirometry √

B. Serial peak expiratory flow measurements

C. Histamine stimulation test

D. Methacholine stimulation test

E. A trial of inhaled steroids with FEV1 measurements before and after

558. A 76-year-old man is reviewed in the Elderly Medicine clinic. He is concerned about his increasing forgetfulness over the past six months. His daughter notes he has generally 'slowed down' and struggles to follow conversations. Over the past month he has noted increasingly frequent episodes of urinary incontinence. He has also had one episode of faecal incontinence in the past week. On examination he is noted to have brisk reflexes and a short, shuffling gait. No cerebellar signs are noted. What is the most likely diagnosis?

A. Multiple system atrophy

B. Parkinson's disease

C. Normal pressure hydrocephalus √

D. Urinary tract infection

E. Pick's disease



559. Each of the following features is seen in Marfan's syndrome, except:

A. Pectus excavatum

B. Tall stature

C. Mental retardation √

D. High-arched palate

E. Upwards lens dislocation

Theme: Causes of pruritus



A. Liver disease

B. Hypothyroidism

C. Diabetes mellitus

D. Menopause-related pruritus

E. Chronic kidney disease

F. Polycythaemia

G. Iron deficiency anaemia

H. Tuberculosis

I. Scabies

J. Lymphoma



For each of the following scenarios select the most likely diagnosis:

560. A 61-year-old man presents with pruritus. He has had recurrent episodes of painful swelling in the MTP joints and a history of peptic ulcer disease. On examination he has a 'ruddy' complexion Polycythaemia

561. A 41-year-old woman requests a repeat prescription for citalopram. She also mentions she is constantly itchy and bruises easily. On examination she has reddened palms and a distended abdomen Liver disease

562. A 37-year-old woman presents with itch and lethargy. She is having difficulty sleeping due to night sweats and is wondering if she may be 'going through the change'. Lymphoma

563. The first line treatment in amoebiasis is:

A. Albendazole

B. Hydroxychloroquine

C. Metronidazole √

D. Ciprofloxacin

E. Ivermectin

564. A 23-year-old female with a history of migraine presents for review. Her headaches are now occurring about once a week. She describes unilateral, throbbing headaches that may last over 24 hours. Neurological examination is unremarkable. Other than a history of asthma she is fit and well. What is the most suitable therapy to reduce the frequency of migraine attacks?



A. Propranolol

B. Zolmitriptan

C. Amitriptyline √

D. Carbamazepine

E. Pizotifen



565. A 52-year-old has a fasting lipid profile checked as part of an annual occupational health check. Combined with his blood pressure and current smoking status his 10-year risk of cardiovascular disease is calculated to be 23% percent. Following appropriate counselling he chooses to start simvastatin 40mg on. What should his target cholesterol be?

A. Total cholesterol:HDL ratio < 5

B. Total cholesterol < 5 mmol/l

C. Target cholesterol is inappropriate in this situation √

D. Total cholesterol < 4 mmol/l

E. Total cholesterol:HDL ratio < 4

566. Each one of the following is a recognised complication of gastro-oesophageal reflux disease, except:



A. Oesophageal carcinoma

B. Barrett's oesophagus

C. Anaemia √

D. Achalasia

E. Benign strictures



567. A 39-year-old man presents with shortness of breath following one week of flu-like symptoms. He also has a non-productive cough but no chest pain. A chest x-ray shows bilateral consolidation and examination reveals erythematous lesions on his limbs and trunk. Which one of the following investigations is most likely to be diagnostic?

A. Cold agglutins

B. Sputum culture

C. Urinary antigen for Legionella

D. Serology for Mycoplasma √

E. Blood culture

568. A 71-year-old man is reviewed following an ischaemic stroke. He is known to be allergic to aspirin. What is the most appropriate therapy to help reduce his chance of having a further stroke?

A. Lansoprazole + aspirin + dipyridamole. Stop dipyridamole after 2 years

B. Clopidogrel + dipyridamole. Stop dipyridamole after 2 years

C. Dipyridamole

D. Warfarin

E. Clopidogrel √

569. A 45-year-old female develops pleuritic chest pain following a hysterectomy 10 days ago. You admit her to the acute medical unit and a CTPA confirms a pulmonary embolism. There is no previous history of venous thromboembolism. How long should the patient be warfarinised for?

A. Not suitable for anticoagulation

B. 6 weeks - 3 months √

C. 6 months

D. 12 months

E. Life-long

570. A man presents with severe vomiting. He reports not being able to keep fluids down for the past 12 hours. You suspect a diagnosis of gastroenteritis and on discussing possible causes he mentions reheating curry with rice the night before. What is the most likely causative organism?

A. Escherichia coli

B. Campylobacter

C. Salmonella

D. Shigella

E. Bacillus cereus √

571. Which one of the following drugs is least associated with pancytopaenia?



A. Carbamazepine

B. Carbimazole

C. Lithium √

D. Gold

E. Chloramphenicol



Theme: Facial pain



A. Sinusitis

B. Dental abscess

C. Acute glaucoma

D. Temporal arteritis

E. Shingles

F. Cluster headache

G. Trigeminal neuralgia

H. Atypical facial pain

I. Temporomandibular joint dysfunction

J. Parotitis



For each one of the following scenarios select the most likely diagnosis:

572. A 64-year-old woman with a one week history of pain above and lateral to her left eye. On examination she is tender over that area.DGA Temporal arteritis

573. A 62-year-old woman presents with a two week history of shooting pains across her left cheek. The pain is sometimes triggered by touching her face. She has no past medical history note Trigeminal neuralgia

574. A 42-year-old man with a 3 month history of chronic cough presents with a persistent headache Sinusitis

575. A 27-year-old woman is reviewed due to sudden loss of vision in her left eye. She has is known to have severe rheumatoid arthritis and is currently treated with methotrexate, infliximab and prednisolone. For the past 6 weeks she has developed troublesome headaches. Examination demonstrates bilateral papilloedema. Which one of the following is most likely to be responsible for this presentation?

A. Acute angle-closure glaucoma

B. Prednisolone √

C. Infliximab

D. Methotrexate

E. Keratoconjunctivitis sicca

576. Which one of the following statements regarding dipeptidyl peptidase-4 inhibitors in the management of type 2 diabetes mellitus is correct?

A. Metformin should always be co-prescribed

B. Do not cause weight gain √

C. Is given via a subcutaneous injection

D. An example is exenatide

E. Patients should be warned that hypoglycaemia is the most common side-effect

577. Which of the following drugs is considered most likely to precipitate an attack of acute intermittent porphyria?



A. Morphine

B. Aspirin

C. Atenolol

D. Metformin

E. Oral contraceptive pill √



578. Each one of the following is a feature of organophosphate poisoning, except:

A. Defecation

B. Mydriasis √

C. Salivation

D. Lacrimation

E. Urination





579. Which one of the following conditions has polygenic inheritance?

A. Bartter's syndrome

B. Huntington disease

C. Ankylosing spondylitis √

D. Fragile X syndrome

E. Von Willebrand's disease

580. A 30-year-old woman presents with abdominal pain that is associated with alternating diarrhoea and constipation. Which one of the following symptoms is least consistent with a diagnosis of irritable bowel syndrome?

A. Feeling of incomplete stool evacuation

B. Waking at night due to the pain √

C. Abdominal bloating

D. Faecal urgency

E. Passage of mucous with stool

581. A 34-year-old male is admitted with central abdominal pain radiating through to the back and vomiting. The following results are obtained: Amylase= 1,245 u/dl

Which one of the following medications is most likely to be responsible?

A. Phenytoin

B. Sodium valproate √

C. Metoclopramide

D. Sumatriptan

E. Pizotifen

582. A 63-year-old man is prescribed minoxidil for resistant hypertension. Which one of the following side-effects is not recognised?

A. Fluid retention

B. Tachycardia

C. Alopecia √

D. Rise in urea

E. Weight gain

583. A 24-year-old woman presents for advice. Over the past few months she has been having increasing problems with migraine around the time of menstruation. Her current migraine started around 24 hours ago and has not responded to a combination of paracetamol and aspirin. What is the most appropriate next step to relieve her headache?

A. Codeine

B. Ergotamine

C. Sumatriptan √

D. Venlafaxine

E. Norethisterone

Theme: Cytotoxic agents: side-effects



A. Doxorubicin

B. Cisplatin

C. Methotrexate

D. Dactinomycin

E. Vincristine

F. Fludarabine

G. Bleomycin

H. Cyclophosphamide

I. Paclitaxel

J. Pentostatin



For each of the following side-effects please select the cytotoxic agent which is most likely to be responsible:

584. Lung fibrosis Bleomycin

585. Myelosuppression, liver fibrosis and oral mucositis Methotrexate

586. Peripheral neuropathy in a patient being treated for lymphoma Vincristine

587. A 35-year-old woman presents with menorrhagia and a persistent sore throat. A full blood count shows her to be pancytopenic. Which one of the following medications is most likely to account for this finding?

A. Trimethoprim √

B. Rifampicin

C. Olanzapine

D. Montelukast

E. Clomifene

588. You are doing a medication review on a patient with chronic kidney disease who is prescribed metformin. At what creatinine value do NICE recommend stopping metformin?



A. 110 µmol/l

B. 120 µmol/l

C. 130 µmol/l

D. 140 µmol/l

E. 150 µmol/l √







589. A rise in alkaline phosphotase can be caused by each one of the following except:

A. Pregnancy

B. Paget's disease

C. Healing bone fractures

D. Osteomalacia

E. Hypoparathyroidism √

590. Which one of the following drugs is least likely to cause gynaecomastia?

A. Spironolactone

B. Sodium valproate √

C. Digoxin

D. Cimetidine

E. Anabolic steroids

591. A patient presents as she has a strong family history of cancer. Which one of the following cancers is least likely to be inherited?

A. Colorectal cancer

B. Breast cancer

C. Gastric cancer √

D. Endometrial cancer

E. Ovarian cancer

592. A 72-year-old woman is brought to surgery with confusion and pallor. Her daughter reports that she has been getting more confused are tired for the past three months. Blood tests are reported as follows: Hb= 8.9 g/dl MCV= 125 fl Plt= 148 * 109/l WBC= 4.4 * 109/l; In light of the macrocytic anaemia some further tests are ordered: Intrinsic factor antibodies= Negative Vitamin B12= 94 ng/l (200-900 ng/l) Folic acid= 1.1 nmol/l (> 3.0 nmol/l) What is the most appropriate management?

A. Perform an ECG immediately

B. Oral folic acid + start Intramuscular vitamin B12 when folic acid levels are normal

C. Intramuscular vitamin B12 + start oral folic acid when vitamin B12 levels are normal √

D. Refer to the local alcohol dependency services

E. Admit for blood transfusion

593. Which of the following is not a recognised complication of coeliac disease?

A. Hypersplenism √

B. Lymphoma of the small intestine

C. Osteoporosis

D. Oesophageal cancer

E. Subfertility

594. Which one of the following agents is most useful in the maintenance of sinus rhythm in patients with atrial fibrillation?

A. Verapamil

B. Diltiazem

C. Ibutilide

D. Amiodarone √

E. Digoxin

595. Which one of the following drugs is used in the management of multiple sclerosis?

A. Beta-interferon √

B. Gamma-interferon

C. Infliximab

D. Rituximab

E. Alpha-interferon

Theme: Dysphagia



A. Pharyngeal pouch

B. Achalasia

C. Globus hystericus

D. Systemic sclerosis

E. Oesophageal cancer

F. Myasthenia gravis

G. Oesophagitis

H. Motor neuron disease

I. Oesophageal candidiasis

J. Plummer-Vinson syndrome



For each one of the following scenarios please select the most likely diagnosis:

596. A 54-year-old man presents with a 3 month history of difficultly swallowing. He has noticed that swallowing is painful, particularly when he eats meat or bread. After eating and at night he has an 'unpleasant' retrosternal sensation. Clinical examination is unremarkable GED Oesophagitis

597. A 47-year-old woman presents with a 5 week history of food getting stuck. She is currently treated for COPD and was recently noted to have a macrocytosis on routine bloods. On a number of occasions she has vomited during the meal and says she has no taste for food any more. Oesophageal cancer

598. A 43-year-old woman with a history of anxiety complains of problems swallowing. On examination she is noted to have a number of small white lumps on her hands and telangiectasia on her face Systemic sclerosis

599. Which of the following is least recognised as a cause of macroglossia?

A. Sarcoidosis √

B. Acromegaly

C. Hypothyroidism

D. Hurler syndrome

E. Amyloidosis

600. A 54-year-old man presents with a persistent tremor. On examination there is 6-8 Hz tremor of the arms which is worse when his arms are outstretched. His father suffered from a similar complaint. What is the most suitable first-line treatment?

A. Amitriptyline

B. Propranolol √

C. D-penicillamine

D. Levodopa

E. Diazepam

601. A 61-year-old presents for review. She has been having atypical lower back pain for the past two months. An x-ray of her lumbar spine reported raised the possibility of spinal metastases but there is no current evidence of a primary tumour. A series of tumour markers were sent and a referral to oncology was made. Which one of the following is most associated with raised levels of CA 15-3?

A. Pancreatic cancer

B. Lung cancer

C. Breast cancer √

D. Ovarian cancer

E. Hepatocellular carcinoma

602. Which one of the following is least recognised as an adverse effect of phenytoin use?



A. Megaloblastic anaemia

B. Peripheral neuropathy

C. Alopecia √

D. Osteomalacia

E. Coarsening of facial features



603. A 65-year-old life-long smoker with a significant past history of asbestos exposure is investigated for lung cancer. Given his history of both smoking and asbestos exposure, what is his increased risk of lung cancer?

A. 5

B. 10

C. 50 √

D. 500

E. 1,000

604. A 72-year-old man is investigated for exertional chest pain and has a positive exercise tolerance test. He declines an angiogram and is discharged on a combination of aspirin 75mg od, simvastatin 40mg on, atenolol 50mg od and a GTN spray prn. Examination reveals a pulse of 72 bpm and a blood pressure of 130/80 mmHg. On review he is still regularly using his GTN spray. What is the most appropriate next step in management?

A. Add nifedipine MR 30mg od

B. Add isosorbide mononitrate 30mg bd

C. Increase atenolol to 100mg od √

D. Add nicorandil 10mg bd

E. Add verapamil 80mg tds

605. Which one of the following is least likely to precipitate haemolysis in a patient with G6PD deficiency?

A. Broad beans

B. Sepsis

C. Ciprofloxacin

D. Primaquine

E. Penicillin √

606. Which one of the following side-effects is most associated with ciclosporin use?

A. Hepatotoxicity √

B. Bone marrow toxicity

C. Red cell aplasia

D. Haemorrhagic cystitis

E. Tinnitus







Theme: Adverse effects of drugs used in rheumatoid arthritis drugs



A. Diclofenac

B. Etanercept

C. Sulfasalazine

D. Rituximab

E. Methotrexate

F. Infliximab

G. Adalimumab

H. Abatacept

I. Gold

J. Penicillamine



Please select the drug most associated with the following adverse effects:

607. Pneumonitis Methotrexate

608. Demyelination Etanercept

609. Proteinuria Gold

610. A 48-year-old female who has just completed a course of chemotherapy complains of difficulty using her hands associated with 'pins and needles'. She has also experienced urinary hesitancy. Which cytotoxic drug is most likely to be responsible?

A. Doxorubicin

B. Cyclophosphamide

C. Methotrexate

D. Vincristine √

E. Bleomycin

611. A 59-year-old man with a known history of type 2 diabetes mellitus, atrial fibrillation and epilepsy presents as he is feeling generally unwell. His main complaint is a blue tinge to his vision. Which one of his medications is most likely to be responsible?

A. Phenytoin

B. Metformin

C. Sildenafil √

D. Pioglitazone

E. Digoxin

612. A 59-year-old man with a known history of type 2 diabetes mellitus, atrial fibrillation and epilepsy presents as he is feeling generally unwell. His main complaint is a yellow tinge to his vision. Which one of his medications is most likely to be responsible?



F. Phenytoin

G. Metformin

H. Sildenafil

I. Pioglitazone

J. Digoxin √



613. Which one of the following drugs used in the management of diabetes mellitus is most likely to cause cholestasis?

A. Metformin

B. Gliclazide

C. Acarbose

D. Rosiglitazone √

E. Insulin

614. A 62-year-old woman with a history of recurrent deep vein thrombosis secondary to antiphospholipid syndrome presents for review. She has takes warfarin for the past 7 years, with a target INR of 2.0 - 3.0. Her control is normally very good but her last reading was 1.2. Which one of the following would explain her current INR?

A. Starting fluoxetine for depression

B. The formation of lupus anticoagulant autoantibodies

C. Giving up smoking

D. Recent rifampicin as she was a contact of a patient with meningococcal meningitis √

E. A course of ciprofloxacin for a urinary tract infection

615. Which of the following drugs is least likely to cause cholestasis?

A. Anabolic steroids

B. Erythromycin

C. Prochlorperazine √

D. Halothane

E. Flucloxacillin

616. Which one of the following increases the risk of developing peripheral oedema in a patient taking pioglitazone?

A. Concomitant use with gliclazide

B. Serum sodium < 140 mmol/l

C. Concomitant use with insulin √

D. Concomitant use with metformin

E. Serum potassium < 4.0 mmol/l





617. One of your patients who is an intravenous drug user contracts the hepatitis C virus. What percentage of such patients will become chronically infected with hepatitis C?

A. 30-35%

B. 80-85% √

C. 50-60%

D. 5-10%

E. 15-20%

618. A 74-year-old man with symptomatic aortic stenosis is reviewed in the cardiology clinic. He is otherwise fit and well and keen for intervention if possible. What type of intervention is he most likely to be offered?

A. Annual echocardiography, intervention when valve gradient > 75 mmHg

B. Aortic bypass graft √

C. Bioprosthetic aortic valve replacement

D. Balloon valvuloplasty

E. Mechanical aortic valve replacement

619. A middle-aged man with type 2 diabetes mellitus is reviewed. Despite weight loss, metformin and gliclazide his HbA1c is 8.4%. The patient agrees to start insulin therapy. According to NICE guidelines which type of insulin should be tried initially?

A. Basal bolus regime

B. Isophane √

C. Biphasic insulin

D. Glargine

E. Detemir

Theme: Vitamin deficiency



A. Vitamin A

B. Thiamine

C. Niacin

D. Pyridoxine

E. Folic acid

F. Vitamin B12

G. Vitamin C

H. Vitamin D

I. Vitamin E

J. Vitamin K



For each one of the following scenarios select the vitamin which may cause these features if deficient:

620. Wernicke-Korsakoff syndrome Thiamine

621. Neural tube defects Folic acid

622. Haemorrhagic disease of the newborn Vitamin K

623. What is the target INR for a patient with a mechanical mitral valve?

A. 4.0-4.5

B. 4.0

C. 3.0-4.0

D. 2.5-3.5 √

E. 2.0-3.0

624. A 68-year-old woman comes back for review. Two weeks ago she presented with pain in her left knee not responding to paracetamol and was commenced on diclofenac 50mg tds and lansoprazole 30mg od. Shortly afterwards she developed some indigestion which seems to resolve if she skips the diclofenac dose. She is otherwise asymptomatic and got good pain relief from diclofenac. Clinical examination is normal. What is the most appropriate action?

A. 13C-urea breath test

B. Stop diclofenac, continue lansoprazole + review in 1 week √

C. Switch diclofenac to ibuprofen, continue lansoprazole

D. Refer urgently for endoscopy

E. Admit

625. Which one of the following is characteristic of Mikulicz's syndrome?

A. Erythema nodosum

B. Enlargement of the parotid glands √

C. Cholestatic jaundice

D. Urethral discharge

E. Goitre

626. Which one of the following causes of diarrhoea has the shortest incubation period?

A. Salmonella

B. Shigella

C. Campylobacter

D. Escherichia coli

E. Bacillus cereus √





627. A 55-year-old female is reviewed in the diabetes clinic. The following results are obtained:

Urinalysis= protein + HbA1c= 10.0% What average blood glucose level for the past 2 months is this most likely to represent?

A. 9

B. 10

C. 11

D. 15 √

E. There is no relation between HbA1c and average blood glucose

628. A 55-year-old business man presents with a 15 day history of watery, non-bloody diarrhoea associated with anorexia and abdominal bloating. His symptoms started 4 days after returning from a trip to Pakistan. On examination he is apyrexial with dry mucous membranes but normal skin turgor. What is the most likely causative organism?

A. Salmonella

B. Giardia lamblia √

C. Shigella

D. Escherichia coli

E. Norovirus

629. Which one of the following markers is most useful for monitoring the progression of patients with chronic obstructive pulmonary disease?

A. FEV1/FVC ratio

B. Lifestyle questionnaire

C. Oxygen saturations

D. FEV1 √

E. Number of exacerbations per year

630. A 19-year-old with 'brittle asthma' is seen in clinic. Three weeks ago she started taking prednisolone 15mg od as her asthma was poorly controlled on beclometasone dipropionate 800 mcg bd., salmeterol, oral montelukast and salbutamol as required. What should happen with regards to inhaled steroids?

A. Continue beclometasone dipropionate 800 mcg bd

B. Stop inhaled steroids

C. Increase beclometasone dipropionate to 1000 mcg bd √

D. Decrease beclometasone dipropionate to 400 mcg bd

E. Use beclometasone dipropionate 200 mcg on an 'as required' basis with salbutamol

631. A 62-year-old woman who is known to have metastatic breast cancer presents with increasing shortness of breath. She is currently receiving a chemotherapy regime. On examination she has a third heart sound and the apex beat is displaced to the 6th intercostal space, anterior axillary line. Which one of the following chemotherapeutic agents is most likely to be responsible?

A. Paclitaxel

B. Docetaxel

C. Bleomycin

D. Dactinomycin

E. Doxorubicin √

Theme: Parkinson's disease: management



A. Levodopa

B. Amantadine

C. Entacapone

D. Procyclidine

E. Ropinirole

F. Cabergoline



For each one of the following select the most appropriate answer from the options listed above:

632. Useful for managing tremor in drug-induced parkinsonism Procyclidine

633. Has been associated with pulmonary fibrosis Cabergoline

634. Often has a reduced effectiveness with time Levodopa

635. A 24-year-old female presents to her GP with lethargy and dizzy spells. On examination she is noted to have an absent left radial pulse. Blood tests are as follows:Na+= 136mmol/l K+= 4.1 mmol/l Urea= 2.3 mmol/l Creatinine= 77µmol/l ESR= 66 mm/hr

What is the most likely diagnosis?

A. Turner's syndrome

B. Takayasu's arteritis √

C. Kawasaki disease

D. Systemic lupus erythematous

E. Breast carcinoma with local spread





636. Each one of the following is associated with hirsuitism, except:

A. Porphyria cutanea tarda √

B. Congenital adrenal hyperplasia

C. Polycystic ovarian syndrome

D. Adrenal tumour

E. Cushing's syndrome

637. Which one of the following features is least associated with ecstasy poisoning?

A. Rhabdomyolysis

B. Hyperthermia

C. Ataxia

D. Hypertension

E. Hypernatraemia √

638. A 45-year-old man is reviewed in the diabetic clinic. His current medication is basal bolus insulin regime combined with Ramipril 10mg OD, Simvastatin 40mg OD and Aspirin 75mg OD. On examination blood pressure is 134/84 mmHg. The following results are obtained:

HbA1c 7.3% Albumin: Creatinine ratio=10.5 What is the most appropriate change to his treatment?

A. Stop Aspirin

B. Increase dose of Ramipril

C. Add an angiotensin-II receptor antagonist √

D. Add Doxazosin

E. Stop Ramipril

639. A 28-year-old man develops nausea and a severe headache whilst trekking in Nepal. Within the next hour he becomes ataxic and confused. A diagnosis of high altitude cerebral oedema is suspected. Other than descent and oxygen, what is the most important treatment?

A. Acetazolamide

B. Dexamethasone √

C. Burr hole

D. Mannitol

E. Frusemide

640. Thrombocytopenia is associated with each of the following drugs except:

A. Abciximab

B. Quinine

C. Warfarin √

D. Penicillin

E. Sodium valproate

641. Which one of the following combinations of treatments should be avoided in patients with type 2 diabetes mellitus?

A. Metformin + insulin + exenatide √

B. Sulfonylurea + DPP-4 inhibitor

C. Metformin + sulfonylurea + exenatide

D. Metform + DPP-4 inhibitor

E. Insulin + metformin + sulfonylurea

642. A 71-year-old woman with metastatic breast cancer comes to surgery with her husband. She is known to have spinal metastases but her back pain is not controlled with a combination of paracetamol, diclofenac and MST 30mg bd. Her husband reports she is using 10mg of oral morphine solution around 6-7 times a day for breakthrough pain. The palliative care team at the hospice tried using a bisphosphonate but this unfortunately resulted in persistent myalgia and arthralgia. What is the most appropriate next step?

A. Switch to oxycodone

B. Increase MST

C. Increase MST + add dexamethasone √

D. Increase MST + suggest course of complimentary therapies

E. Increase MST + refer for radiotherapy

643. Each one of the following is a risk factor for gastric cancer, except:

A. Blood group A

B. Pernicious anaemia

C. Pylori infection

D. Smoking √

E. History of duodenal ulceration





644. A 55-year-old man is diagnosed with motor neuron disease. Which one of the following drugs has been shown to confer a survival benefit?

A. Rituximab

B. Riluzole √

C. Interferon-beta

D. Cyclophosphamide

E. Interferon-alpha

645. Each one of the following is associated with non-alcoholic steatohepatitis, except:

A. Hyperlipidaemia

B. Obesity

C. Sudden weight loss or starvation

D. Jejunoileal bypass

E. Type 1 diabetes mellitus √

646. A 33-year-old woman who is known to have familial hypercholesterolaemia comes for review. She is planning to have children and asks for advice regarding medication as she currently takes atorvastatin 80mg on. What is the most appropriate advice?

A. Switch to atorvastatin 10mg

B. Continue current drug at same dose

C. Stop atorvastatin before trying to conceive √

D. Switch to ezetimibe

E. Switch to simvastatin 40mg

647. A 49-year-old man with type 2 diabetes mellitus is reviewed. Despite weight loss and therapy with metformin and gliclazide his last HbA1c is 7.2%. Which one of the following factors would suggest that the patient may benefit from a meglitinide?

A. Obesity

B. Not adhering to diabetic diet

C. Problems with hypoglycaemia from gliclazide

D. Erratic lifestyle √

E. Elderly and frail patients

648. A 60-year-old woman being treated for breast cancer is complaining of fatigue, diarrhea, and hair loss. Which one of the following statements about these adverse effects from chemotherapy is correct?

A. Anemia is typically found within the first week following chemotherapy.

Alopecia usually does not occur for several weeks after therapy.
Erythropeitin is recommended for anemia when the hemoglobin level is above 12 g per dl (120 g per L).
Loperamide can be used to treat chemotherapy-related diarrhea. √
None of the above statements is correct

1 comment:

feed